FP513 Investing Flashcards

You may prefer our related Brainscape-certified flashcards:
1
Q

All of the following correctly identify advantages of U.S. Treasury bills except

A) investors can tailor purchases to meet short-term goals and obligations.

B) investors are provided a high degree of safety.

C) they are not subject to default risk.

D) interest income is not subject to federal income tax.

A

D)
Explanation
Interest income from U.S. Treasury bills is taxed at ordinary federal income tax rates but is not subject to state income tax.

LO 1.3.1

How well did you know this?
1
Not at all
2
3
4
5
Perfectly
2
Q

Which of the following is NOT a characteristic of negotiable CDs?

A) Negotiable CDs are bought and sold in the secondary market at a market-determined price.
B) They are used as short-term drafts drawn to finance imports and exports.
C) They are deposits of $100,000 or more placed with commercial banks at a specified interest rate.
D) Negotiable CDs are bought most often by institutional investors rather than by individuals.

A

B)

Banker’s acceptances are short-term drafts drawn by a private company on a major bank used to finance imports and exports.

LO 1.3.1

How well did you know this?
1
Not at all
2
3
4
5
Perfectly
3
Q

Which of the following terms is considered early-stage business funding for the purpose of research and development of an idea?

A) Seed financing
B) Start-up financing
C) First-stage financing
D) Bridge financing

A

A) Explanation
Bridge financing is for firms that expect to go public within approximately one year. First-stage financing is for initial manufacturing and sales. Start-up financing is for product development and marketing for firms who have not sold products or services commercially.

LO 1.1.1

How well did you know this?
1
Not at all
2
3
4
5
Perfectly
4
Q

All of the following statements correctly describe certificates of deposit (CDs) except:

A) redemption prior to maturity typically results in an early withdrawal penalty.

B) CDs are eligible for FDIC coverage.

C) CDs are commonly referred to as time deposits.

D) CDs typically pay a variable interest rate based on the term of the certificate.

A

D). Explanation
CDs typically pay a fixed interest rate, with higher interest rates offered for longer-term certificates.

LO 1.3.1

How well did you know this?
1
Not at all
2
3
4
5
Perfectly
5
Q

Lloyd is a dealer in government securities. He has purchased government securities from another dealer, Fred, and has agreed to sell them back at a later date. From Lloyd’s perspective, which transaction has been executed?

A) Repurchase agreement
B) Reverse repurchase agreement
C) Commercial paper investment
D) Promissory note

A

B) Explanation
Lloyd, as the buyer, has entered into a reverse repurchase agreement, and Fred, as the seller, has entered into a repurchase agreement.

LO 1.3.1

How well did you know this?
1
Not at all
2
3
4
5
Perfectly
6
Q

All of the following correctly identify features of limited partnerships except

A) the limited partners may participate in the management of the partnership.

B) the limited partners have limited liability.

C) the general partner controls the business activities of the partnership.

D) the general partner determines when distributions are made to the limited partners.

A

A)
The disadvantages of limited partnerships include:

(1) they are generally riskier than bonds or exchange-traded equities;
(2) they are generally illiquid;
(3) limited partners cannot participate in the management; and
(4) the sale of partnership interest may be restricted. In addition, the general partner has unlimited liability.

LO 1.1.1

How well did you know this?
1
Not at all
2
3
4
5
Perfectly
7
Q

Which of the following statements best describes banker’s acceptances?

A) Promissory notes traded at a premium from their face value in the secondary market

B) Short-term drafts drawn by a private company on a major bank used to finance imports and exports

C) U.S. dollar-denominated deposits at banks outside the United States

D) Negotiable, short-term, unsecured promissory notes issued by large corporations

A

B) Explanation
Banker’s acceptances are typically traded at a discount from their face value in the secondary market. Eurodollars are U.S. dollar-denominated deposits at banks outside the United States.

LO 1.3.1

How well did you know this?
1
Not at all
2
3
4
5
Perfectly
8
Q

Greg calls his broker and tells her to sell his XYZ stock if it falls to $20, but he does not want less than $19.50 for his shares. What type of order should his broker recommend to sell the stock?

A) Limit order

B) Stop limit order

C) Good-till-canceled order

D) Market order

A

B) Explanation
The stop limit order turns into a limit order when triggered (both the stop order price and the limit order price are specified). However, this type of order will not guarantee execution if the stock leapfrogs below the $19.50 mark.

LO 1.2.1

How well did you know this?
1
Not at all
2
3
4
5
Perfectly
9
Q

Choose the risk that is attributable to cash and cash equivalents.

A) None of these because cash and cash equivalents are considered risk-free
B) Purchasing power risk
C) Marketability risk
D) Liquidity risk

A

B) Explanation
Cash and cash equivalents are subject to purchasing power (inflation) risk because they offer limited potential for growth.

LO 1.3.1

How well did you know this?
1
Not at all
2
3
4
5
Perfectly
10
Q

Identify which of the following statements regarding U.S. Treasury bills is CORRECT.

I. They are purchased at 50% of face value.
II. They are sold at auction in denominations ranging from $50 to $10,000.
III. They are not subject to the original issue discount (OID) taxation rules.
IV. They are sold with maturities up to two years.

A) I, II, and III
B) I and II
C) II and IV
D) III only

A

Explanation
D) U.S. Treasury bills are purchased at a discount to face value determined at auction. The lowest purchase amount is $100. They are not subject to the OID rules. T-bills have a maturity date of no more than one year.

LO 1.3.1

How well did you know this?
1
Not at all
2
3
4
5
Perfectly
11
Q

Select the market designed to facilitate the initial sale of securities to the public.

A) Third market
B) Primary market
C) Fourth market
D) Secondary market

A

B) Explanation
The purpose of the primary market is to facilitate the sale of initial public offerings (IPOs) of securities to the public.

LO 1.2.1

How well did you know this?
1
Not at all
2
3
4
5
Perfectly
12
Q

Robert owns 400 shares of Intel stock that he purchased several years ago for $60 per share. Intel’s current market price is $48 per share. On December 17, Robert decides to buy an additional 200 shares of Intel stock. On December 23, he decides to sell 200 shares that he purchased several years ago so that he can claim a loss on his current year’s tax return. Which of the following statements is true?

I. The loss will be disallowed, but Robert will have to reduce his tax basis in the shares he purchased on December 17 by the amount of the loss.

II. The loss will be disallowed; the transactions are illegal and tax penalties will be imposed.

III. The loss will be disallowed; the amount of the disallowed loss will be added to the cost basis of the shares purchased on December 17.

IV. The transaction is called a wash sale; wash sale rules apply when shares are sold for a loss and repurchased within 30 days before or after the sale date.

A) III and IV
B) IV only
C) II and IV
D) I only

A

A) Explanation
The transaction is a wash sale; losses are disallowed when substantially identical shares are repurchased within 30 days before or after a sale. The transaction is not illegal and no tax penalties are imposed on the transaction itself. The basis of the stock is adjusted for the disallowed loss.

LO 1.4.1

How well did you know this?
1
Not at all
2
3
4
5
Perfectly
13
Q

Which of the following statements regarding Eurodollar CDs is CORRECT?

I. Eurodollar CDs are obligations of non-U.S. banks.
II. Eurodollar CDs are more liquid than domestic CDs.
III. Eurodollar CDs offer a slightly higher yield than domestic CDs.
IV. Eurodollar CDs are only used to settle transactions in the U.S.

A) I only
B) II and IV
C) I and III
D) I, II, and III

A

Explanation
C) Statements II and IV are incorrect. Eurodollar CDs are less liquid than domestic CDs and are used to settle international transactions.

LO 1.3.1

How well did you know this?
1
Not at all
2
3
4
5
Perfectly
14
Q

Which of the following is an unsecured line of credit provided to a business typically to finance imports and exports?

A) Banker’s acceptance
B) Reverse repurchase agreement
C) Commercial paper
D) Negotiable CD

A

A) Explanation
A banker’s acceptance is an unsecured line of credit provided to a business customer, typically used to finance imports and exports.

LO 1.3.1

How well did you know this?
1
Not at all
2
3
4
5
Perfectly
15
Q

Which of the following statements best describes Eurodollars?

A) Deposits of $100,000 or more placed with commercial banks at a specified interest rate

B) Negotiable, short-term, unsecured promissory notes issued by large corporations

C) U.S. dollar-denominated deposits at banks outside the United States

D) Short-term drafts drawn by a private company on a major bank used to finance imports and exports

A

C)
Explanation
In addition, the average deposit is very large (in the millions) and has a maturity of less than six months.

LO 1.3.1

How well did you know this?
1
Not at all
2
3
4
5
Perfectly
16
Q

Limited partnerships are distinguished by which of the following?

I. The general partner controls the business activities of the partnership.

II. The limited partners participate in the business venture with limited liability.

III. The general partner determines when distributions are made to the limited partners.

IV. The limited partners may have difficulty selling their interests.

A) I, II, III, and IV
B) I and III
C) I, II, and III
D) II and IV

A

Explanation
All of these statements are correct. Limited partnerships are characterized by a partnership entity that consists of a general partner and limited partners.

LO 1.1.1

How well did you know this?
1
Not at all
2
3
4
5
Perfectly
17
Q

Aidan purchased 100 shares of MNO stock on margin three years ago when the stock price was $32 per share. Today MNO stock is selling for $42 per share. Over the past three years, MNO has paid total dividends of $1 per share.

Assuming Aidan’s broker requires an initial margin of 50% and charges 6% annual margin interest, calculate his holding period return for the three years.

A) 68.75%
B) 50.75%
C) 31.38%
D) 62.75%

A

B) Explanation
HPR = [(ending value – beginning value) +/– cash flows] ÷ initial investment

= {[($4,200 – $3,200) + $100] – [6% × 50% × $3,200 × 3]} ÷ $1,600 = 50.75%.

LO 1.2.1

How well did you know this?
1
Not at all
2
3
4
5
Perfectly
18
Q

Your client has just opened a margin account with your brokerage firm and purchased 500 shares of stock for $60 per share. The firm has a 55% initial margin and 35% maintenance margin policy. Calculate the stock price at which your client will receive a margin call.

A) $50.76
B) $41.54
C) $31.43
D) $27.00

A

B) Explanation
The client will receive a margin call when the price of the stock drops to $41.54, calculated as follows:

margin call = ($60 × 0.45) ÷ (1 – 0.35)

margin call = $27.00 ÷ 0.65 = $41.5385, or $41.54

LO 1.2.1

How well did you know this?
1
Not at all
2
3
4
5
Perfectly
19
Q

Which of the following statements describes the purpose of holding cash and cash equivalents?

I. They provide a totally risk-free investment that can safeguard the asset values of a retired individual.

II. They supply highly liquid investments that provide funds for financial emergencies.

A) II only
B) I only
C) Neither I nor II
D) Both I and II

A

A) Explanation
Cash and cash equivalents are highly liquid investments that are generally included in a client’s emergency fund. Because of their low returns, cash and cash equivalents are subject to purchasing power risk, making them inappropriate for protecting asset value.

LO 1.3.1

How well did you know this?
1
Not at all
2
3
4
5
Perfectly
20
Q

Settings
Arrange, in order, the steps required to complete a short sale transaction.

I. The investor repurchases the stock in the open market.

II. The investor replaces, or covers, the borrowed stock.

III. The investor uses a stockbroker to borrow the stock from another investor’s account.

IV. The investor sells the borrowed stock in the open market.

A) I, II, III, IV
B) IV, I, II, III
C) I, III, IV, II
D) III, IV, I, II

A

D) Explanation
To complete a short sale transaction, the investor uses a stockbroker to borrow the stock from another investor’s account, the investor sells the borrowed stock in the open market, the investor repurchases the stock in the open market, and finally, the investor replaces, or covers, the borrowed stock.

LO 1.2.1

How well did you know this?
1
Not at all
2
3
4
5
Perfectly
21
Q

When investment bankers absorb the loss on an initial public offering, which one of the following terms represents this type of offering?

A) Green shoes
B) Firm commitment
C) Best efforts
D) Secondary offering

A

Explanation
B) The answer is firm commitment. Firm commitment underwriting occurs when investment bankers purchase all shares from a company and resell them to the public at their own risk.

LO 1.1.1

How well did you know this?
1
Not at all
2
3
4
5
Perfectly
22
Q

The maintenance margin is

A) usually greater than the initial margin percentage.
B) the amount owed to the broker/dealer.
C) a requirement once the investor takes a margin account position.
D) not a factor in a margin call.

A

Explanation
C) The answer is a requirement once the investor takes a margin account position. The maintenance margin is the minimum required percentage of cash equity in the position.

LO 1.1.1

How well did you know this?
1
Not at all
2
3
4
5
Perfectly
23
Q

Equity investments made for the launch, early development, or expansion of a business are known as

A) leveraged buyouts.
B) venture capital.
C) mezzanine financing.
D) distressed debt investing.

A

Explanation
The answer is venture capital. Equity financing associated with the early development of a business is called venture capital. Mezzanine financing is provided for expansion and new products. Leveraged buyout financing is provided to allow management to buy all or part of a business; often used when a public company divests a division that it feels is no longer part of its long-term plans. Distressed debt is investing in the debt of companies that are in trouble or failing.

LO 1.1.1

How well did you know this?
1
Not at all
2
3
4
5
Perfectly
24
Q

Short selling is selling

A) borrowed securities.
B) stock owned for less than a year.
C)an odd lot.
D) against the advice of an investor’s broker.

A

Explanation
A) The answer is borrowed securities. To sell short, an investor must first borrow shares from the brokerage firm’s customers.

LO 1.1.1

How well did you know this?
1
Not at all
2
3
4
5
Perfectly
25
Q

A strategy where investors with relatively large amounts of money to invest purchase multiple certificates with varying terms to maturity is

A) swapping.
B) staging.
C) laddering.
D) bulleting.

A

Explanation
C) Laddering is the process of purchasing multiple CDs with staggered maturities, that are equally spaced, and with varying interest rates. As each CD matures, a CD is purchased with a maturity equal to the longest in the ladder. This strategy is used to manage interest rate risk.

LO 1.1.1

How well did you know this?
1
Not at all
2
3
4
5
Perfectly
26
Q

Which market is designed to facilitate the initial sale of securities to the public?

A) Third market
B) Secondary market
C) Primary market
D) Fourth market

A

Explanation
The answer is primary market. The purpose of the primary market is to facilitate the sale of initial public offerings (IPOs) of securities to the public.

LO 1.2.1

How well did you know this?
1
Not at all
2
3
4
5
Perfectly
27
Q

Alice uses a stockbroker to borrow shares of stock from another investor’s account and then sells the borrowed stock in the open market. She later repurchases the stock in the open market and replaces, or covers, the borrowed stock. Which type of transaction has Alice used in her account?

A) Wash sale
B) Zero-cost collar
C) Protective put
D) Short sale

A

Explanation
D) The answer is short sale. A short sale involves the sale and subsequent repurchase of borrowed shares, allowing an investor to take advantage of falling stock prices.

LO 1.2.1

How well did you know this?
1
Not at all
2
3
4
5
Perfectly
28
Q

Your client has owned ABC Corp. stock for several years, and it has risen in price to $46 per share. She wants to continue owning the stock as long as it is going up in price but wants to protect herself on the downside and get out of the position if the stock falls in value to $38 or less per share. In order to do this, she would need to place which kind of order?

A) Good-til-canceled sell order at $38
B) Good-til-canceled sell stop at $38
C) Good-til-canceled sell stop limit at $38
D) Day order to sell at $38

A

Explanation

B) The answer is good-til-canceled sell stop at $38. A good-til-canceled order is an order that will remain in effect until either it is executed or canceled. A good-til-canceled sell stop order becomes a market sell order when the market price drops below the stop price.

LO 1.2.1

How well did you know this?
1
Not at all
2
3
4
5
Perfectly
29
Q

Carly purchased $80,000 of JEM stock for $40 per share utilizing her margin account. She used $40,000 in her money market fund plus she borrowed $40,000 from her broker. She acquired a total of 2,000 shares of JEM stock. JEM stock is currently trading at $39.65 per share. Calculate the stock price that Carly would receive a margin call from her broker. Assume a maintenance margin requirement of 35% and an initial margin requirement of 50%.

A) $29.68
B) $30.50
C) $30.77
D) $30.23

A

Explanation

C) The answer is $30.77. Carly would receive a margin call when the stock fell to $30.77 per share.

Margin call = [(1 − initial margin percentage) ÷ (1 − maintenance margin)] × purchase price of the stock = [(1 – 0.50) ÷ (1 – 0.35)] × 40 = 30.7692, or $30.77.

LO 1.2.1

How well did you know this?
1
Not at all
2
3
4
5
Perfectly
30
Q

Cosmo has a margin account with a balance of $50,000 with a national broker-dealer. The initial margin requirement on this account is 50%. Cosmo is interested in purchasing shares of Aardvark Inc., which is currently selling at $40 per share.

Given the above information, calculate the number of shares of Aardvark that Cosmo can purchase on margin.

A) 1,250
B) 625
C) 2,000
D) 2,500

A

Explanation

D) The answer is 2,500. Cosmo can purchase 2,500 shares calculated as follows: $50,000 ÷ 50% initial margin = $100,000 of buying power. $100,000 ÷ $40/share = 2,500 shares.

LO 1.2.1

How well did you know this?
1
Not at all
2
3
4
5
Perfectly
31
Q

An investor holding a Treasury bill as of the date of maturity includes

A) the amount of the discount as ordinary income.
B) the amount of the discount as a capital gain.
C) the face value as a capital gain.
D) the discounted sales price as ordinary income.

A

Explanation

A) The answer is the amount of the discount as ordinary income.

An investor holding a Treasury bill as of the date of maturity includes the amount of the discount as ordinary income. An investor who sells the bill before maturity includes as ordinary income only a portion of the acquisition discount based on the total time he held the bill. The remaining portion is capital gain income.

LO 1.3.1

How well did you know this?
1
Not at all
2
3
4
5
Perfectly
32
Q

Identify which of the following statements pertaining to the various types of money market investments is CORRECT.

I. Commercial paper offers higher yields than T-bills.

II. Eurodollars are U.S. dollar-denominated deposits at banks outside of the United States.

III. Banker’s acceptances are short-term drafts drawn by a private company on a major bank to finance imports and exports.

IV. T-bills are subject to default risk and a lack of marketability.

A) IV only
B) II, III, and IV
C) II and IV
D) I, II, and III

A

Explanation
D) The answer is I, II, and III. T-bills are not subject to default risk and exhibit a high degree of marketability. As a result, the 90-day T-bill is often used as a proxy for the risk-free investment.

LO 1.3.1

How well did you know this?
1
Not at all
2
3
4
5
Perfectly
33
Q

Identify which of the following statements regarding money market deposit accounts (MMDAs) are NOT correct.

I. They are FDIC insured.
II. They offer unlimited check writing privileges.
III. They are primarily offered by open-end investment companies.
IV. They require a minimum balance.

A) I and IV
B) II and III
C) III and IV
D) I and II

A

Explanation

B) The answer is II and III.

MMDAs provide limited check writing privileges and are offered by banks and savings and loans. MMDAs require a minimum balance. Unlike money market mutual funds, MMDAs are FDIC insured.

LO 1.3.1

How well did you know this?
1
Not at all
2
3
4
5
Perfectly
34
Q

Which of the following statements best describes banker’s acceptances?

A) U.S. dollar-denominated deposits at banks outside the United States

B) Negotiable, short-term, unsecured promissory notes issued by large corporations

C) Promissory notes traded at a premium from their face value in the secondary market

D) Short-term drafts drawn by a private company on a major bank used to finance imports and exports

A

Explanation

D) The answer is short-term drafts drawn by a private company on a major bank used to finance imports and exports.

Banker’s acceptances are typically traded at a discount from their face value in the secondary market. Eurodollars are U.S. dollar-denominated deposits at banks outside the United States.

LO 1.3.1

How well did you know this?
1
Not at all
2
3
4
5
Perfectly
35
Q

Identify the incorrect statement regarding passbook savings accounts.

A) Depositors are permitted to withdraw their savings at any time without penalty

B) They offer a relatively low interest rate

C) They require a minimum balance of $500

D) Accounts are established with a commercial bank or savings and loan

A

Explanation

C) The answer is they require a minimum balance of $500. Passbook savings accounts do not require a minimum balance.

LO 1.3.1

How well did you know this?
1
Not at all
2
3
4
5
Perfectly
36
Q

Select the arrangement that is commonly used by dealers in government securities to satisfy short-term liquidity needs.

A) Banker’s acceptance
B) Negotiable CDs
C) Repurchase agreement
D) Commercial paper

A

Explanation

C) The answer is repurchase agreement. Dealers in government securities use repurchase agreements, or repos, to satisfy short-term liquidity needs.

LO 1.3.1

How well did you know this?
1
Not at all
2
3
4
5
Perfectly
37
Q

A money market mutual fund manager recently purchased negotiable, short-term, unsecured promissory notes issued by a number of large corporations for the portfolio. Select the type of investment the money manager purchased.

A) Banker’s acceptances
B) Repurchase agreements
C) Reverse repurchase agreements
D) Commercial paper

A

Explanation

D) The answer is commercial paper.

Commercial paper is usually issued in denominations of $100,000 or more and is a substitute for short-term bank financing. Commercial paper is normally sold at a discount and is rated for quality by a rating service.

LO 1.3.1

How well did you know this?
1
Not at all
2
3
4
5
Perfectly
38
Q

Which of the following statements regarding Eurodollar CDs is CORRECT?

I. Eurodollar CDs are obligations of non-U.S. banks.

II. Eurodollar CDs are more liquid than domestic CDs.

III. Eurodollar CDs offer a slightly higher yield than domestic CDs.

IV. Eurodollar CDs are only used to settle transactions in the U.S.

A) I and III
B) I, II, and III
C) I only
D) II and IV

A

Explanation

A) The answer is I and III.

Statements II and IV are incorrect. Eurodollar CDs are less liquid than domestic CDs and are used to settle international transactions.

LO 1.3.1

How well did you know this?
1
Not at all
2
3
4
5
Perfectly
39
Q

Choose the characteristic that does not apply to Eurodollars.

A) Maturities of less than six months
B) Dollar-denominated deposits
C) Issued by the U.S. Treasury
D) Interest earned is taxed as ordinary income in the year received

A

Explanation

C) The answer is issued by the U.S. Treasury. Eurodollars are issued by banks outside of the United States, not the U.S. Treasury.

LO 1.3.1

How well did you know this?
1
Not at all
2
3
4
5
Perfectly
40
Q

All of the following statements correctly explain money market securities except

A) money markets securities include short-term, highly liquid, relatively low-risk debt instruments sold by governments, financial institutions, and corporations to investors with temporary funds to invest.

B) Treasury bills are an example of a money market security and are used as a proxy for the risk-free rate of return.

C) money market mutual funds are used by investors as part of their emergency fund because the funds are extremely liquid.

D) all money market mutual funds are exempt from taxation on both the state and federal levels.

A

Explanation

D) The answer is all money market mutual funds are exempt from taxation on both the state and federal levels. Open-end investment companies offer both taxable and tax-exempt money market mutual funds. Only state specific money market mutual funds are exempt from taxation for the taxpayers in those specific states.

LO 1.3.1

How well did you know this?
1
Not at all
2
3
4
5
Perfectly
41
Q

Which asset is used to satisfy the short-term liquidity needs of dealers in government securities, when such dealers sell some of those securities to another dealer with an agreement to buy them back at a later date at an agreed-upon price?

A) Eurodollars

B) Banker’s acceptances

C) Repurchase agreements

D) Commercial paper

A

Explanation

C) The answer is repurchase agreements. Repurchase agreements are used by dealers in government securities. Because of their low risk and short maturities, they are frequently found in money market mutual funds.

LO 1.3.1

How well did you know this?
1
Not at all
2
3
4
5
Perfectly
42
Q

Which of the following would be held in a money market portfolio?

I. Treasury bill
II. Negotiable CDs
III. Commericial paper

A) II and III
B) I and II
C) I, II, and III
D) I only

A

Explanation
C) The answer is I, II, and III. All of these financial instruments would be held in a money market portfolio.

LO 1.3.1

How well did you know this?
1
Not at all
2
3
4
5
Perfectly
43
Q

Identify the features of U.S. Treasury bills (T-bills).

I. U.S. Treasury bills are default risk-free.

II. The 90-day T-bill rate is frequently used as the risk-free rate of return in the capital asset pricing model.

III. U.S. Treasury bills offer a low rate of return when compared to riskier alternatives.

IV. T-bills offer a high degree of marketability.

A) I only
B) II and IV
C) I, II, III, and IV
D) III and IV

A

Explanation
The answer is I, II, III, and IV.

U.S. Treasury bills are short-term government securities. Because of their lack of default risk and high degree of marketability, T-bills are often used as the proxy for a risk-free investment, an asset having the lowest level of risk among all available assets, in modern portfolio management theory.

LO 1.3.1

How well did you know this?
1
Not at all
2
3
4
5
Perfectly
44
Q

A wash sale is deemed to have occurred within which of the following time frames?

A) 30 days
B) 60 days
C) 61 days
D) 31 days

A

Explanation
The answer is 61 days. 30 days before + date of sale + 30 days after = 61 total days.

LO 1.4.1

How well did you know this?
1
Not at all
2
3
4
5
Perfectly
45
Q

Which of the following statements regarding wash sales is CORRECT?

I. A wash sale occurs if the taxpayer sells or exchanges stock or securities for a loss and, within 30 days before or after the date of the sale or exchange, acquires
similar securities.

II. The wash sale rules are easily avoided in the case of fixed-income securities by substituting a bond with the same or similar characteristics as long as it is issued by a different company.

A) Both I and II
B) II only
C) Neither I nor II
D) I only

A

Explanation
The answer is both I and II. Both of these statements describe characteristics of wash sales.

LO 1.4.1

How well did you know this?
1
Not at all
2
3
4
5
Perfectly
46
Q

Distributions of dividend and capital gains in cash to mutual fund investors

I. are fully taxable to the investor.

II. are added to the tax basis of the shares once taxes on the distributions are paid.

III. decrease the taxable gain or increase the loss on sale of the shares after taxes are paid.

IV. decrease the cost basis of the shares whether or not taxes are paid.

A) I and II
B) I only
C) IV only
D) I, II, and III

A

Explanation
The answer is I only.

If the dividends and capital gains are reinvested, the individual receives an increased tax basis. If the distributions are made in cash, there is no increase in the tax basis of the underlying securities.

LO 1.4.1

How well did you know this?
1
Not at all
2
3
4
5
Perfectly
47
Q

Which of the following statements regarding dividend and interest income is CORRECT?

I. Generally, income from U.S. government obligations is taxable on state income tax returns.

II. All dividend income is taxable to individuals on both federal and state income tax returns.

A) II only
B) I only
C) Both I and II
D) Neither I nor II

A

Explanation
The answer is II only. Statement I is incorrect. Generally, income from U.S. government obligations is not taxable on state income tax returns.

LO 1.4.1

How well did you know this?
1
Not at all
2
3
4
5
Perfectly
48
Q

Which of the following statements regarding net investment income is CORRECT?

I. A Medicare contribution tax is imposed on taxpayers with net investment income.

II. Net investment income is investment income reduced by certain deductible investment expenses.

A) Both I and II
B) Neither I nor II
C) I only
D) II only

A

Explanation
The answer is both I and II. Both of these statements regarding net investment income are correct.

LO 1.4.1

How well did you know this?
1
Not at all
2
3
4
5
Perfectly
49
Q

A client has $12,000 of capital gains and $15,000 of capital losses. How much unused loss is carried forward to the following tax year?

A) $3,000
B) $12,000
C) $15,000
D) $0

A

Explanation

D) The answer is $0. After netting capital gain and losses, the client has a net capital loss of $3,000. Because $3,000 of net losses can be deducted during any one tax year, there is no carryforward.

LO 1.4.1

How well did you know this?
1
Not at all
2
3
4
5
Perfectly
50
Q

Galen has come to his financial planner with questions about dividends he received on some of his stock this year. He has received $1,000 in qualified dividends paid in cash. He also has received stock dividends of $4,000, but without a cash dividend option. How much will Galen have to report as dividend income for the current year?

A) $1,000
B) $5,000
C) $0
D) $4,000

A

Explanation
The answer is $1,000. Only the dividends paid in cash to Galen that are reported as income. The stock dividends did not have a cash-dividend option and are not taxable.

LO 1.4.1

How well did you know this?
1
Not at all
2
3
4
5
Perfectly
51
Q

One advantage of convertible bonds is that they

A) have a higher coupon rate than the underlying stock’s dividend.
B) have higher coupon rates than straight coupon bonds.
C) are usually offered only by firms with high bond credit ratings.
D) offer the ability to buy the underlying stock at a discount.

A

A) Explanation
The coupon rate on the convertible bond is usually greater than the stock’s dividend yield because the stock usually pays little, if any dividends. This enables the investor to get a higher periodic cash flow while waiting for the stock to appreciate.

LO 2.2.1

How well did you know this?
1
Not at all
2
3
4
5
Perfectly
52
Q

Your client owns a ZIM Corporation convertible bond that has a coupon rate of 9% paid semiannually and matures in 10 years. Comparable debt yields 8% currently. The ZIM bond conversion price is $40 per share. The current market price of the underlying stock is $46. What is the conversion value of this convertible bond?

A) $1,223
B) $1,150
C) $1,000
D) $1,067

A

B) The answer is $1,150.

How well did you know this?
1
Not at all
2
3
4
5
Perfectly
53
Q

To immunize a bond portfolio over a specific investment horizon, an investor would do which of the following?

A) Match the maturity of each bond to the investment horizon.
B) Match the duration of each bond to the investment horizon.
C) Match the average weighted maturity of the portfolio to the investment horizon.
D) Match the average weighted duration of the bond portfolio to the investment horizon.

A

D) When a portfolio is immunized, its liabilities and expected future cash outflows are funded by making sure that the cash flow from investments (income and principal) will be there at the time that the cash outflow is needed. That is done by matching the duration, not the maturity, of the bond portfolio to the number of years until the cash outflow will occur. The duration of the portfolio as a whole should be matched, not the duration of each bond in the portfolio.

LO 2.3.1

How well did you know this?
1
Not at all
2
3
4
5
Perfectly
54
Q

Klaus Copenhagen’s objective is to receive income, and he is considering a preferred stock with a $1.50 dividend that is currently trading at $25. What would be the approximate price movement of this preferred stock if interest rates were to rise 1%?

A) –14%
B) –4%
C) –10%
D) –7%

A

A)

First, determine what the current interest rate is, $1.50 ÷ $25 = 0.06. Now, determine the percentage price movement if interest rates climb 1%—$1.50 ÷ 0.07 = $21.43. This is a decline of $3.57, or 14.28%.

LO 2.5.1

How well did you know this?
1
Not at all
2
3
4
5
Perfectly
55
Q

A convertible bond has a conversion price of $40 per share. At which one of the following combinations of values would it make sense for an investor to convert a convertible bond into shares of stock?

Stock Price. Bond Intrinsic Value. Conversion Value

A $32 $900 $800

B. $42 $1,100 $1,050

C. $38 $1,050 $950

D. $45 $1,100 $1,125

A) Option A
B) Option D
C) Option B
D) Option C

A

Explanation

B) Option D

At all other values, the bond’s intrinsic value is greater than the conversion value; a conversion under those circumstances means that the investor is automatically losing money on the conversion, excluding the fact that the convertible bond sells at a premium to its intrinsic value as a bond.

At a minimum, the conversion value should be greater than the bond intrinsic value for conversion to be considered.

LO 2.2.1

How well did you know this?
1
Not at all
2
3
4
5
Perfectly
56
Q

Bonds issued by state and local governments that are backed by the full faith and credit of the issuing government and repaid by the issuing municipality’s taxing power are categorized as

A) revenue bonds.
B) general obligation bonds.
C) tax anticipation notes.
D) revenue anticipation bonds.

A

B)
General obligation bonds (GOs) are the most secure of all municipal debt because they are backed by the full faith and credit of the issuer. Municipalities may increase taxes to make principal and interest payments on any debt issue; therefore, a voter referendum is usually required to approve their issuance.

LO 2.1.1

How well did you know this?
1
Not at all
2
3
4
5
Perfectly
57
Q

Identify the CORRECT statements regarding portfolio immunization.

I. A bond portfolio is said to be immunized from default risk and business risk when the duration of the portfolio is equal to the time horizon of the investor.

II. If the time horizon of an investor is 10 years, a simple way to protect against interest rate risk and reinvestment rate risk would be to fund the portfolio only with zero-coupon bonds with a 12-year term to maturity.

A) Both I and II
B) Neither I nor II
C) II only
D) I only

A

B)
Neither Statement I nor II is correct.

A bond portfolio is said to be immunized from interest rate risk and reinvestment rate risk when the duration of the portfolio is equal to the time horizon of the investor.

If the time horizon of an investor is 10 years, a simple way to protect against interest rate risk and reinvestment rate risk would be to fund the portfolio only with zero-coupon bonds with a 10-year term to maturity.

LO 2.3.1

How well did you know this?
1
Not at all
2
3
4
5
Perfectly
58
Q

The reason for using a ladder bond strategy is to

A) turn a paper loss into an actual loss.
B) lower interest rate risk.
C) magnify gains.
D) spread cash flows evenly over a given time horizon to eliminate default risk.

A

Explanation
B) For example, with a ladder bond strategy, instead of investing all money in a seven-year bond, an investor may divide the dollars among bonds with one, three, five, seven, and nine-year maturities. With this approach, instead of making a single bet on interest rates, the investor has both longer and shorter maturities, so that regardless of which way interest rates move the investor will not experience either great losses or great gains.

LO 2.3.1

How well did you know this?
1
Not at all
2
3
4
5
Perfectly
59
Q

A convertible bond has a 6.5% coupon rate, interest is paid semiannually, and the bond matures in five years. Comparable debt currently yields 7.5%. The bond is convertible into common stock at $25 per share. The current price of the stock is $28, and the current price of the convertible bond is $1,050. What is the investment value of the bond?

A) $958.94
B) $1,000.00
C) $968.95
D) $1,008.90

A

Explanation
A bond’s investment value is the same as its intrinsic value as a straight bond. Using an HP 10bII+ calculator, the bond price is determined by inputting the following keystrokes:

(END mode)

10, N

  1. 5 ÷, 2 I/YR
  2. 5, PMT

1000, FV

Solve for PV = –958.94, or $958.94.

LO 2.2.1

How well did you know this?
1
Not at all
2
3
4
5
Perfectly
60
Q

An investor pays a premium for a convertible bond because:

A) the investor is buying a straight bond and buying a call option.

B) the investor is buying a straight bond and selling a call option.

C) the investor is buying a straight bond and buying a put option.

D) the investor is buying a straight bond and selling a put option.

A

A) The investor is long the straight bond (bought) and long the call option (bought).

LO 2.2.1

How well did you know this?
1
Not at all
2
3
4
5
Perfectly
61
Q

Assume YC1 is an inverted yield curve and one year later all interest rates have decreased to where the new yield curve, YC2, is a normal yield curve. Which of the following provides information that can be interpreted from these yield curves?

I. Duration has decreased.
II. Duration has increased.
III. Treasury bill rates were lower than Treasury bond rates at the time of YC2.
IV. Inflation may have been high at the time of YC1.

A) II, III, and IV
B) I and IV
C) II and IV
D) I, III, and IV

A

Explanation

A) Market interest rates have decreased from YC1 to YC2. When market rates decrease, duration increases, because market interest rates and duration are inversely related. Treasury bills are short-term debt and Treasury bonds are long-term debt. When an inverted yield curve changes to a normal yield curve, short-term rates will decline more than long-term rates. An inverted yield curve normally occurs when inflation is high and the Fed fights inflation by raising short-term interest rates above long-term interest rates.

LO 2.4.1

How well did you know this?
1
Not at all
2
3
4
5
Perfectly
62
Q

A client has a cash need at the end of seven years. Which of the following investments might initially immunize the portfolio?

I. A nine-year maturity coupon bond
II. A seven-year maturity coupon Treasury note
III. A series of Treasury bills

A) I, II, and III
B) I only
C) II only
D) II and III

A

Your primary objective is to match duration to the cash need of seven years.

Treasury bills do not come close because the maturity of the bills is one year or less, meaning that the duration must be less than one year. A seven-year note must have a duration less than seven years because the note has a coupon; therefore, it is likely that the seven-year note duration will not be sufficient to match the duration of the need either. The duration of a 9 year bond with a coupon must be less than nine years. You can estimate that the duration of an intermediate-term bond might be about 80% of the bond’s maturity, or, in this case, about seven years. That is a close enough estimate to arrive at the answer to this question.

LO 2.3.1

How well did you know this?
1
Not at all
2
3
4
5
Perfectly
63
Q

Which of the following is CORRECT with respect to convertible bonds and convertible preferred stock if the value of the common stock rises?

A) The value of convertible bonds and convertible preferred stock declines.
B) The value of convertible bonds and convertible preferred stock rises.
C) The value of convertible bonds rises, but convertible preferred stock falls.
D) The value of convertible bonds falls, but convertible preferred stock rises.

A

Explanation
B) The value of any convertible will rise with a rise in the underlying common stock into which it can be converted. The convertibles will not necessarily have the same gain; it will depend on their premium and conversion terms.

LO 2.5.1

How well did you know this?
1
Not at all
2
3
4
5
Perfectly
64
Q

The reason for using a barbell bond strategy is to

A) maximize the potential capital gain in a bond portfolio.
B) increase interest rate risk.
C) decrease default risk.
D) offset price and reinvestment rate risk.

A

Explanation
D) The purpose for using a barbell strategy in a bond portfolio is to offset the opposite effects of interest rates on bond prices. If rates rise, short-term bonds can be reinvested at higher rates. If rates drop, long-term bonds are used to lock in rates.

LO 2.3.1

How well did you know this?
1
Not at all
2
3
4
5
Perfectly
65
Q

You want to generate additional income from your portfolio, and are considering purchasing either bonds or preferred stocks. Which of the following statements is NOT correct concerning the characteristics of bonds and preferred stocks?

A) Bonds are subject to more interest rate risk than preferred stocks.

B) In the event of a company’s bankruptcy, bondholders would be paid first ahead of preferred stock shareholders.

C) Bonds pay interest while preferred stocks pay dividends.

D) Corporations pay taxes on preferred stock dividends prior to distribution to preferred shareholders, whereas interest on bonds is a deductible expense.

A

Explanation
A) Because preferred stock does not have a maturity date, it is subject to more interest rate risk than bonds.

LO 2.5.1

How well did you know this?
1
Not at all
2
3
4
5
Perfectly
66
Q

Jennifer owns a state public purpose bond. She sells the bond and realizes a capital gain of $4,000. Prior to selling the bond, the total interest she had earned for the year was $99. Considering the sale and the interest amount, calculate the amount she must include in gross income.

A) $99
B) $3,001
C) $4,099
D) $4,000

A

D) Explanation
The interest on public purpose bonds is received tax-free by the holder. Only the capital gain realized on the sale is subject to income taxation.

LO 2.1.1

How well did you know this?
1
Not at all
2
3
4
5
Perfectly
67
Q

Emily will be retiring in a few months and would like some additional income to supplement her retirement benefits. A significant portion of her portfolio is in growth stocks, and she has some concern about how to structure the fixed-income portion of her portfolio.

Which of the following types of investments would be most appropriate for Emily?

A) A 20-year, 8%, A rated income bond
B) A “Z” tranche CMO
C) A medium duration, AA rated, high-coupon corporate bond
D) A high mortgage interest rate GNMA certificate selling at a premium, with a high prepayment experience

A

Explanation
C) Emily already owns higher-risk stocks and needs some additional income. As a retiree, she should not be invested in the higher-risk bonds in the other possible options.

A high mortgage interest rate GNMA certificate selling at a premium, with a high prepayment experience is not a good choice because monthly cash flows are highly unpredictable, and principal is being returned at a high rate.

A “Z” tranche CMO is not a good choice because it is similar to a zero-coupon bond.

A 20-year, 8%, A rated income bond is not appropriate because it is an income bond that will only pay interest when and if earned, so it is not a dependable source of income.

LO 2.3.2

How well did you know this?
1
Not at all
2
3
4
5
Perfectly
68
Q

The coupon rate or nominal yield of a bond is the stated annual interest rate that will be paid each period for the term of the bond. Select the statement that best describes how the coupon rate is stated.

A) As a percentage of the par value of the bond
B) As a percentage of the discount rate of the bond
C) As a percentage of the annuitized value of the bond
D) As a percentage of the intrinsic value of the bond

A

Explanation
A) The coupon rate is stated as a percentage of the par value.

LO 2.1.1

How well did you know this?
1
Not at all
2
3
4
5
Perfectly
69
Q

Identify which of these statements regarding revenue bonds is NOT correct.

I. They are secured by a specific pledge or property.
II. They are a type of full faith and credit bond.
III. Their interest is tax-exempt at the federal level.
IV. They are analyzed by the project’s ability to generate earnings.

A) I and III
B) I and II
C) II and IV
D) III and IV

A

Explanation
The answer is I and II.

Revenue bonds are not secured by property and are not a type of general obligation bond. They are only secured by user fees.

LO 2.1.1

How well did you know this?
1
Not at all
2
3
4
5
Perfectly
70
Q

Darla, a U.S. citizen and resident of Georgia, owns a 5% coupon corporate bond, a 4% coupon State of Georgia municipal bond, and a 3% coupon U.S. Treasury note. Darla’s marginal state income tax rate is 6% and federal tax rate is 24%. If Darla invested equal amounts in each of the three bonds, what is her after-tax rate of return on the portfolio?

A) 4.00%
B) 4.91%
C) 2.86%
D) 3.26%

A

Explanation
D) The answer is 3.26%.

Because the corporate bond is taxable by the state and the federal government, its after-tax return is 3.5% [5% × (1 – 0.30)].

The State of Georgia municipal is not taxable by either government entity.

The Treasury note is taxable by the federal government; therefore, its after-tax return is 2.28% [3% × (1 – 0.24)].

Averaging the three rates equals 3.26% [(3.5% + 4% + 2.28%) ÷ 3].

LO 2.1.1

How well did you know this?
1
Not at all
2
3
4
5
Perfectly
71
Q

Identify which of these statements regarding Treasury Inflation-Protected Securities (TIPS) is CORRECT.

A) The TIPS coupon rate is adjusted every 12 months based on changes in the Consumer Price Index (CPI).

B) A semiannual inflation rate is combined with the stated coupon rate to determine the TIPS interest rate for the next six months.

C) TIPS are issued at 50% of the par value with the par value being adjusted every six months for inflation.

D) The principal value is adjusted for inflation every six months based on the Consumer Price Index (CPI), and one-half of the stated coupon rate is paid semiannually on the inflation-adjusted principal value.

A

Explanation
D) The answer is the principal value is adjusted for inflation every six months based on the CPI, and one-half of the stated coupon rate is paid semiannually on the inflation-adjusted principal value. TIPS coupon rate stays the same for the life of the security, but the interest payment changes based on the inflation-adjusted principal or par value.

LO 2.1.1

How well did you know this?
1
Not at all
2
3
4
5
Perfectly
72
Q

Question #4 of 30
Question ID: 1239957
The state of Louisiana has a new municipal five-year bond issue offering a 4% coupon rate. Which of these is the greatest tax benefit investors would receive from purchasing this bond?

A) Georgia resident, 6% state income tax bracket, 24% federal income tax bracket
B) Florida resident, 0% state income tax bracket, 37% federal income tax bracket
C) Louisiana resident, 6% state income tax bracket, 35% federal income tax bracket
D) Mississippi resident, 6% state income tax bracket, 12% federal income tax bracket

A

Explanation

C) The answer is Louisiana resident, 6% state income tax bracket, 35% federal income tax bracket. The municipal bond interest is generally exempt from federal income tax, but is subject to both state and local taxes unless the bond is purchased by a resident of the issuing state. While the Florida resident is taxed at the highest federal tax rate and pays no state income tax, the overall tax savings is greatest for the Louisiana resident whose combined state and federal tax bracket is 41%.

LO 2.1.1

How well did you know this?
1
Not at all
2
3
4
5
Perfectly
73
Q

Sam holds a considerable amount of both Series EE and Series HH savings bonds. He is nearing retirement and likes the fact that his Series HH bonds pay interest semiannually and would like to exchange most of his Series EE bonds for Series HH bonds to increase his cash flow. Choose which of these statements regarding such an exchange is CORRECT.

A) Series EE bonds may no longer be exchanged for Series HH bonds.
B) Sam may exchange the bonds but will be subject to a three-month interest penalty.
C) Only Sam’s Series EE bonds issued prior to 2004 may be exchanged.
D) Sam may exchange the bonds but must recognize the Series EE accrued interest at the time of exchange.

A

Explanation

A) The answer is Series EE bonds may no longer be exchanged for Series HH bonds. Until September 2004 (when Series HH bonds were no longer issued by the Treasury), the exchange of EE bonds for HH bonds was a popular way of continuing the income tax deferral on the accrued interest portion of the EE bonds. Such changes are no longer possible.

LO 2.1.1

How well did you know this?
1
Not at all
2
3
4
5
Perfectly
74
Q

Select which of these statements regarding Treasury notes and Treasury bonds is CORRECT.

I. Both Treasury notes’ and bonds’ interest payments are income tax free at the state and federal level.

II. Treasury notes and bonds are considered default risk free.

III. If held for more than one year, interest paid on Treasury bonds is eligible for long-term capital gain treatment.

IV. Both Treasury notes and bonds are not traded in the secondary market.

A) II only
B) III and IV
C) I, III, and IV
D) I and II

A

Explanation
A) The answer is II only. Both Treasury notes and bonds are considered default risk free. Both obligations trade in the secondary market and pay interest, which is income tax free at both the state and local level, but taxed as ordinary income in the year earned at the federal level.

LO 2.1.1

How well did you know this?
1
Not at all
2
3
4
5
Perfectly
75
Q

Question #7 of 30
Question ID: 1239960
Identify the statement that best describes Treasury STRIPS (Separate Trading of Registered Interest and Principal of Securities).

A) A negotiable, short-term, unsecured promissory note issued by a regional government agency

B) A short-term draft drawn by a government agency on a major bank

C) A trust receipt issued by a U.S. government agency for shares of a domestic company purchased and held by a credit union

D) Zero-coupon bonds created by separating the semiannual coupon payments and the principal repayment portions of a U.S. Treasury note or bond

A

Explanation

D) The answer is zero-coupon bonds created by separating the semiannual coupon payments and the principal repayment portions of a U.S. Treasury note and bond. Although the securities underlying Treasury STRIPS are the U.S. government’s direct obligation, major banks and dealers perform the actual separation and trading.

LO 2.1.1

How well did you know this?
1
Not at all
2
3
4
5
Perfectly
76
Q

Select which of these is NOT a primary risk associated with a coupon-paying bond.

A) Purchasing power risk
B) Debenture risk
C) Interest rate risk
D) Default risk

A

Explanation
The answer is debenture risk. A coupon-paying bond is also subject to reinvestment rate risk.

LO 2.1.1

How well did you know this?
1
Not at all
2
3
4
5
Perfectly
77
Q

Tom owns a taxable investment that earns 8% interest annually. Tom pays taxes at a marginal rate of 24%. Calculate the after-tax rate of return that Tom will receive on this investment.

A) 6.30%
B) 6.08%
C) 2.24%
D) 6.25%

A

Explanation
The answer is 6.08%. The after-tax return is 8% × (1 − 0.24) = 6.08%.

LO 2.1.1

How well did you know this?
1
Not at all
2
3
4
5
Perfectly
78
Q

An investor would consider converting a convertible bond into common stock if the bond’s

A) yield to call is the same as a comparable municipal bond.
B) market price is less than the conversion value.
C) duration exceeds 10 years.
D) yield to maturity is less than its conversion premium.

A

Explanation

B) The answer is market price is less than the conversion value. An investor would consider converting a convertible bond into common stock if the bond’s conversion value exceeds its market price.

LO 2.2.1

How well did you know this?
1
Not at all
2
3
4
5
Perfectly
79
Q

Ellen Hyson purchased a BB rated convertible bond of TCD Corporation that has a 10% coupon and matures in nine years. Comparable debt (BB rated, nine years to maturity) yields 12%. The bonds are convertible at $32 per share of common stock, and the current market price of TCD common stock is $25.

What is the conversion value of this bond?

A) $916.25
B) $800.00
C) $893.50
D) $781.25

A

Explanation

D) The answer is $781.25. The conversion value = conversion ratio × market price of common stock. Therefore, the conversion value equals ($1,000 ÷ $32) × $25 = $781.25.

LO 2.2.1

How well did you know this?
1
Not at all
2
3
4
5
Perfectly
80
Q

Chuck Johnson owns a convertible bond that has a conversion price of $40 per share and a coupon of 5.5%. Interest is paid semiannually. The current market price of the stock is $41 per share. The investment value of the bond is $940, and the bond currently sells for a market price of $1,120. What is the downside risk of this bond?

A) $120
B) $180
C) $95
D) $85

A

Explanation

The answer is $180. The downside risk of a convertible bond is the dollar or percentage decline from the current market price of the convertible bond to the investment value of the bond: $1,120 - $940 = $180.

LO 2.2.1

How well did you know this?
1
Not at all
2
3
4
5
Perfectly
81
Q

George Jones owns a convertible bond that has a conversion price of $50 per share and an annual coupon rate of 6.0%. Interest is paid semiannually. The current market price of the stock is $51 per share. The investment value of the bond is $890, and the bond currently sells for a market price of $1,080.

What is the downside risk of this bond?

A) $210
B) $155
C) $130
D) $190

A

Explanation
The answer is $190. The downside risk of a convertible bond is the dollar or percentage decline from the current market price of the convertible bond to the investment value of the bond: $1,080 - $890 = $190.

LO 2.2.1

How well did you know this?
1
Not at all
2
3
4
5
Perfectly
82
Q

All of these are advantages of convertible bonds except

A) they offer a yield that is less than the yield on straight bonds with similar risk and maturities.
B) they lack the downside risk protection for investors.
C) they are normally not converted.
D) because they are not callable, convertible bonds must be held until maturity.

A

Explanation

A) The answer is they offer a yield that is less than the yield on straight bonds with similar risk and maturities.

Convertible bonds can be converted into a specified number of common shares at the option of the investor. Because of this option, they are sold at a premium over the price of comparable nonconvertible bonds. In other words, they offer a yield that is less than the yield on straight bonds with similar risk and maturities. When stocks sell for a price that is above the conversion price, convertible bonds sell for their stock values. However, convertible bonds have downside risk protection because they have the advantage of bonds. Most convertible bonds are converted. Also, most are callable, and companies often call their bonds to force conversion.

LO 2.2.1

How well did you know this?
1
Not at all
2
3
4
5
Perfectly
83
Q

An investor wants all of her bonds to mature in 10 years. She buys two bonds immediately, two bonds two years from now, and two more bonds four years from now. As a result, the bonds purchased immediately have a maturity of 10 years, the bonds purchased two years later have a maturity of eight years, and the bonds purchased four years later have a maturity of six years. Select the type of bond strategy she is using for her portfolio.

A) Bond barbell
B) Bond bullet
C) Bond swap
D) Bond ladder

A

Explanation

B) The answer is bond bullet. When using the bond bullet strategy, an investor purchases a series of bonds with similar maturities focused on one point in time. This strategy may be an effective method in matching duration to the cash needs of an investor.

LO 2.3.1

How well did you know this?
1
Not at all
2
3
4
5
Perfectly
84
Q

Question #16 of 30
Question ID: 1239996
Identify the statement that is NOT correct regarding bond ladders.

A) A laddered portfolio consists of a combination of long-term and short-term bonds.

B) Shorter maturity bonds in a laddered portfolio are more subject to price fluctuations than the longer maturity bonds.

C) A laddered portfolio will provide higher yields than a portfolio consisting entirely of short-term bonds.

D) When a bond matures, the investor can purchase another bond with a maturity that will maintain the original structure of the bond portfolio.

A

Explanation
B) The answer is shorter maturity bonds in a laddered portfolio are more subject to price fluctuations than the longer maturity bonds. The shorter maturity bonds in a laddered portfolio are less subject to price fluctuations than the longer maturity bonds because of their shorter duration.

LO 2.3.1

How well did you know this?
1
Not at all
2
3
4
5
Perfectly
85
Q

A transaction whereby an investor sells a bond for a loss, in order to reduce capital gains, while investing the proceeds of the sale in a bond of similar quality and maturity is considered

A) a tax swap.
B) a pure yield pickup swap.
C) a substitution swap.
D) an intermarket spread swap.

A

Explanation
The answer is a tax swap. A tax swap results in a tax savings generated by the realized loss.

LO 2.3.1

How well did you know this?
1
Not at all
2
3
4
5
Perfectly
86
Q

Which of these statements regarding the bond ladder strategy is CORRECT?

A) The bond ladder strategy is used to immunize a portfolio against interest rate risk.

B) The bond ladder strategy involves the purchase of a mixture of very long-term and very short-term bonds.

C) The bond ladder strategy is generally more aggressive than the barbell strategy.

D) A laddered portfolio will provide lower yields than a portfolio consisting entirely of short-term bonds.

A

Explanation

A) The answer is the bond ladder strategy is used to immunize a portfolio against interest rate risk. It is an investment strategy in which equal amounts of money are invested in a series of bonds with staggered maturities. The barbell strategy involves the purchase of a mixture of very long-term and very short-term bonds. The laddered portfolio will provide higher yields than a portfolio consisting entirely of short-term bonds. The barbell strategy is generally more aggressive than the ladder strategy because the barbell strategy only utilizes short-term and long-term bonds.

LO 2.3.1

How well did you know this?
1
Not at all
2
3
4
5
Perfectly
87
Q

If a bond is immunized against interest rate risk, a dollar decline in the bond’s price, resulting from rising interest rates, will be approximately offset by a dollar increase in the

A) income from coupons reinvested over the investment horizon.
B) price of comparable bonds in the market.
C) bond’s call price.
D) bond issuer’s common stock.

A

Explanation

A) The answer is income from coupons reinvested over the investment horizon. By investing in bonds that have a duration equal to the investor’s investment time horizon, any bond price/value changes caused by interest rate fluctuations will be approximately offset by changes in the interest earned on the reinvested coupons.

LO 2.3.1

How well did you know this?
1
Not at all
2
3
4
5
Perfectly
88
Q

Which of the following statements is CORRECT?

I. If an investor expects a decline in market interest rates, she should attempt to construct a portfolio of long maturity bonds with low coupon rates.

II. If the investor expects an increase in market interest rates, he should attempt to construct a portfolio of short maturity bonds with high coupon rates.

A) II only

B) Neither I nor II

C) Both I and II

D) I only

A

Explanation

The answer is both I and II.

The portfolio in Statement I will provide the investor with a portfolio that has the maximum interest rate sensitivity to take advantage of the capital gains experienced by bonds from the decrease in market interest rates.

The portfolio in Statement II will provide the investor with a portfolio that has the minimum interest rate sensitivity to minimize the capital losses experienced by bonds from the increase in market interest rates.

LO 2.3.2

How well did you know this?
1
Not at all
2
3
4
5
Perfectly
89
Q

Which one of these is CORRECT regarding preferred stock?

A) Preferred stock’s dividends are tax deductible for corporations.

B) Preferred stockholders have voting rights.

C) Preferred stock’s value is based on prevailing interest rates.

D) Failure to pay preferred stock dividends results in bankruptcy.

A

Explanation

C) The answer is preferred stock’s value is based on prevailing interest rates. The value for a preferred stock is its dividend divided by prevailing interest rates.

LO 2.5.1

How well did you know this?
1
Not at all
2
3
4
5
Perfectly
90
Q

An individual with a short-term investment time horizon would choose what type of bonds when interest rates are expected to rise?

A) High-yield bonds
B) Low-coupon, long-term bonds
C) Short-term bonds
D) Long-term bonds

A

Explanation

C) The answer is short-term bonds.

Long-term bonds are affected by interest rate changes more than short-term bonds. If interest rates are expected to rise, an investor should invest in short-term bonds until rates peak. Risk of default of high-yield bonds increases when rates rise.

LO 2.3.2

How well did you know this?
1
Not at all
2
3
4
5
Perfectly
91
Q

If an investor is looking to purchase bonds that are free from default risk, which of these should be purchased?

A) Municipal bonds
B) Revenue bonds
C) Corporate bonds
D) Government bonds

A

Explanation
The answer is government bonds. Unlike corporate, revenue, and municipal bonds, government bonds are free from default risk.

LO 2.3.2

How well did you know this?
1
Not at all
2
3
4
5
Perfectly
92
Q

Assume that the yield curve currently is shaped as shown in YC2 (INVERTED) and that you anticipate it will be shaped as shown in YC1 (NORMAL) one year from now.

Assuming that you want to maximize the opportunity for capital appreciation, which one of these investment strategies would you recommend for clients, based on the current and anticipated shapes of the yield curves?

A) Sell short-term bonds and buy long-term bonds
B) Sell long-term bonds and buy short-term bonds
C) Sell long-term bonds and buy intermediate-term bonds
D) Sell intermediate-term bonds and buy short-term bonds

A

Explanation

A) The answer is sell short-term bonds and buy long-term bonds. If interest rates are expected to fall soon, an investor should sell short-term bonds and buy long-term bonds to lock in the current higher interest rates offered by the long-term bonds.

LO 2.4.1

How well did you know this?
1
Not at all
2
3
4
5
Perfectly
93
Q

Choose the CORRECT statement regarding yield curve theories.

A) The liquidity preference theory holds that investors will pay a premium for shorter maturity bonds to avoid the higher interest rate risk associated with long-term bonds.

B) Unbiased expectations theory states that long-term rates consist of many short-term rates and that long-term rates will be the average of short-term rates.

C) The market segmentation theory relies on the laws of supply and demand for various maturities of borrowing and lending.

D) All of these statements are correct.

A

Explanation
The answer is all of these statements are correct. The term structure of interest rates theory has several iterations. Although unbiased expectations theory and market segmentation theory may be used to explain any shaped yield curve, the liquidity preference theory only explains a normal, upward-sloping yield curve.

LO 2.3.2

How well did you know this?
1
Not at all
2
3
4
5
Perfectly
94
Q

All of these statements correctly describe yield curves except

A) a flat yield curve occurs when the economy is peaking and, therefore, no change in future interest rates is expected.

B) a normal yield curve occurs during periods of economic expansion and generally predicts that market interest rates will rise in the future.

C) an inverted yield curve occurs when the Federal Reserve has tightened credit in an overheating economy.

D) a positive yield curve can signal an upcoming economic recession.

A

Explanation
The answer is a positive yield curve can signal an upcoming economic recession. A positive (or normal) yield curve occurs during periods of economic expansion and generally predicts that market interest rates will rise.

LO 2.4.1

How well did you know this?
1
Not at all
2
3
4
5
Perfectly
95
Q

According to the unbiased expectations theory of interest rates,

A) an upward sloping yield curve indicates that investors require a yield premium to invest in long-term securities.

B) investors require a higher yield on long-term bonds because they are riskier.

C) yields are a function of the supply and demand of funds in each maturity segment of the market.

D) the current long-term rate is the average of today’s short-term rate and expected future short-term rates.

A

Explanation

The answer is the current long-term rate is the average of today’s short-term rate and expected future short-term rates. The other statements relate to the liquidity premium theory and the market segmentation theory. Under the unbiased expectations theory, an upward sloping yield curve indicates increasing inflation expectations.

LO 2.4.1

How well did you know this?
1
Not at all
2
3
4
5
Perfectly
96
Q

The yield curve theory that states current long-term interest rates contain an implicit prediction of future short-term interest rates is known as

A) liquidity preference theory.
B) unbiased expectations theory.
C) market segmentation theory.
D) preferred habitat theory.

A

Explanation
B) The answer is unbiased expectations theory. The unbiased expectations theory states that long-term rates consist of many short-term rates and that long-term rates will be the average (or geometric mean) of short-term rates.

LO 2.5.1

How well did you know this?
1
Not at all
2
3
4
5
Perfectly
97
Q

Select the CORRECT statements concerning preferred stock.

I. Preferred stock is known as a hybrid security because it resembles both a fixed-income security and an equity instrument.

II. Preferred stock has a maturity date and pays interest similar to a bond.

III. A large portion of the preferred stock issued can be converted into corporate bonds.

IV. A company’s preferred stock may be cumulative where all unpaid dividends must be paid to the preferred shareholders before dividends can be paid to common shareholders.

A) I and II
B) I and IV
C) II, III, and IV
D) III and IV

A

Explanation

B) The answer is I and IV.

Preferred stock is considered a hybrid security. This type of stock has an infinite life span and pays dividends like an equity instrument. Similar to a debt security, the dividend amount is fixed and known in advance. Preferred stock may be cumulative.

LO 2.5.1

How well did you know this?
1
Not at all
2
3
4
5
Perfectly
98
Q

Which of these describe similarities between preferred stock and long-term bonds?

I. Both dividends and interest are tax-deductible expenses for the issuing corporations.

II. Both generally pay a fixed periodic payment.

III. Both preferred dividends and interest must be paid before common stock cash dividends are paid.

A) III only
B) I and III
C) II and III
D) I and II

A

Explanation

C) The answer is II and III.

Both preferred stock and long-term bonds generally pay a fixed periodic payment, and both preferred dividends and interest must be paid before common stock cash dividends are paid.

LO 2.5.1

How well did you know this?
1
Not at all
2
3
4
5
Perfectly
99
Q

Max bought 100 shares of PET Corporation stock 10 years ago. He paid $10 per share for the stock. The stock currently has a fair market value of $50 per share. Choose the CORRECT statement regarding Max’s stock.

A) If Max does not sell the stock this year, he will not have to pay taxes on the difference between the amount he paid for the stock and the current market price this year.

B) If Max sells the stock now, he must pay taxes on the full $5,000 he receives from the sale.

C) If Max sells the stock now, he must pay taxes on only half the amount he receives from the sale.

D) If Max does not sell the stock now, he must pay taxes on the difference between the amount he paid for the stock and the current market price, which will increase his basis in the stock.

A

Explanation

A) The answer is if Max does not sell the stock this year, he will not have to pay taxes on the difference between the amount he paid for the stock and the current market price this year.

If Max sells the stock today, he realizes the gain in the value of the stock and must pay taxes on the gain. The gain would equal the price at which Max sells the stock minus the original purchase price, or $40 per share. If the gain is not realized, Max has no tax liability for the increase in the value of the stock. Capital gains are only taxable when they are realized at the time of the sale.

LO 3.1.1

How well did you know this?
1
Not at all
2
3
4
5
Perfectly
100
Q

Identify the CORRECT statements regarding warrants.

I. Warrants give the owner the right to purchase a specified number of shares for a specified period at a specified price.

II. Warrants are typically written with a maturity date of nine months.

III. Warrants must include standardized terms required by the Options Clearing Corporation.

IV. Warrants are issued by a corporation rather than written by an individual.

A) I, II, III, and IV

B) I and II

C) III and IV

D) I and IV

A

Explanation

The answer is I and IV.

Warrants typically have a maturity date of several years, not months, and are customized to fit the needs of the issuing corporation.

LO 3.1.1

How well did you know this?
1
Not at all
2
3
4
5
Perfectly
101
Q

Select the investment that gives the shareholder a short-term opportunity to buy new shares of the new stock issue, thereby maintaining the shareholder’s respective overall percentage ownership in the corporation.

A) Warrants
B) Preferred stock
C) Call options
D) Rights

A

Explanation
The answer is rights.

Rights are a purchase option for stock that allows a shareholder the opportunity to buy shares of the new stock issue, thereby maintaining the overall percentage ownership in the corporation.

LO 3.1.1

How well did you know this?
1
Not at all
2
3
4
5
Perfectly
102
Q

Which of these is a correct justification for use of an investment in a client’s portfolio?

I. Blue chip common stocks because they provide a hedge against inflation

II. FNMA (Federal National Mortgage Association) securities because they are backed by the full faith and credit of the U.S. government

III. Aggressive growth stocks because they perform better during economic contractions

A) I and II
B) I only
C) II and III
D) I and III

A

Explanation
B) The answer is I only.

Stocks generally are considered an inflation hedge; in periods of hyper-inflation, this may not be true, but the question does not ask about periods of hyper-inflation. FNMA securities are not backed by the full faith of the government (the government did step in as a result of the credit crisis of 2008, but there has not been a commitment to permanently back FNMAs in the same way that GNMAs (Government National Mortgage Association) have historically been backed).

LO 3.1.1

How well did you know this?
1
Not at all
2
3
4
5
Perfectly
103
Q

Select the CORRECT statements regarding stock splits.

I. Following a stock split, an investor owns an increased percentage of the company.

II. Because of the increased number of shares an investor owns following a stock split, the stock split is a taxable event to the investor.

III. A stock split results in a downward adjustment in a shareholder’s per share basis.

IV. A reverse stock split reduces the total shares outstanding and increases the price per share.

A) II and IV
B) I and III
C) III and IV
D) I, II, III, and IV

A

Explanation
The answer is III and IV.

A stock split is not a taxable event to the investor. Following a stock split, the investor owns the same percentage of the company as before, but each share now represents a correspondingly smaller percentage.

LO 3.2.1

How well did you know this?
1
Not at all
2
3
4
5
Perfectly
104
Q

Identify the CORRECT statements regarding a 2-for-1 stock split.

I. The total market value of the outstanding stock decreases.

II. The total number of shares outstanding doubles.

III. The share price is reduced by one-half.

IV. The shareholder will own a higher proportion of the company.

A) I and III
B) II, III, and IV
C) I and II
D) II and III

A

Explanation
D) The answer is II and III.

In a 2-for-1 stock split, the number of outstanding shares is doubles and the share price is reduced by one-half. The total market value of the company’s stock and the shareholder’s interest remains the same.

LO 3.2.1

How well did you know this?
1
Not at all
2
3
4
5
Perfectly
105
Q

John has just been informed that his XYZ stock will be incurring a 2-for-1 stock split. Calculate how many additional shares John will acquire if he already owns 200 shares of XYZ.

A) 50
B) 100
C) 200
D) 400

A

Explanation
C) The answer is 200.

John will acquire 200 additional shares and therefore, will own a total of 400 shares of XYZ stock. In a stock split, the par value of each share of stock is reduced, and the number of shares is increased proportionately.

LO 3.2.1

How well did you know this?
1
Not at all
2
3
4
5
Perfectly
106
Q

Which of these statements correctly explains dividend reinvestment programs (DRIPs)?

I. They allow stockholders to reinvest their dividends in additional shares of stock.

II. They provide for automatic investing.

III. They provide cost savings to firms and shareholders.

IV. Reinvested dividends are not currently taxable to the investor.

A) II and III
B) I and II
C) I, II, and III
D) I, II, III, and IV

A

Explanation

The answer is I, II, and III.

Only statement IV is incorrect. The IRS treats reinvested dividends the same as cash dividends for tax purposes.

LO 3.2.1

How well did you know this?
1
Not at all
2
3
4
5
Perfectly
107
Q

Which of these statements regarding cash dividends is CORRECT?

I. All cash dividends paid to shareholders are tax deductible by the corporation.

II. Most dividends declared by a corporate board of directors of a domestic corporation are considered qualifying dividends and are permitted preferential tax treatment.

A) II only
B) Neither I nor II
C) I only
D) Both I and II

A

Explanation
The answer is II only.

Cash dividends paid by a corporation to the shareholders are not tax deductible. Qualifying (or qualified) dividend income may be treated at favorable long-term capital gain rates (0%, 15%, or 20%) if it meets certain criteria.

LO 3.2.1

How well did you know this?
1
Not at all
2
3
4
5
Perfectly
108
Q

RNO Mutual Fund invests in domestic debt and equity securities. The fund’s current bond holdings are valued at $63 million, and its equity holdings are valued at $85 million. RNO currently has 3 million outstanding shares; although it is not limited in the number of shares it may sell. Which of these statements is CORRECT?

A) RNO is a closed-end investment company.
B) RNO shares are traded on the major exchanges.
C) RNO shares are priced at $49.33 per share.
D) RNO is a money market mutual fund.

A

Explanation
The answer is RNO shares are priced at $49.33 per share.

Because RNO invests in both bonds and equities, it is not a money market mutual fund. RNO’s shares are valued at NAV, calculated as follows: ($63 million + $85 million) ÷ 3 million shares = $49.33 per share.

Mutual fund shares are sold and redeemed directly by the mutual fund company and do not trade on the major exchanges. RNO is an example of an open-end investment company.

LO 3.3.1

How well did you know this?
1
Not at all
2
3
4
5
Perfectly
109
Q

Which of the following statements regarding an open-end investment company (mutual fund) is CORRECT?

I. After the initial public offering, an open-end fund will generally not issue additional shares.

II. An open-end investment company is the most popular form of investment asset for small or individual investors.

A) II only
B) Neither I nor II
C) I only
D) Both I and II

A

Explanation
A) The answer is II only.

Statement I is incorrect. An open-end investment company can raise an unlimited amount of capital by continuously issuing new shares. After the initial public offering, a closed-end fund will generally not issue additional shares.

LO 3.3.1

How well did you know this?
1
Not at all
2
3
4
5
Perfectly
110
Q

Grace’s portfolio is comprised of 40% U.S. corporate bond fund, 50% U.S. growth and income equity fund, and 10% municipal bond fund. Grace would like to reduce her portfolio’s level of risk and maintain or improve return. Which of these could be recommended to Grace to achieve her goal?

I. Global equity fund
II. Biotechnology sector equity fund
III. Louisiana municipal bond fund
IV. Emerging market fund

A) III only
B) II, III, and IV
C) I only
D)I and IV

A

Explanation
The answer is I and IV.

Adding foreign investments could reduce her portfolio’s level of risk and possibly improve return. Foreign investments have a low correlation with U.S. securities and thus provide diversification benefits. Additional investment in U.S. equities or bonds will not provide as much diversification as international investing.

LO 3.3.1

How well did you know this?
1
Not at all
2
3
4
5
Perfectly
111
Q

All of these statements correctly describe types of investment companies EXCEPT

A) a unit investment trust is a type of investment company whose units are sold in the secondary market but not on the major exchanges.

B) a closed-end investment company is a type of company whose shares trade in the same manner as publicly traded stocks in the secondary market.

C) an exchange-traded fund is an investment that can be bought and sold throughout the trading day.

D) an open-end investment company is categorized as open-end because it is limited in the number of shares that are sold.

A

Explanation
The answer is an open-end investment company is categorized as open-end because it is limited in the number of shares that are sold.

An open-end investment company is the most popular form of investment for the small or individual investor. The investment company is categorized as open-end because it is not limited in the number of shares sold.

LO 3.3.1

How well did you know this?
1
Not at all
2
3
4
5
Perfectly
112
Q

Louis owns an investment that is an unmanaged portfolio in which the money manager initially selects the securities to be included in the portfolio and then holds those securities until they mature or the investment portfolio terminates. This statement best describes which type of investment?

A) Closed-end investment company
B) Unit investment trust
C) Open-end investment company
D) Hedge fund

A

Explanation
The answer is unit investment trust. A unit investment trust (UIT) is an investment company whose units are sold in the secondary market and is generally unmanaged, or passively managed as the money manager initially selects the securities to be included in the portfolio and then holds those securities until they mature or the UIT terminates.

LO 3.3.1

How well did you know this?
1
Not at all
2
3
4
5
Perfectly
113
Q

Select the type of mutual fund that generally focuses its investment objective in a narrow area such as natural resources, technology, or health care.

A) Growth funds
B) Balanced funds
C) Income funds
D) Sector funds

A

Explanation
The answer is sector funds.

Sector funds tend to limit their investments to one sector of the economy, such as natural resources, technology, or health care.

LO 3.3.1

How well did you know this?
1
Not at all
2
3
4
5
Perfectly
114
Q

All of these statements correctly identifies a separately managed account except

A) one advantage of a separately managed account is the ability to maintain an individual cost basis in the securities held in the account.

B) a separately managed account holds a diversified portfolio of securities managed by a professional money manager.

C) in a separately managed account, the investor owns 100% of the securities in the account.

D) a separately managed account is a privately offered pool of capital for wealthy, sophisticated investors.

A

Explanation
D) The answer is a separately managed account is a privately offered pool of capital for wealthy, sophisticated investors.

In a separately managed account, the money manager purchases securities on behalf of the individual investor, not as a pool of capital for numerous investors.

LO 3.3.1

How well did you know this?
1
Not at all
2
3
4
5
Perfectly
115
Q

Daniel has several investment company products within his retirement portfolio. One of these investments trades on an exchange, may trade at a premium or discount to its net asset value, and has a fixed capital structure. These features illustrate which of these investments?

A) Hedge fund
B) Open-end investment company
C) Closed-end investment company
D) Unit investment trust

A

Explanation
The answer is closed-end investment company.

A closed-end investment company (closed-end fund) is a type of company whose shares trade in the secondary market.

LO 3.3.1

How well did you know this?
1
Not at all
2
3
4
5
Perfectly
116
Q

During the current year, Peter sold 300 mutual fund shares for $23 per share. He had purchased these shares over several years as follows: 75 shares in year one for $13 per share; 100 shares in year two for $15 per share; 100 shares in year three for $17 per share; and 90 shares in year four for $12 per share. Before the sale of the shares, Peter specified to his broker the particular shares to be sold, and he received written confirmation of his selection from the broker. In specifying the specific shares he wanted to sell, Peter selected the shares that would produce the smallest amount of taxable gain on the sale. Calculate the cost basis of the shares selected and the amount of realized gain on the sale.

A) $4,000 cost basis, $2,900 gain on sale
B) $4,475 cost basis, $2,425 gain on sale
C) $4,319 cost basis, $2,581 gain on sale
D) $4,410 cost basis, $2,490 gain on sale

A

Explanation
The answer is $4,475 cost basis, $2,425 gain on sale. The highest cost shares are as follows: (100 × $17) + (100 × $15) + (75 × $13) + (25 × $12) = $4,475. The gain is $2,425 ($6,900 − $4,475).

LO 3.3.1

How well did you know this?
1
Not at all
2
3
4
5
Perfectly
117
Q

Which of the following methods can be used in determining the basis in a mutual fund when the shares were acquired at different times?

I. Specific identification
II. First in, first out (FIFO)
III. Average cost method

A) I, II, and III
B) II and III
C) I and III
D) II only

A

Explanation
The answer is I, II, and III. All of these can be used to make this determination.

LO 3.3.1

How well did you know this?
1
Not at all
2
3
4
5
Perfectly
118
Q

Choose the statement that is NOT a characteristic of a closed-end investment company (closed-end fund).

A) A closed-end fund typically redeems its shares as a courtesy to all shareholders.
B) Like common stocks, shares trade in the secondary market.
C) Shares may sell at a discount or premium to net asset value.
D) The fund manager of a closed-end fund may invest in less liquid securities than a manager of an open-end fund.

A

Explanation
The answer is a closed-end fund typically redeems its shares as a courtesy to all shareholders.

Generally, a closed-end investment company does not redeem its own shares. Closed-end fund shares trade in the same manner as publicly traded stocks, either on a stock exchange or over the counter.

LO 3.3.1

How well did you know this?
1
Not at all
2
3
4
5
Perfectly
119
Q

Choose the CORRECT statements regarding exchange-traded funds (ETFs) and hedge funds.

I. ETFs typically have higher annual expenses than mutual funds.

II. Most ETFs permit tax-free, in-kind redemptions.

III. ETFs can be either passively or actively managed.

IV. Hedge funds typically use margin and short-selling strategies.

A)I and IV
B) I, II, III, and IV
C) II, III, and IV
D) II and III

A

Explanation
The answer is II, III, and IV.

Because they mimic indexes, ETFs typically have lower annual expenses than mutual funds. Most ETFs permit in-kind redemptions, and such transactions are income tax free. Hedge funds rely heavily on borrowing strategies in an attempt to accomplish superior returns.

LO 3.4.1

How well did you know this?
1
Not at all
2
3
4
5
Perfectly
120
Q

Select the CORRECT statement regarding hedge funds.

A) Short sales by the fund are not allowed.
B) The fund is required to register with FINRA prior to soliciting potential clients.
C) Purchasers of hedge funds are required to be accredited investors.
D) Hedge fund managers are paid on a commission basis.

A

Explanation
The answer is purchasers of hedge funds are required to be accredited investors.

A hedge fund is an unregistered, privately offered, managed pool of capital for wealthy investors. In addition to short selling, a hedge fund will implement a wide array of risky trading strategies in order to exploit market inefficiencies. Hedge fund managers are paid based on fund performance.

LO 3.4.1

How well did you know this?
1
Not at all
2
3
4
5
Perfectly
121
Q

Identify those risks that pertain to hedge funds.

Overuse of leverage
Excessive short selling
Lack of transparency
Lack of regulation

A) II, III, and IV
B) I and IV
C) I, II, III, and IV
D) II and III

A

Explanation
The answer is I, II, III, and IV.

A hedge fund is subject to all these risks. In addition, hedge funds are usually subject to higher investment risk than other types of funds.

LO 3.4.1

How well did you know this?
1
Not at all
2
3
4
5
Perfectly
122
Q

Select the entity that issues guaranteed investment contracts (GICs).

A) Insurance companies
B) Credit unions
C) Commercial banks
D) Open-end investment companies

A

Explanation
The answer is insurance companies. GICs are issued by insurance companies. They are called guaranteed investment contracts because their rate of return is guaranteed by the insurance company for a fixed period. They are not guaranteed by the FDIC.

LO 3.4.1

How well did you know this?
1
Not at all
2
3
4
5
Perfectly
123
Q

An investor should choose index funds if he or she believes in

A) fundamental analysis.
B) specialized investing.
C) a passive investment strategy.
D) technical analysis.

A

Explanation
The answer is a passive investment strategy.

Index funds are used in a passive investment strategy. The purpose of an indexed portfolio is not to beat the targeted index (e.g., S&P 500 Index) but merely to match its long-term performance, less any management fees and administrative costs.

LO 3.4.1

How well did you know this?
1
Not at all
2
3
4
5
Perfectly
124
Q

Which of these statements correctly explains dollar cost averaging as a portfolio management technique?

I. This technique involves investing a specific amount into an investment vehicle, regardless of whether the recent trend in the investment has been up or down.

II. If prices decline, the fixed investment amount will purchase a greater quantity of the security.

III. For the long-term investor, the presumption is that prices will eventually rise, so a lower average price translates into greater profits.

IV. If prices rise, the fixed investment amount will purchase a greater amount of the security.

A) I, III, and IV
B) II and III
C) I and II
D) I, II, and III

A

Explanation
The answer is I, II, and III. Only statement IV is incorrect. If prices rise, the fixed investment amount will purchase a lower amount of the security. The effect of dollar cost averaging is to increase the number of shares gradually over a long period. Because more shares are acquired when the price of the stock or mutual fund declines, the average cost per share is reduced.

LO 3.5.1

How well did you know this?
1
Not at all
2
3
4
5
Perfectly
125
Q

To be on a corporation’s books as holder-of-record (and thus have a right to the next dividend payment), the investor must purchase stock

A) two business days before the record date.
B) before the declaration date.
C) three days before the payment date.
D) between the ex-dividend date and the record date.

A

Explanation
The answer is two business days before the record date.

Under the T+2 rules in effect, ex-dividend date is one day business prior to the record date. A trade made on the ex-dividend date will clear in two business days, one day after the record date. The investor will not be on the corporation’s record book as a shareholder unless the purchase is made at least two days before the record date.

LO 3.5.1

How well did you know this?
1
Not at all
2
3
4
5
Perfectly
126
Q

Under which of these circumstances will dollar cost averaging result in an average cost per share lower than the average price per share?

I. The price of the stock fluctuates over time.
II. A fixed number of shares is purchased regularly.
III. A fixed dollar amount is invested regularly.
IV. A constant dollar plan is maintained.

A) I, III and IV
B) I and III
C) II and III
D) I and II

A

Explanation
The answer is I and III.

Dollar cost averaging benefits the investor if the same amount is invested on a regular basis over a substantial period, during which the price of the stock fluctuates. A constant dollar plan is one in which the investor maintains a constant dollar value of securities in the investment portfolio.

LO 3.5.1

How well did you know this?
1
Not at all
2
3
4
5
Perfectly
127
Q

You are about to choose a new mutual fund to add to client portfolios. As you review the Morningstar reports for the funds you are considering, you have focused on each fund’s alpha as reported by Morningstar. Alpha tells you

A) a fund’s percentage return above the risk-free rate of return.
B) each fund’s performance relative to the S&P 500.
C) the difference between a fund’s realized return and its risk-adjusted expected return.
D) by what percentage a fund’s capital appreciation exceeded the capital appreciation of the average fund in its asset class.

A

Explanation
C) The answer is the difference between a fund’s realized return and its risk-adjusted expected return.

Alpha does not compare directly to the S&P 500, but rather to the fund’s expected return, which is risk-adjusted for the fund’s beta. The total return, not just the capital appreciation component, is used in the Jensen formula. The risk-adjusted required return is the risk-free rate plus the risk premium multiplied by the fund’s beta.

LO 3.6.1

How well did you know this?
1
Not at all
2
3
4
5
Perfectly
128
Q

Question #30 of 30
Question ID: 1240128
Two mutual funds have these performance statistics:

Fund E. Fund F
Three-year total return
16.5% 17.2%

Standard deviation
18.1 16.4

R-squared
81%. 87%

Sharpe ratio
.58 .68

Alpha
1.1 1.6

Which one of the two funds has the better risk-adjusted performance, and why?

A)Fund E, because its Sharpe ratio is lower.
B)Fund F, because its R-squared is higher.
C)Fund E, because its coefficient of variation is lower.
D)Fund F, because its alpha is higher.

A

Explanation
The answer is Fund F, because its alpha is higher.

In this case, the investor should choose the fund with the higher alpha. With an alpha of 1.6, Fund F exhibits the best risk-adjusted performance.

LO 3.6.1

How well did you know this?
1
Not at all
2
3
4
5
Perfectly
129
Q

All of the following are considered risks inherent with equity investments except

A) investment rate risk.

B) reinvestment rate risk.

C) market risk.

D) financial risk.

A

Explanation
Interest rate risk, market risk, business risk, and financial risk are all risks that impact equity investments. Reinvestment rate risk is a primary consideration for fixed-income investments.

LO 3.1.1

How well did you know this?
1
Not at all
2
3
4
5
Perfectly
130
Q

Gordon, age 40, wants to invest in a mutual fund that will provide capital appreciation. He wants a fund that will do as well as the overall market and has a low expense ratio, but he does not want to assume a high risk to achieve his objective. He is considering purchasing one of the following mutual funds:

Fund A: a growth mutual fund that has a beta of 1.10 and invests in medium- to high-grade common stock

Fund B: an index mutual fund that has a beta of 1.00 and invests in common stock that mirrors the S&P 500 Index

Which of these funds would best meet Gordon’s objective?

A) Fund A, because it can be expected to outperform the market and has an acceptable level of risk

B) neither alternative is appropriate for his objective

C) Fund A, because it invests in lower-risk stocks than Fund B

D) Fund B, because it has a beta of 1.00, has low expenses, and is less risky

A

Explanation

D) Fund B can be expected to do as well as the overall market, will have a low expense ratio, and is less risk, as measured by beta, than Fund A.

LO 3.6.1

How well did you know this?
1
Not at all
2
3
4
5
Perfectly
131
Q

Which of the following statements is CORRECT regarding exchange-traded funds (ETFs)?

A) They trade right at net asset value due to arbitrage transactions.

B) They are permitted to sample an index.

C) They can be exchanged for other ETFs in the same family.

D) They are usually tax-inefficient.

A

Explanation

B) Rather than buy every security in an index, some ETFs only sample the index, called representative sampling, especially when the index contains a very large number of issues.

LO 3.4.1

How well did you know this?
1
Not at all
2
3
4
5
Perfectly
132
Q

Which of the following are advantages of dividend reinvestment plans?

I. The investor avoids having to account for the cost basis of shares.

II. Discounts are available on stocks bought with dividends.

III. They are a convenient and low-cost way of accumulating shares of stock.

IV. Reinvested dividends are tax-deferred income.

A) I and II

B) I, II, and III

C) III and IV

D) II and III

A

Explanation

D) Option IV is an incorrect statement because dividends are not tax deferred.

They are taxable (potentially at preferential rates) as income at the time they are received. Option I is incorrect because cost basis is needed to compute taxable income when the shares are sold in the future.

LO 3.2.1

How well did you know this?
1
Not at all
2
3
4
5
Perfectly
133
Q

Which of the following statements are NOT correct regarding Barbara’s participation in the dividend reinvestment plan with First Mutual Growth Fund?

I. The amount of dividend reinvestment adds to Barbara’s tax basis in the fund.

II. Because they are directly reinvested in the fund, the reinvested dividends are not subject to current taxation.

III. Typically, additional shares purchased through dividend reinvestment plans do not incur a brokerage commission.

IV. The term dividend reinvestment refers to the strategy of receiving dividends in cash from one investment and reinvesting them in a different investment offering higher returns.

A) II and IV

B) I and II

C) III and IV

D) I and III

A

Explanation

A) Statements II and IV are incorrect.

Reinvested dividends are treated the same as dividends received in cash for tax purposes. The term dividend reinvestment refers to dividends that are reinvested directly into the investment from which they were initially earned.

LO 3.2.1

How well did you know this?
1
Not at all
2
3
4
5
Perfectly
134
Q

A middle-aged husband and wife have their portfolio currently invested in a diversified mix that includes long- and short-term bonds, savings account, certificates of deposit (CDs), and growth and value stock funds. They are not pleased with their stock investments, which seem to only be matching the market’s poor performance. Which of the following types of stock funds are they most likely invested in?

A) Index fund

B) Technology fund

C) Balanced fund

D) Bond fund

A

Explanation

A) An index fund would match the market’s performance. In a poor market, the technology fund would have even greater losses and the balanced fund should do better because of the bond mix.

LO 3.6.1

How well did you know this?
1
Not at all
2
3
4
5
Perfectly
135
Q

Which of the following is a type of growth mutual fund?

A) GNMA fund

B) Money market fund

C) Asset allocation fund

D) U.S. government fund

A

Explanation
An asset allocation fund is a type of growth fund. The other options are all considered income funds.

LO 3.3.1

How well did you know this?
1
Not at all
2
3
4
5
Perfectly
136
Q

A fund that invests in both U.S. stocks and international stocks is called

A) an asset allocation fund.

B) an international fund.

C) a balanced fund.

D) a global fund.

A

Explanation

A global fund invests in U.S. stocks and in international stocks.

How well did you know this?
1
Not at all
2
3
4
5
Perfectly
137
Q

Which of the following best represents a characteristic of balanced mutual funds?

A) They combine domestic and international securities.

B) They combine high- and low-risk stock.

C) They combine income and growth stock.

D) They combine debt and equity securities.

A

Explanation

D) Balanced funds can be securities that produce both income (dividends and interest) and capital gains. Income securities can be high-dividend paying stocks or bonds. Capital gains securities generally are stocks. Most balanced funds contain some combination of both stocks and bonds (often 60% stocks and 40% bonds).

LO 3.3.1

How well did you know this?
1
Not at all
2
3
4
5
Perfectly
138
Q

Which of the following statements concerning dollar cost averaging as a portfolio management technique are CORRECT?

I. This technique involves investing a specific amount into an investment vehicle, regardless of whether the recent trend in the investment has been up or down.

II. If prices decline, the fixed investment amount will purchase a greater quantity of the security.

III. For the long-term investor, the presumption is that prices will eventually rise, so a lower average price translates into greater profits.

IV. If prices rise, the fixed investment amount will purchase a greater amount of the security.

A) I, II, and III

B) II and III

C) I and II

D) I, III, and IV

A

Explanation

A) Only statement IV is incorrect. If prices rise, the fixed investment amount will purchase a lower amount of the security.

LO 3.5.1

How well did you know this?
1
Not at all
2
3
4
5
Perfectly
139
Q

Which of the following are characteristics of exchange-traded funds (ETFs)?

I. They may not be sold short.
II. They are generally tax-efficient.
III. Large investors known as authorized participants buy or sell shares on an “in-kind” basis.
IV. They usually trade at or near their net asset value.

A) II, III, and IV

B) I, II, and III

C) II and IV

D) I and III

LO 3.4.1

A

Explanation

A) ETFs trade like stock and can be sold short. They are tax-efficient, generally low cost, and large investors conduct trades by making in-kind exchanges, whereby they give or receive shares of stock that are in the fund. Generally ETFs trade near net asset value (NAV), if not at NAV.

How well did you know this?
1
Not at all
2
3
4
5
Perfectly
140
Q

An investor using a dollar cost averaging approach to buying a mutual fund will buy

A) the same number of shares regardless of which direction the NAV takes.

B) more shares when the NAV of the fund rises since the last purchase.

C) more shares when the NAV of the fund falls since the last purchase.

D) fewer shares when the NAV of the fund rises since the last purchase.

A

Explanation

C) The investor will purchase more shares when the NAV falls because the dollar amount invested remains the same.

LO 3.5.1

How well did you know this?
1
Not at all
2
3
4
5
Perfectly
141
Q

Chelsea purchases a warrant for $2 per share that gives her the right to buy 80 shares of XYZ stock at $20 per share for a period of five years from date of purchase. Assume XYZ stock goes up to $25 per share after three years and Chelsea exercises the warrant. What profit does she make on the 80 shares?

A) $210

B) $150

C) $180

D) $240

A

Explanation

D) profit = (gain on stock – cost of warrant) × number of shares; ($5/share – $2/share) × 80 shares = $240

LO 3.1.1

How well did you know this?
1
Not at all
2
3
4
5
Perfectly
142
Q

A client wants to buy a mutual fund that offers above-average dividend income, lower volatility than the average stock fund, and has some capital appreciation potential. Which of the following is the best recommendation under these criteria?

A) Asset allocation fund

B) Stock index fund

C) Small company fund

D) Equity income fund

A

Explanation

D) An equity income fund would fit these criteria by owning stocks of companies paying above-average dividends.

A stock index fund is too general a description in that it could own small company stocks that pay little, if any, dividends, and are more volatile than large company stocks. An asset allocation fund would own stocks, bonds, cash equivalents, precious metal stocks, and even currencies, so income would not necessarily be above-average.

LO 3.3.1

How well did you know this?
1
Not at all
2
3
4
5
Perfectly
143
Q

Which of the following is a general characteristic of hedge funds?

A) Little or no use of leverage

B) Full transparency and disclosure

C) Charge both a management fee and a carried interest fee

D) High marketability

A

Explanation

C) Hedge funds have few public disclosure requirements, may lack marketability, and use leverage.

LO 3.4.1

How well did you know this?
1
Not at all
2
3
4
5
Perfectly
144
Q

Holly and Jeff are a married couple who have recently retired and are no longer earning an income. They would like to change their asset allocation to provide more income in their retirement years. Which of the following investments should be recommended to help the couple in achieving their financial objectives?

I. Aggressive growth mutual fund
II. AA rated corporate bonds
III. Zero-coupon bonds
IV. Equity income mutual fund

A) I, II, and IV

B) II, III, and IV

C) II and IV

D) I and III

A

Explanation

Investments II and IV are the only two that will provide current income for the couple.

LO 3.6.1

How well did you know this?
1
Not at all
2
3
4
5
Perfectly
145
Q

A couple has noted they are aggressive risk-takers and want to allocate $10,000 from their savings account into investments. Currently, their portfolio is comprised of money market mutual funds and blue-chip stocks. Which of the following stocks would be the best addition for this couple to achieve their risk-taking attitude?

A) Semiconductor stock

B) Waste management stock

C) Domestic airline stock

D) Big box store stock

A

Explanation

A) The best choice for this couple would be a semiconductor stock; as a competitive and high-growth sector, semiconductors should add the appropriate degree of risk and return for this couple’s conservative portfolio. A big box store stock would be considered defensive, a waste management stock would be an income stock, and a domestic airline stock would be considered a cyclical stock.

LO 3.1.1

How well did you know this?
1
Not at all
2
3
4
5
Perfectly
146
Q

Long-term bond funds have

A) minimal purchasing power risk.

B) more interest rate risk than short-term bonds.

C) no interest rate risk.

D) no reinvestment rate risk.

A

Explanation

Long-term bonds have greater purchasing power risk than short-term bonds.

LO 3.3.1

How well did you know this?
1
Not at all
2
3
4
5
Perfectly
147
Q

A stock has a current market price of $48, expected earnings of $2.40, and an annual dividend of $.72. The stock historically has had a price-to-earnings (P/E) ratio range of 22 to 35. Which of the following are CORRECT statements about the stock’s P/E ratio, valuation, and possible future action?

I. P/E = 20
II. P/E = 67
III. Stock is overvalued, downward pressure expected
IV. Stock is undervalued, upward pressure expected

A) I and III

B) I and IV

C) II and III

D) II and IV

A

Explanation

B) The P/E ratio is 20, calculated as follows:
$48 ÷ $2.40 = 20.

When a stock exhibits a P/E ratio below the historical range, this could indicate the stock is undervalued in the market.

LO 4.3.2

How well did you know this?
1
Not at all
2
3
4
5
Perfectly
148
Q

Interest rate changes have the greatest effect on

A) long-term bonds.

B) staggered maturity bonds.

C) short-term bonds.

D) medium-term bonds.

A

Explanation
A) The rule-of-thumb approach to measuring the estimated price change of a bond is to multiply the bond’s duration by the estimated change in interest rates (for small rate changes—less than 1% or 100 basis points—only). Therefore, longer-term bonds have the greatest duration and the most price volatility.

LO 4.1.1

How well did you know this?
1
Not at all
2
3
4
5
Perfectly
149
Q

Which of the following are factors used in industry analysis for investment purposes?

I. Financial leverage
II. Government rules and regulations
III. Labor conditions
IV. Technological advances

A) I and II

B) II, III, and IV

C) I, II, and IV

D) III and IV

LO 4.2.1

A

Explanation

B) Option I is not a factor used for industry analysis, but rather for company analysis.

How well did you know this?
1
Not at all
2
3
4
5
Perfectly
150
Q

Company A and Company B are in the same industry and have approximately the same dollar amount of assets and operating income. Company A has a return on equity (ROE) of 28% and Company B has an ROE of 12%. Which of the following statements best identifies the major difference causing the disparity in ROE between Company A and Company B?

A) Company B has an extraordinary loss.

B) Company A has lower selling, general, and administrative (S, G, & A) expenses.

C) Company B has a higher level of depreciation expense than Company A.

D) Company A has more debt than Company B.

A

Explanation

D) Both depreciation expense and S, G & A expenses are used to obtain operating income, which is the same for both companies. Generally, the most significant factor in raising one company’s ROE above another company’s is the greater use of debt. The company having the greater percentage of debt, assuming the cost of the debt is less than the return earned from the debt proceeds, will have the highest ROE.

LO 4.2.2

How well did you know this?
1
Not at all
2
3
4
5
Perfectly
151
Q

Which one of the following statements CORRECTLY matches a technical indicator to the information it provides in signaling a change from a bear to a bull market?

A) Barron’s Confidence Index indicates that the yield differential between low-quality bonds and high-quality bonds is decreasing.

B) Odd lot purchases exceed sales.

C) A moving average chart indicates that actual prices have dropped through the average.

D) Most financial advisors become bullish.

A

Explanation

A) In a bull market, there is less fear so there is a lower spread between high-quality and lower-quality bonds.

LO 4.2.1

How well did you know this?
1
Not at all
2
3
4
5
Perfectly
152
Q

Al and Stacy have a long-term goal to retire in 20 years and have $1,000 per month that they would like to invest. They are currently in the top tax bracket but expect to be in a lower tax bracket at retirement. They have a moderate- to high-risk tolerance. Which of the following asset allocation strategies should be recommended to Al and Stacy?

A) Portfolio 3—$200 Balanced Fund, $200 S&P Index Fund, $300 Growth Fund, $300 Traditional IRA (U.S. Government Bond Fund)

B) Portfolio 2—$500 Bond Fund, $300 Growth Fund, $200 Balanced Fund

C) Portfolio 1—$300 Balanced Fund, $400 Growth Fund, $300 Roth IRA (U.S. Government Bond Fund

D) Portfolio 4—$100 Money Market Fund, $200 Balanced Fund, $200 Growth Fund, $200 S&P Index Fund, $300 Roth IRA (U.S. Government Bond Fund)

A

A)
Explanation
Because Al and Stacy are in the top tax bracket, they do not qualify for a Roth IRA, therefore eliminating portfolios 1 and 4. Portfolio 2 is too conservative for their risk tolerance and long-term goals.

LO 4.1.1

How well did you know this?
1
Not at all
2
3
4
5
Perfectly
153
Q

Identify basic assumptions pertaining to the constant growth dividend discount model.

I. The growth rate cannot be equal to or greater than the investor’s required rate of return.

II. The model assumes that the investor’s required rate of return is known and remains constant.

A) II only

B) Both I and II

C) I only

D) Neither I nor II

A

Explanation

B) The growth rate cannot be equal to or greater than the investor’s required rate of return or the denominator becomes meaningless.

The model assumes the required rate of return is constant.

LO 4.3.1

How well did you know this?
1
Not at all
2
3
4
5
Perfectly
154
Q

Which of the following statements is CORRECT regarding fundamental and technical analysis?

A) Technical analysis is considered to be valid only under the weak form of the efficient market hypothesis (EMH).

B) Fundamental analysis may provide better returns under the weak form of the efficient market hypothesis (EMH).

C) Top-down analysis starts with examining individual companies, and then examining the impact of the overall economy on that particular company.

D) An example of technical analysis would be an investor looking at debt-to-equity ratios and price-to-earnings ratios.

A

B)Explanation
Technical analysis is not considered valid under any of the three forms of the EMH. Ratio analysis fits with fundamental analysis techniques. Bottom-up analysis begins with identifying attractive individual companies for investment. Fundamental analysis may indeed allow for an investor to outperform the market, but only under the weak form of the EMH.

LO 4.2.1

How well did you know this?
1
Not at all
2
3
4
5
Perfectly
155
Q

A 7% coupon bond pays interest semiannually and has a duration of 12 (computed using semiannual compounding) and a maturity of 25 years. The bond sells for $1,100 and has a YTM of 6.2%. If the YTM is expected to increase by 50 basis points, by what percentage can the price of the bond be expected to change?

A) –5.82%

B) +5.82%

C) –5.65%

D) +5.65%

A

A)
Explanation
Because interest rates are expected to increase, the percentage change in price will be negative. Because semiannual compounding was used to compute duration, modified duration must be computed by using one-half the current YTM (3.1%). The computation using the formula is as follows.

ΔP
P=
 −D×Δy
1+y
= 
−12×0.005
1.031=−0.0582=−5.82%
LO 4.1.2
How well did you know this?
1
Not at all
2
3
4
5
Perfectly
156
Q

Which of the following are NOT used in technical analysis?

A) Moving averages

B) Graphs

C) Supply and demand of stocks

D) Financial statement ratios

A

Explanation

D) Financial statement ratios are part of fundamental analysis.

LO 4.2.1

How well did you know this?
1
Not at all
2
3
4
5
Perfectly
157
Q

Which of the following statements regarding fundamental and technical analysis is CORRECT?

A) In top-down analysis, an investor would start by researching various industries, and then choose stocks within that industry.

B) Technical analysis is not considered valid under the efficient market hypothesis, because this type of analysis is attempting to predict future prices based on past price movement.

C) Fundamental analysis may result in better returns than the overall market under both the weak and semistrong forms of the efficient market hypothesis.

D) Investors looking for excellent companies to invest in may use bottom-up analysis, which is a form of technical analysis.

A

B)
Explanation
This is correct, as any form of EMH does not coexist with technical analysis.

LO 4.2.1

How well did you know this?
1
Not at all
2
3
4
5
Perfectly
158
Q

Kevin owns a $1,000 par value corporate bond with three years remaining until maturity. This bond is currently trading for $1,020.91. The bond has a coupon rate of 4.5% (annual coupon payments) and a current YTM of 3.75%. What is the duration of this bond?

A) 2.0910

B) 2.8741

C) 1.0067

D) 3.7500

A

B)
Explanation
Determine the duration of the bond.

Year. Cash Flow(CF) (PV) of CF PV × Year

  1. $45 $43.37 $43.37
  2. $45 $41.81 $83.62
  3. $1,045 $935.73 $2,807.19
                                  $1,020.91         $2,934.18

Divide the sum in the last column ($2,934.18) by the total PV/market price of the bond ($1,020.91) to derive the duration of 2.8741 years.

For year 1, FV = 45, I/YR = 3.75, N = 1, solve for PV.

For year 2, change N to 2 without clearing your calculator and solve for PV.

For year 3, FV = 1,045, I/YR = 3.75, N = 3, solve for PV.

LO 4.1.2

How well did you know this?
1
Not at all
2
3
4
5
Perfectly
159
Q

Assume a $1,000 par value bond with 3 years until maturity is currently trading for $1,027.23. The bond has a coupon rate of 6% (annual coupon payments) and a current YTM of 5%. The bond has a duration of 2.51 years. Calculate what the new market price for the bond would be if the YTM changed from 5% to 4.5%.

A) $1,016.75

B) $1,032.36

C) $1,053.01

D) $1,041.23

A

Explanation
The new price of the bond should be $1,041.23.

FV = 1,000

PMT = 60

I/YR = 4.5

N = 3

Solve for PV = –1,041.2345, or $1,041.23

LO 4.1.1

How well did you know this?
1
Not at all
2
3
4
5
Perfectly
160
Q

Companies A and B have exactly the same dollar amount of assets and net income. Company A has a capitalization structure of 70% equity and 30% debt; Company B has a capitalization structure of 40% equity and 60% debt. Which of the following statements is CORRECT?

A) Company B has a higher return on equity (ROE) than Company A.

B) Company B has a higher return on assets (ROA) than Company A.

C) Company A has a higher earnings before interest, tax, depreciation, and amortization (EBITDA) than Company B.

D) Company A has a higher debt-to-equity ratio than Company B.

A

A) Explanation
All else being equal, a profitable company with a higher debt level will have a higher return on equity. If income is the same for both companies, then the only difference is the percentage of equity. With a lower equity, Company B will have a higher return on equity. EBITDA and ROA would be equal, and Company B has a higher debt-to-equity ratio.

LO 4.2.2

How well did you know this?
1
Not at all
2
3
4
5
Perfectly
161
Q

All of the following statements concerning convexity are CORRECT except

A) high-duration bonds have low convexity.

B) convexity is a measurement of the curvature of the price-yield relationship.

C) convexity is a measure of how much a bond’s price-yield curve deviates from the linear approximation of that curve.

D) convexity is likely to be greatest with low-coupon bonds, long-maturity bonds, and low-YTM bonds.

A

A)
Explanation
The determinants of both duration’s and convexity’s direction of impact is the same (maturity [direct relationship], coupon rate, and yield [both inverse relationships]). Therefore, high-duration bonds also have high convexity.

LO 4.1.2

How well did you know this?
1
Not at all
2
3
4
5
Perfectly
162
Q

Two stocks, in different industries, have the following characteristics:

                      Stock A                           Stock B

Market price. $27 $47

Dividend 5% None

Dividend 4.5% N/A
growth rate

Earnings $2.46 $2.14
per share

P/E ratio 11 22

Which of the following statements is true?

I. A comparison of the relative merits of the two stocks cannot be evaluated because the dividend growth model cannot be used for Stock B.

II. The average price-to-earnings (P/E) ratio of the industry of each stock is important to evaluate overvaluation or undervaluation.

III. The historic range of each stock’s P/E ratio is important to evaluate each stock’s relative value.

IV. Stock B is undervalued relative to Stock A because its P/E ratio is twice Stock A’s P/E ratio, yet its market price is less than twice Stock A’s market price.

A) I and IV
B) IV only
C) II and III
D) I only

A

C) Explanation
The dividend growth model is just one of many models that investment analysts use to estimate the intrinsic value of a company. Other models can be used if a company does not pay a dividend. Determining intrinsic value by comparing relative P/E ratios compared to relative market prices is not a valid approach to valuation.

LO 4.3.2

How well did you know this?
1
Not at all
2
3
4
5
Perfectly
163
Q

Which of the following statements concerning duration are CORRECT?

I. Duration is used to compare potential price volatility between bonds with different maturities and coupons.

II. Duration can be used to select bonds to immunize an investor against interest rate and reinvestment rate risks.

III. Duration time-weights the present value of a bond’s cash flows.

IV. Like yield to maturity, duration reflects the timing of repayment of principal upon maturity, not the timing and magnitude of interest payments made in the interim.

A) II, III, and IV

B) I, II, and III

C) I, II, and IV

D) I, III, and IV

A

B) Explanation
Bonds with longer durations have greater volatility than bonds with shorter durations. The time horizon of a goal should be matched with a bond’s duration rather than with its maturity. Duration does take into account all interest and principal payments, not just the final payment at maturity.

LO 4.1.2

How well did you know this?
1
Not at all
2
3
4
5
Perfectly
164
Q

Assuming Victor has a bond portfolio with duration of 8.6 years, identify the CORRECT statement.

A) Victor’s bond portfolio is properly immunized to reach financial goals with a time horizon of 6 years.

B) Victor’s bond portfolio is immunized if its average time to maturity matches the time horizon of his current liabilities.

C) Immunizing a bond portfolio involves balancing reinvestment rate risk with default risk.

D) Victor’s bond portfolio may be composed of $2,000 worth of 3-year, zero-coupon bonds and $8,000 worth of 10-year duration bonds.

A

D) Explanation
Victor’s bond portfolio is properly immunized to reach financial goals or pay liabilities when the goals’ time horizon matches the portfolio’s duration (8.6 years). Portfolio immunization balances reinvestment rate risk with interest rate risk.

Because zero-coupon bonds’ durations are equal to their time to maturity and portfolio duration is a weighted average, Victor’s portfolio could consist of $2,000 worth of 3-year zero-coupon bonds and $8,000 worth of 10-year duration bonds, calculated as follows: (20% × 3 years) + (80% × 10 years) = 8.6 years.

LO 4.1.2

How well did you know this?
1
Not at all
2
3
4
5
Perfectly
165
Q

What is the duration of a zero-coupon bond yielding 9%, maturing in 10 years, and selling for $422.41?

A) 8 years

B) 7 years

C) 10 years

D) 9 years

Explanation
Because the bond is a zero-coupon bond, the duration must be 10 years.

LO 4.1.2

A

C) Explanation
Because the bond is a zero-coupon bond, the duration must be 10 years.

LO 4.1.2

How well did you know this?
1
Not at all
2
3
4
5
Perfectly
166
Q

Juliet owns a PRT Inc. bond with a par value of $1,000. PRT is a AA rated bond maturing in seven years. Juliet receives $55 of interest income from PRT semiannually. Comparable debt, i.e., AA rated, seven-year maturity, yields 12%. The bond’s duration is five years.

Assume the Fed is concerned about inflation and increases the discount rate. As a consequence, market interest rates on seven-year AA rated bonds change from 12% to 13%. How will the price of Juliet’s bond change?

A) The price will increase by approximately 7%.

B) The price will decrease by approximately 5%.

C) The price will increase by approximately 5%.

D) The price will decrease by approximately 7%.

A

Explanation

B) When interest rates change by 1%, the approximate price change of a bond will be the bond’s duration multiplied by the rate change. The bond’s duration can be seven years only if it is a zero-coupon bond. No information in the facts states that it is a zero-coupon bond. The facts do state that the bond has a coupon. Therefore, its duration will be less than seven years. The only other indicated possibility is a five-year duration. So, a five-year duration multiplied by an interest rate change of 1% results in a price change of 5%. Because rates rose, the price of the bond must decrease.

LO 4.1.2

How well did you know this?
1
Not at all
2
3
4
5
Perfectly
167
Q

Calculate the present value of a five-year bond with a coupon rate of 5.50% (paid semiannually) if similar quality bonds are currently yielding 4.35%.

A) $929.47
B) $1,026.97
C) $950.32
D) $1,051.18

A

Explanation

D) The answer is $1,051.18.

The present value of the bond is $1,051.18, calculated as follows: N = 10 (5 × 2); I/YR = 2.175 (4.35% ÷ 2); PMT = 27.50 (5.50% × 1,000 ÷ 2); FV = 1,000; solve for PV = 1,051.18, or $1,051.18.

LO 4.1.1

How well did you know this?
1
Not at all
2
3
4
5
Perfectly
168
Q

LJM Corporation has a bond issue with a coupon rate of 8% and seven years remaining until maturity. Assuming a par value of $1,000 and semiannual coupon payments, calculate the intrinsic value of the bond if current market conditions justify a 10% required rate of return.

A) $1,033.32
B) $901.01
C) $941.58
D) $920.81

A

B) The answer is $901.01.

The intrinsic value of LJM’s bond is $901.01, calculated as follows: N = 14 (7 × 2); I/YR = 5 (10% ÷ 2); PMT = 40 (8% × $1,000 ÷ 2); FV = 1,000; solve for PV = 901.01, or $901.01.

LO 4.1.1

How well did you know this?
1
Not at all
2
3
4
5
Perfectly
169
Q

Which of these choices correctly illustrates the relationship between a bond’s price and various yields?

A) For a premium bond: coupon rate > yield to maturity > current yield

B) For a discount bond: yield to maturity > coupon rate > current yield

C) For a discount bond: coupon rate < current yield < yield to maturity

D) For a premium bond: current yield > yield to maturity > coupon rate

A

Explanation

C) The answer is for a discount bond: coupon rate < current yield < yield to maturity.

When a bond trades at a discount, its yield to maturity will be greater than its current yield, which will be greater than its coupon rate.

LO 4.1.1

How well did you know this?
1
Not at all
2
3
4
5
Perfectly
170
Q

Which of these statements correctly analyze a bond’s coupon rate?

I. If a bond’s coupon rate is higher than the market yield, the bond will sell for less than its par value.

II. If the bond’s coupon rate is higher than the market yield, the bond’s current yield is higher than its yield to maturity (YTM).

III. If a bond’s coupon rate is higher than the market yield, the bond will sell at a premium.

IV. The only circumstance in which a bond will sell at par is if the bond’s coupon rate is equal to the market yield.

A) I, II, and IV
B) II and III
C) II, III, and IV
D) I and II

LO 4.1.1

A

C) The answer is II, III, and IV.

Only statement I is incorrect. If a bond’s coupon rate is greater than the market yield, the bond will sell for more (at a premium) than its par value.

How well did you know this?
1
Not at all
2
3
4
5
Perfectly
171
Q

Amber purchased a bond for $1,038.90 exactly two years ago. At that time, the bond had a maturity of five years and a coupon rate of 10% (paid semiannually). Assuming the rates below are the prevailing rates for this type of bond at different maturities, calculate the price that Amber could sell her bond for today.

Maturities 1 year. 3 years. 5 years. 10 years 30 years
Interest rates 6% 7% 8.5% 10% 12%

A) $1,060.08
B) $1,078.73
C) $1,038.31
D) $1,079.93

A

Explanation
D) The answer is $1,079.93. Using the 3-year rate of 7% for the calculation:

FV = 1,000

I/YR = 3.5 (7 ÷ 2)

PMT = 50 (100 ÷ 2)

N = 6 (3 × 2)

PV = 1,079.9283, or $1,079.93

LO 4.1.1

How well did you know this?
1
Not at all
2
3
4
5
Perfectly
172
Q

Choose the CORRECT statements regarding convexity relationships.

I. Convexity has a direct relationship with coupon rate.
II. Convexity has an inverse relationship with yield to maturity.
III. Convexity has an inverse relationship with term to maturity.
IV. Convexity has a direct relationship with term to maturity.

A) II and IV
B) II and III
C) I and III
D) I only

A

A) Explanation
The answer is II and IV.

Convexity has an inverse relationship with coupon rate and a direct relationship with term to maturity. The higher the coupon, the lower the convexity. The longer the maturity the greater the convexity.

LO 4.1.2

How well did you know this?
1
Not at all
2
3
4
5
Perfectly
173
Q

Calculate the estimated change in the price of a bond with a present value of $987.56 and Macaulay duration of 4.8 years when its YTM changes from 7% to 6%.

A) –4.53%
B) +4.53%
C) +4.49%
D) –4.49

A

C) Explanation
The answer is +4.49%.

Given the inverse relationship between bond prices and market interest rates, the price of the bond must increase by 4.49%, calculated as follows:
ΔP/P = –4.8 × [(0.06 – 0.07) ÷ (1 + 0.07)] = 0.0449, or 4.49%.

LO 4.1.2

How well did you know this?
1
Not at all
2
3
4
5
Perfectly
174
Q

Which of these statements best describes the concept of bond duration?

A) Duration measures the extent to which two variables move together, either positively (together) or negatively (opposite).

B) Duration assumes the price of a bond will stay constant because the market price of the underlying stock stays constant.

C) Duration is the average weighted time it takes the bondholder to receive the interest and principal payments from a bond in present value dollars.

D) Duration is used as an estimate of the change in the price of a bond given a negative one percentage point change in mortgage rates.

A

Explanation

C) The answer is duration is the average weighted time it takes the bondholder to receive the interest and principal payments from a bond in present value dollars. Covariance measures the extent to which two variables move together, either positively (together) or negatively (opposite). Duration is a measure of the sensitivity of a bond’s price to changes in interest rates.

LO 4.1.2

How well did you know this?
1
Not at all
2
3
4
5
Perfectly
175
Q

Yvette is saving for her son’s college education. Her son is expected to start college in 8 years. Choose the bond portfolio that would most likely be immunized with respect to this goal.

A) Weighted average time to maturity of bonds is 6 years with coupon of 8%.

B) Weighted average time to maturity of bonds is 10 years with coupon of 0%.

C) Weighted average time to maturity of bonds is 8 years with coupon of 3%.

D) Weighted average time to maturity of bonds is 10 years with coupon of 5%.

A

D). Explanation
The answer is weighted average time to maturity of bonds is 10 years with coupon of 5%. To immunize the portfolio, the duration of the portfolio should match the investor’s time horizon. Coupon-paying bonds have durations that are less than their time to maturity. Zero-coupon bonds have durations equal to their time to maturity.

LO 4.1.2

How well did you know this?
1
Not at all
2
3
4
5
Perfectly
176
Q

Assume a 3-year, $1,000 par value corporate bond is currently trading for $959.53. The bond has a coupon rate of 4% (paid once per year) and a yield to maturity of 5.50%. Calculate the duration for this bond.

A) 1.4418 years
B) 3.4680 years
C) 2.8835 years
D) 3.5871 years

A

B)
Explanation
The answer is 2.8835 years. The duration for this bond is 2.8835 years, calculated as follows:

Year Cash Flow(CF) (PV) of CF PV × Year
1 40.00 37.91 37.91
2 40.00 35.94 71.88
3 1,040.00 885.68 2,657.04
959.53 2,766.83

To solve for the PV of a given CF (example Year 1): FV = 40, N = 1, PMT = 0, 5.5 = I/YR, solve for PV. Divide the sum in the last column (2,766.83) by the total PV/market price of the bond (959.53) to derive the duration of 2.8835 years.

LO 4.1.2

How well did you know this?
1
Not at all
2
3
4
5
Perfectly
177
Q

Identify which of these items is generally listed as current assets on a business balance sheet.

I. Cash
II. Goodwill
III. Taxes payable
IV. Accounts receivable

A) I and IV
B) I and III
C) I, II, and III
D) I and II

A

Explanation

A) The answer is I and IV.

Cash and accounts receivable are current assets. Taxes payable is a liability and goodwill is generally considered an intangible asset.

LO 4.2.1

How well did you know this?
1
Not at all
2
3
4
5
Perfectly
178
Q

All of these statements explain the attributes of technical analysis except

A) technical analysts rely heavily on financial ratios in their analysis of stocks.
B) technical analysts attempt to predict the future movement of stock prices based on past trends.
C) technical analysts rely on charts to predict the future prices of stocks.
D) technical analysts use terms such as “trendline,” “support,” and “resistance” in analyzing stocks.

A

A)
Explanation
The answer is technical analysts rely heavily on financial ratios in their analysis of stocks. Analysts do not rely on financial ratios in their analysis of stocks. Instead, they rely on charts of past price history and volume to predict future price movements.

LO 4.2.1

How well did you know this?
1
Not at all
2
3
4
5
Perfectly
179
Q

Choose the form of the efficient market hypothesis that supports technical analysis.

A) Weak
B) Strong
C) None of these
D) Semistrong

A

Explanation

C) The answer is none of these. The efficient market hypothesis is in direct contradiction to technical analysis because the efficient market hypothesis is founded on the notion that all historical price and volume data, which is used by technical analysts, is already accounted for in the current stock price.

LO 4.2.1

How well did you know this?
1
Not at all
2
3
4
5
Perfectly
180
Q

Financial leverage affects

A) risk to stockholders.
B) return on equity.
C) return on equity, earnings per share, and risk to stockholders.
D) earnings per share.

A

Explanation
C)
The answer is return on equity, earnings per share, and risk to stockholders.

ROE and earnings per share are magnified with leverage, and the risk to stockholders increases as a firm’s leverage increases.

LO 4.2.2

How well did you know this?
1
Not at all
2
3
4
5
Perfectly
181
Q

JEM Technologies, Inc. has assets of $500 million and $50 million in liabilities. For the past year the company earned $125 million, and paid out $50 million in dividends. Calculate the company’s return on equity (ROE).

A) 52%
B) 20%
C) 38%
D) 28%

A

Explanation
D) The answer is 28%.
$500,000,000 – $50,000,000 = $450,000,000 in equity. $125,000,000 profit ÷ $450,000,000 equity = 0.2778, or 28% ROE.

LO 4.2.2

How well did you know this?
1
Not at all
2
3
4
5
Perfectly
182
Q

Assume a company’s net income steadily increases each year and it needs additional capital to grow. Which of these financial management actions may help increase the company’s return on common equity (ROE)?

I. issuing stock warrants
II. issuing long-term bonds
III. issuing additional shares of common stock
IV. refinancing 11% bonds with 7% bonds

A) I and II only
B) I and III only
C) I and IV only
D) II and IV only

A

Explanation

D) The answer is II and IV only.

To increase ROE, the company would want to issue debt instead of equity. Warrants are an option to purchase additional equity that would increase the denominator of the equation. The issuance of additional shares of common stock does the same. Debt in place of equity can increase ROE, as can the reduction of interest expense, which increases the numerator of the equation.

LO 4.2.2

How well did you know this?
1
Not at all
2
3
4
5
Perfectly
183
Q

Amelia is considering buying shares of HSO stock valued at $50 per share. She forecasts the stock to trade in excess of $75 per share over the next three years. During this time, she expects to receive annual dividends of $4.50 per share. Given a 10% required rate of return, calculate the intrinsic value of the stock.

A) $50.00
B) $45.00
C) $46.50
D) $29.50

A

B) Explanation
The answer is $45.00. The intrinsic value of the stock is $45, using the perpetuity dividend discount model, calculated as follows: $4.50 ÷ 0.10 = $45.

LO 4.3.1

How well did you know this?
1
Not at all
2
3
4
5
Perfectly
184
Q

LOK stock is currently paying an annual dividend of $2.15 per share, which is expected to grow at a constant rate of 2% annually. Calculate the price for the stock if an investor’s required rate of return is 10%.

A) $19.94
B) $27.41
C) $17.17
D) $21.50

A

B) The answer is $27.41.

Using the constant growth dividend discount model, the stock has an intrinsic value of $27.41, calculated as follows: [$2.15 × (1 + 0.02)] ÷ (0.10 – 0.02) = $2.1930 ÷ 0.08 = $27.41.

LO 4.3.1

How well did you know this?
1
Not at all
2
3
4
5
Perfectly
185
Q

LFM Corporation has an estimated free cash flow to equity (FCFE) of $2.50 per share in the current year. Moreover, its FCFE is expected to grow at a constant rate of 2% per year. Assuming an institutional investor has a required rate of return of 6.5%, calculate the intrinsic value of LFM stock.

A) $56.67
B) $40.76
C) $55.56
D) $133.13

A

A) Explanation
The answer is $56.67.

The formula for the discounted free cash-flow model:

V = FCFE1 ÷ (r – g) = ($2.50 × 1.02) ÷ (0.065 - 0.02) = 2.55 ÷ 0.045 = 56.6667, or $56.67

LO 4.3.1

How well did you know this?
1
Not at all
2
3
4
5
Perfectly
186
Q

FER stock has a current dividend of $0.75 per share that has been growing at a rate of 1.25% per year. If an investor’s required rate of return is 15% and the stock is currently selling for $6.34 per share, determine whether the investor should purchase the stock.

A) Yes, the stock is undervalued using the perpetuity dividend discount model.

B) No, the stock is not a wise purchase based on the risk-return trade-off.

C) Yes, the stock is undervalued based on the constant growth dividend discount model.

D) No, the stock is overvalued based on the constant growth dividend discount model.

A

D) Explanation

The answer is no, the stock is overvalued based on the constant growth dividend discount model.

Based on the constant growth dividend discount model, the intrinsic value of the stock is $5.52, calculated as follows: [0.75 × (1 + 0.0125)] ÷ (0.15 – 0.0125) = 0.7594 ÷ 0.1375 = 5.5227, or $5.52.

Because FER is currently trading at a price of $6.34 per share, it is overvalued, and the investor should not buy the stock.

LO 4.3.1

How well did you know this?
1
Not at all
2
3
4
5
Perfectly
187
Q

An analyst’s report on Derjet Industries has provided the following corporate information:

Total assets	$100,000,000
Total equity	$50,000,000
Net income	$12,500,000
Earnings per share	$3.50
P/E ratio	2.2
Dividend growth rate 1.65%

Assuming an investor has a required rate of return of 15%, calculate the maximum price that should be paid for this stock in the secondary market.

A) $14.38
B) $35.46
C) $7.70
D) $25.83

A

Explanation

C) The answer is $7.70.

Calculate the intrinsic value of the stock using EPS and the P/E ratio, which is $7.70 ($3.50 × 2.2).

LO 4.3.1

How well did you know this?
1
Not at all
2
3
4
5
Perfectly
188
Q

CPM Corporation has a constant dividend of $3.50 per share. The same rate is expected to be paid into the future. The risk-free rate of return is 3.5%. If an investor requires a 10% rate of return, calculate the present value of the company’s stock.

A) $25.92
B) $35.00
C) $53.85
D) $10.00

A

Explanation

B) The answer is $35.00.

The present (intrinsic) value of this investment is $35 ($3.50 ÷ 0.10) using the no-growth (perpetuity) dividend discount model.

LO 4.3.1

How well did you know this?
1
Not at all
2
3
4
5
Perfectly
189
Q

Marcy may add 100 shares of LKM corporation stock to her investment portfolio. The stock recently paid a dividend of $1.85 per share. The dividend is expected to grow at a constant rate of 2.25% per year. Her required rate of return is 7%. The stock is currently trading for $35.75 per share. Determine whether she should purchase the stock and why.

A) Yes, the stock is undervalued based on an intrinsic value of $33.46.

B) No, the stock is overvalued based on an intrinsic value of $38.95.

C) No, the stock is overvalued based on an intrinsic value of $32.87.

D) Yes, the stock is undervalued based on an intrinsic value of $39.82.

A

Explanation

D) The answer is yes, the stock is undervalued based on an intrinsic value of $39.82. Using the constant growth dividend discount model, the intrinsic value of the stock is $39.82.

V = ($1.85 × 1.0225) ÷ (0.07 – 0.0225)

V = 1.8916 ÷ 0.04750

V = 39.8232, or $39.82

Based on this value, the stock is undervalued relative to its price in the secondary market.

LO 4.3.1

How well did you know this?
1
Not at all
2
3
4
5
Perfectly
190
Q

Marvin is considering adding XYZ stock to his holdings. The stock has these characteristics:

Beta	1.45
Standard deviation	15.58%
Current dividend	$1.35
Required rate of return	8%
Risk-free rate of return	2%

The current dividend is expected to grow for three years at a rate of 2% and then 3% thereafter. Based on the information provided, calculate the intrinsic value of XYZ stock and determine if Marvin should add XYZ to his portfolio if it is currently trading at $24.50.

A) With an intrinsic value of $22.90, the stock is overvalued and should not be added to the portfolio.

B) With an intrinsic value of $27.13, the stock is undervalued and should be added to the portfolio.

C) With an intrinsic value of $29.60, the stock is undervalued and should be added to the portfolio.

D) With an intrinsic value of $21.60, the stock is overvalued and should not be added to the portfolio.

A

Explanation

B) The answer is with an intrinsic value of $27.13, the stock is undervalued and should be added to the portfolio.

The intrinsic value of the stock using the multistage growth dividend discount model is $27.13. Because the intrinsic value is greater than the fair market value, the stock is considered undervalued by the market and should be purchased for the portfolio.

Compute the value of each future dividend until the growth rate stabilizes (Years 1-3).

D1 = $1.35 × 1.02 = $1.38

D2 = $1.38 × 1.02 = $1.41

D3 = $1.41 × 1.02 = $1.44

Use the constant growth dividend discount model to compute the remaining intrinsic value of the stock at the beginning of the year when the dividend growth rate stabilizes (Year 4).

D4 = $1.44 × 1.03 = $1.48

V = $1.48 ÷ (0.08 – 0.03) = $29.60

Use the uneven cash flow method to solve for the net present (intrinsic) value of the stock.

CF0 = $0

CF1 = $1.38

CF2 = $1.41

CF3 = $1.44 + $29.60 = $31.04

I/YR = 8%

Solve for NPV = 27.1272, or $27.13

LO 4.3.1

191
Q

Caralla Foods produced before-tax earnings of $15 million last year. A large institutional investor has determined an appropriate capitalization for valuing this company is 8%. In addition, the risk-free rate of return is 5%. Based on this information, calculate the value of Caralla Foods using the discounted earnings model.

A) $500,000,000

B) $187,500,000

C) Not enough information is provided.

D) $300,000,000

A

B) The answer is $187,500,000.

The formula for the discounted earnings model:

V = E ÷ RD = 15,000,000 ÷ 0.08 = 187,500,000

Using the discounted earnings method produces a current corporate valuation of $187,500,000.

LO 4.3.1

192
Q

ABC Corporation has a P/E ratio of 5.00 and an expected growth rate in earnings for the next year of 9.5%. Assuming an investor’s required rate of return is 12%, calculate the firm’s PEG ratio.

A) 0.4167

B) 0.8333

C) 0.5263

D) 0.1667

A

C) Explanation
The answer is 0.5263.

Calculate the firm’s PEG ratio as follows: 5.00 ÷ (0.095 × 100) = 0.5263. After calculating this ratio, it then would be compared to ABC Corporation’s peers to determine whether a purchase is warranted.

LO 4.3.2

193
Q

MLM Corporation exhibits an expected growth in earnings of 11% for the next year. If an investor’s required rate of return is 14%, what is the firm’s price-to-free-cash-flow (P/FCF) ratio?

A) 38.00
B) 7.93
C) 37.00
D) 10.36

A

Explanation

C) The answer is 37.00.

The firm’s P/FCF ratio is 37.00, calculated as follows: (1 + 0.11) ÷ (0.14 – 0.11) = 37.00. After calculating this ratio, an investor would compare the result with the stock’s peers to determine if a purchase is warranted.

LO 4.3.2

194
Q

Al Gavin, age 37, wants to add to his common stock portfolio. He wants long-term capital appreciation and requires a 14% rate of return on stock investments. He is considering the purchase of one of these two stocks:

Stock 1

Dividends are currently $1.20 annually and are expected to increase 10% annually; market price = $38

Stock 2

Dividends are currently $1.00 annually and are expected to increase 11% annually; market price = $30

Which stock would be most appropriate for Al to purchase at this time, and why?

A) Stock 1, because the stock is undervalued

B) Stock 2, because the return on investment is greater than Al’s required rate of return

C) Stock 2, because the stock is overvalued

D)Stock 1, because the return on investment is greater than Al’s required rate of return

A

Explanation

The answer is Stock 2, because the return on investment is greater than Al’s required rate of return.

The intrinsic value of Stock 1 = $33 [($1.20 x 1.10) ÷ (0.14 - 0.10)]. Because $33 is less than $38, the stock is overvalued and would return less than his required return.

The intrinsic value of Stock 2 = $36.67 [($1.00 x 1.11) ÷ (0.14 - 0.11)]. Because $36.67 is more than $30, the stock is undervalued and would return more than his required return.

LO 4.3.2

195
Q

VUL stock has a current market price of $25.65 and sales per share of $1.67. Calculate the price-to-sales ratio for this stock.

A) 17.20
B) 22.77
C) 34.87
D) 15.36

A

Explanation

D) The answer is 15.36.

The formula for the price-to-sales (P/S) ratio:

P/S = market price per share ÷ sales per share P/S = $25.65 ÷ $1.67 = 15.3592, or 15.36

This ratio would then be compared to its industry peers to determine whether the stock appears to be overvalued or undervalued.

LO 4.3.2

196
Q

The use of P/E ratios to select stocks suggests which of these?

A) Low P/E ratio stocks are overvalued.
B) A stock should be purchased if it is selling near its historic high P/E.
C) High P/E stocks should be purchased.
D) A stock should be purchased if it is selling near its historic low P/E.

A

Explanation
D) The answer is a stock should be purchased if it is selling near its historic low P/E. While purchasing stocks near their historically high P/E ratio could continue to represent value, a much better time would be to purchase stocks that are at their historic low ratios.

LO 4.3.2

197
Q

Which of the following statements concerning option valuation is NOT correct?

A) If the price of the common stock exceeds the exercise price of the call, the call is said to be in the money and has an immediate exercisable value.

B) If the price of the common stock is less than the exercise price of the call, the call is said to be out of the money.

C) Calls that are at the money are those with exercise prices equal to the stock price.

D) Calls that are near the money are those with exercise prices slightly greater than the current market price.

LO 5.2.1

A

D) Explanation

Near the money is not a term concerning option valuation.

198
Q

You are about to recommend international mutual funds to your clients. Which of the following are characteristics of investing internationally?

I. International markets are less efficient than U.S. markets.

II. International mutual funds have the exchange rate risks of individual foreign stocks.

III. Due to lower correlations with U.S. stocks, foreign stocks can lower total portfolio risk.

IV. Investors in foreign securities avoid U.S. tax on realized capital gains.

A) I and II

B) I, II, and III

C) II, III, and IV

D) I and III

A

B) Explanation
Foreign markets have fewer analysts following stocks than do U.S. markets, and the information available from many foreign companies is minimal, making these markets less efficient than U.S. markets. The covariance of foreign stocks, especially small-cap and emerging market stocks, with U.S. stocks is relatively low, serving to lower the standard deviation of the portfolio in which such stocks are placed. Exchange rate risk is a systematic risk with foreign investments. Foreign income is taxed first in the country of origin and then again in the United States; U.S. taxpayers can take a foreign tax credit for taxes paid to other countries.

LO 5.5.1

199
Q

One of the characteristics of real estate investment trusts (REITs) is that they generally

A) offer new shares continually to investors.

B) have a high degree of marketability.

C) reinvest most of their income.

D) pay federal income tax on their earnings.

A

B) Explanation
Most real estate investments are not readily marketable. Therefore, an investor in real estate can generally expect some difficulty in converting a property to cash if cash is needed quickly. However, a REIT securitizes real estate properties, thereby allowing REIT investors to easily sell REIT shares in the open market. REITs must flow through at least 90% their income to investors; therefore the investors and not the REITs pay tax on these distributions.

LO 5.1.1

200
Q

Which of the following factors should be considered when investing in antiques?

I. Supply
II. Marketability
III. Inflation
IV. Dealer reputation

A) I, III, and IV

B) I, II, III and IV

C) I and II

D) III and IV

A

B) Explanation
The antique/collectibles markets are inefficient, where supply is frequently limited, making supply an important consideration. Some rare and famous antiques may be in great demand and be marketable; for many, however, no, or a limited, market and demand may exist. Rising inflation generally encourages investors to purchase tangibles. Dealer reputation is important in the collectibles market to minimize fraud.

LO 5.4.1

201
Q

Which of the following is CORRECT with regard to the purchasers of gold futures contracts?

A) They do not have to meet margin requirements.

B) They have less speculative positions.

C) They run the risk of government intervention altering the supply and demand for gold.

D) They are considered to be unleveraged positions.

A

C) Explanation
Government intervention has always been an issue for gold. The price was fixed for years and Americans were forbidden from owning gold. Although this has changed, the possibility of government intervention does exist. These positions are very speculative and are highly leveraged and risky.

LO 5.4.1

202
Q

Which of the following statements are true regarding risk and return for tangible investments?

I. Unsystematic risk tends to be high with most forms of tangible assets.

II. High cash flows from collectibles create annual taxable events.

III. The illiquid nature of the investments makes them suitable for income-seeking investors.

IV. The capital gains rate on collectibles is greater than that of other investments.

A) I only

B) I and IV

C) II and III

D) I and II

A

B) Explanation
Generally, collectibles do not provide an investor with predictable cash flows. Therefore, they would not be suitable for an income-seeking investor.

LO 5.4.1

203
Q

Which of the following are reasons an investor might buy a stock index call option instead of an individual stock call option?

A) It is the best way to be fully diversified against unsystematic risk.

B) The investor is more confident about the performance of an individual stock than the market as a whole.

C) The investor wants to hedge his existing stock portfolio against a market decline.

D) It reduces the level of systematic risk.

A

Explanation
A) Buying an index option means the risk associated with any one company (business risk) is avoided. An index option can be used to participate in a broad market advance, but still has systematic risk. One would buy a put to hedge against a market decline.

LO 5.2.2

204
Q

A client with a large, well-diversified common stock portfolio expresses concern about a possible market decline. However, she does not want to incur the cost of selling a portion of her holdings nor the risk of mistiming the market. A possible strategy for her would be

A) sell an index put option.

B) buy an index call option.

C) sell an index call option.

D) buy an index put option.

A

D) Explanation
From the statement of facts, the client wishes to continue holding the current portfolio but wants to hedge. Because the client is long stocks, the appropriate hedge is a short position, which is obtained by purchasing a put option. Because the portfolio is well diversified, an index put option will be an excellent choice. Selling an index put option provides more limited hedging than does buying a put option, as the loss is shielding only to the extent of the income per share received by the option seller.

LO 5.2.2

205
Q

George owns 1,000 shares of XYZ stock. Based on recent analyst projections and George’s own research, he believes XYZ’s price will remain flat over the next few months. Accordingly, which strategy would George likely employ if one of his objectives was to increase his portfolio’s income stream?

A) Sell 10 XYZ call option contracts

B) Buy an index future

C) Buy 10 XYZ put option contracts

D) Buy a warrant

A

A) Explanation
This is known as the covered call strategy and is used to increase the income of the portfolio when the price of the underlying stock is expected to remain flat for a period of time. Assuming the price of the underlying stock continues to remain flat, there is little chance of the option being exercised and the stock being called away.

LO 5.2.2

206
Q

All of the following statements concerning the mechanics of futures trading are CORRECT except

A) the open interest increases when an investor goes long a contract and is reduced when the contract is liquidated.

B) all futures contracts are marked to market daily; that is, all profits and losses are credited and debited daily to each investor’s account.

C) an open interest indicates futures contracts that are not offset by opposite transactions or delivery, and measures the number of unliquidated contracts at any point in time, on a cumulative basis.

D) a long hedge is a transaction involving the sale of futures (a short position) while holding the asset (a long position); whereas, a short hedge is a transaction where the asset is currently not held but futures are purchased to lock in current prices.

A

D) Explanation
A short hedge is a transaction involving the sale of futures (a short position) while holding the asset (a long position); whereas, a long hedge is a transaction where the asset is not currently held but futures are purchased to lock in current prices.

LO 5.3.1

207
Q

What is the intrinsic value of a put option with an exercise price of $45, when the stock is selling for $40?

A) $50
B) $1
C) $4
D) $5

A

D) Explanation
intrinsic value of a put option = (exercise price − market price) = ($45 − $40) = $5

LO 5.2.1

208
Q

A collateralized mortgage obligation (CMO) differs from other mortgage-backed securities in that the cash flows associated with the CMO’s pool of underlying mortgages are divided into repayment periods known as

A) conversion privileges.

B) pre-retirement provisions.

C) guaranteed remittance provisions.

D) tranches.

A

D) Explanation
A CMO differs from mortgage-backed securities in that the cash flows associated with the CMO’s pool of underlying mortgages are divided into repayment periods known as tranches.

LO 5.1.1

209
Q

The Black/Scholes option valuation model specifies variables that affect the value of the option. Which of the following is CORRECT with regard to how the model measures the value of an option?

A) All of these are correct.

B) Increased volatility of the common stock will increase the value of a call option.

C) An increase in interest rates increases the value of a call option.

D) The value of an option declines as it approaches expiration.

A
A) Explanation
As risk (volatility) increases so does the value of an option. As an option approaches expiration there is less time for the option to be profitable, so the value decreases. An increase in rates lowers the present value of the funds needed to exercise, which in effect lowers the present value of the strike price, so the value of the option increases.

LO 5.2.1

210
Q

A sophisticated investor who is broadly diversified in different asset classes is now considering collectibles to further reduce his risk exposure to the stock market. Which of the following factors should she be aware of with respect to the market for collectibles?

I The market for collectibles is efficient.
II The market for collectibles is inefficient.
III The market for collectibles is subject to special risks not normally associated with financial assets.
IV The market for collectibles is more positively correlated with inflation than are financial assets.

A) II and IV

B) I and III

C) II, III, and IV

D) II and III

A

C) Explanation
The collectibles market is inefficient, with large spreads between bid and ask prices and untimely information on trades. Collectibles must be stored and insured, and their prices normally rise and fall with inflation to a greater extent than do financial asset prices.

LO 5.4.1

211
Q

Which of the following is NOT an advantage of purchasing American Depository Receipts (ADRs)?

A) Transactions are done on an organized exchange in the United States.

B) They allow U.S. investors to buy foreign country stock denominated in dollars.

C) They eliminate exchange rate risk.

D) Foreign taxes withheld can be claimed as a credit to offset income taxes on dividends received.

A

C) Explanation
ADRs are priced in U.S. dollars and therefore have exchange rate risk.

LO 5.5.1

212
Q

On January 1, 20XX, one U.S. dollar could buy 105 Japanese yen; on December 31, 20XX, one U.S. dollar could buy 121 Japanese yen. Which of the following statements best illustrates what has happened to the U.S. dollar during year 20XX?

A) The U.S. dollar was hedged.

B) The U.S. dollar was revalued.

C) The U.S. dollar was above parity.

D) The U.S. dollar was devalued.

A

B) Explanation
A revaluation occurs to a U.S. investor when the U.S. dollar can buy more of a particular foreign currency today than it could buy last year. A devaluation occurs when the U.S. dollar can buy less of a particular foreign currency today than it could buy last year.

LO 5.5.2

213
Q

Your client is considering the purchase of an apartment complex with the following anticipated financial characteristics:

Mortgage = 8% for 30 years, amortized monthly, with financing available for 70% of value
Cost recovery period = 27.5 years (residential rental)
Potential gross income (PGI) (year 1) = $1,500,000
Vacancy rate = 8% of PGI
Operating expenses = 30% of PGI
Capitalization rate = 9%
Based on this information, what is the maximum price you would advise your client to pay?

A) $16,666,667

B) $10,333,333

C) $11,666,667

D) $9,400,000

A

B) Explanation
The calculation is as follows: Potential gross income (PGI) of $1,500,000 less vacancy losses of $120,000 equals effective gross income of $1,380,000. Subtract operating expenses of $450,000, resulting in net operating income (NOI) of $930,000. Value = $930,000 ÷ 0.09 = $10,333,333.

LO 5.1.1

214
Q

Which of the following statements concerning derivatives is CORRECT?

I. The value of a derivative security is derived from its underlying security.

II. Similar to equity instruments and fixed-income securities, derivatives represent either an ownership interest or a credit interest in a corporation.

III. An option is the right to buy or sell stock at a specified price within a specified period of time.

IV. An option’s intrinsic value is the worth of the underlying asset.

A) I, III, and IV

B) II only

C) I and II

D) I and III

A

D)

Derivatives give the holder certain rights and options under certain specific conditions. They represent neither an equity (ownership) nor a debt (credit) interest in a corporation.

LO 5.2.1

215
Q

You are advising a high-net-worth individual with large, diversified holdings across many asset classes on the potential use of futures contracts as a means of risk reduction. The client has never invested in futures contracts before and has always considered them risky. In explaining the characteristics of futures, you could tell him all of the following except

A) stock index futures differ from other futures contracts in that they settle in cash rather than in the stocks themselves.

B) an investor who expects interest rates to rise should sell Treasury bond futures.

C) a futures contract is constructed to suit the individual needs of two parties, whereas a forward contract has a standardized delivery date and unit amount.

D) a farmer who is to harvest a wheat crop in two months should hedge by selling two-month wheat futures contracts.

A

C) Explanation
Futures contracts are standardized contracts for delivery, whereas a forward contract is custom made.

LO 5.3.1

216
Q

Which of these statements describing real estate limited partnerships (RELPs) is CORRECT?

A) Interests in RELPs are generally traded in the secondary market.

B) An individual may be the sole owner of a RELP.

C) Income attributed to limited partners is considered passive income for income tax purposes.

D) Limited partners have limited management responsibilities.

A

C) Explanation
The answer is income attributed to limited partners is considered passive income for income tax purposes. Real estate limited partnerships (RELPs) are composed of at least two partners. Limited partners do not have any management responsibilities, and income attributed to them is treated as passive income by the IRS. RELPs are not publicly traded.

LO 5.1.1

217
Q

Identify which of these statements regarding real estate limited partnerships (RELPs) and real estate investment trusts (REITs) is CORRECT.

I. REITs are generally liquid and actively traded on national and regional stock exchanges, as well as over the counter.

II. A REIT shareholder may sell as few or as many shares as he or she deems appropriate, whereas a RELP investor must generally sell the entire interest.

III. A REIT (as a corporation) is managed by a board of directors, whereas a RELP is managed by a general partner.

IV. A RELP (as a limited partnership) suffers from extreme liquidity and marketability risk.

A) I and III
B) II, III, and IV
C) I, II, III, and IV
D) I only

A

Explanation

C) The answer is I, II, III, and IV. All of these statements are correct. A popular form of indirect ownership of real estate is to invest as a limited partner, usually through a non-publicly traded real estate limited partnership (RELP) or syndication. A real estate investment trust (REIT) serves as a source of long-term financing for real estate projects by investing in real estate, short-term construction loans, and mortgages.

LO 5.1.1

218
Q

Choose the REITs that are used to finance real estate ventures that develop property or finance construction.

A) Equity REITs
B) Mortgage REITs
C) Government REITs
D) Hybrid REITs

A

Explanation

B) The answer is mortgage REITs. Mortgage REITs are in the business of financing real estate ventures. They make loans to develop property or finance construction.

LO 5.1.1

219
Q

A collateralized mortgage obligation (CMO) differs from other mortgage-backed securities in that the cash flows associated with the CMO’s pool of underlying mortgages are divided into repayment periods structured as

A) guaranteed remittance provisions.
B) conversion privileges.
C) pre-retirement provisions.
D) tranches.

A

Explanation

D) The answer is tranches. A CMO differs from mortgage-backed securities in that the cash flows associated with the CMO’s pool of underlying mortgages are divided into repayment periods structured as tranches.

LO 5.1.1

220
Q

Prairie View Apartments is an 80-unit apartment complex with 60 one-bedroom units and 20 two-bedroom apartments. One-bedroom apartments rent for $500 and two-bedroom for $600 per month. The complex also earns $15,000 per year from renting the community hall for parties and washing machines. No major repairs are anticipated in the next seven years. This data is also available:

Vacancy and collection losses: 8% of potential gross income (PGI) Property insurance and taxes: $44,000
Management and accounting: $18,600 Repairs and maintenance: $19,800 Utilities including garbage: $31,000
Mortgage payment (20-year 9% fixed rate): $270,000
The property’s value is estimated at $4,000,000, including $400,000 for the land and $3,600,000 for the building and improvements. The property has an outstanding mortgage of $3,000,000. The building and improvements are being depreciated using the straight-line depreciation for an annual depreciation of $131,000. If the capitalization rate for similar properties is estimated to be 9%, calculate the value of Prairie View Apartments.

A) $3,340,275
B) $4,056,444
C) $4,045,333
D) $4,506,667

A

Explanation

D) The answer is $4,045,333.

First, calculate the net operating income for the Prairie View Apartments as follows:

Gross rents (60 X $500 X 12) + (20 X $600 X 12) $504,000
Other income +15,000
Potential gross income (PGI) 519,000
Vacancy losses (8% of PGI) −41,520
Effective gross income 477,480
Operating expenses ($44,000 + 18,600 + 19,800 + 31,000) −113,400
Net operating income (NOI) $364,080
Value = NOI ÷ capitalization rate = $364,080 ÷ 0.09 = $4,045,333

LO 5.1.1

221
Q

Reasons for investing in real estate include all of these except

A) potential long-term appreciation.
B) liquidity.
C) potential tax shelter.
D) relatively constant cash flow.

A

Explanation

B) The answer is liquidity. Real estate is an illiquid investment. Real estate offers the ability to shelter an investment’s cash flow. Income real estate provides a relatively dependable source of cash flow. Real estate offers investors an excellent source of long-term capital appreciation.

LO 5.1.1

222
Q

Identify the INCORRECT statement regarding options.

A) Option contracts are typically issued with expiration dates no longer than nine months.

B) A put option gives the buyer the right to sell the underlying security for a specified price within a specified period.

C) The buyer of an option contract pays a premium for the right to exercise the option.

D) An investor who is bullish is likely to write a call.

A

Explanation

D) The answer is an investor who is bullish is likely to write a call. An investor who is bullish is likely to either purchase a call or write a put. An investor will write a call if he or she believes market prices will decline.

LO 5.2.1

223
Q

Identify which of these statements regarding option pricing models is CORRECT.

I. The binomial option pricing model assumes that the price of the option will change constantly because the market price of the underlying security also changes constantly.

II. The Black-Scholes option valuation model is designed to determine the price of an American call option.

III. The Black-Scholes option valuation model assumes that the price of the option will change in discrete increments on the basis of movements (up or down) in the price of the underlying stock.

IV. The binomial model assumes the call option being valued has an exercise price of $100.

A) II, III, and IV
B) I, II, and III
C) I and IV
D) IV only

A

Explanation
D) The answer is IV only.

The Black-Scholes option valuation model assumes that the price of the option will change constantly because the market price of the underlying security also changes constantly and is designed to determine the price of a European call option. The binomial option pricing model assumes that the price of the option will change in discrete increments on the basis of movements (up or down) in the price of the underlying stock.

LO 5.2.1

224
Q

All of these statements correctly explain option contracts except

A) the maximum gain to the purchaser of a put option is the amount of premium received from the writer.

B) an option is in-the-money when its intrinsic value is positive.

C) a call option is naked when it is written on a security not owned by the writer.

D) if a call option expires, the writer will report the premium received as a short-term capital gain.

A

Explanation

A) The answer is the maximum gain to the purchaser of a put option is the amount of premium received from the writer. The maximum gain to the purchaser of a put option is the exercise price less the amount of premium paid to the writer. The maximum loss to the purchaser of a put option is the amount of premium paid to the writer.

LO 5.2.1

225
Q

Which of the following statements regarding puts and calls is CORRECT?

I. If the investor is bullish (i.e., thinks that market prices will increase), he will either write a call or purchase a put.

II. If the investor is bearish (i.e., thinks that market prices will decline), he will either purchase a call or write a put.

A) Both I and II
B) II only
C) Neither I nor II
D) I only

A

Explanation

C) The answer is neither I nor II. A bullish investor will either purchase a call or write a put. A bearish investor will either write a call or purchase a put.

LO 5.2.1

226
Q

Mary owns a put option with an exercise price of $20 per share. The option is currently trading for $0.53 and the underlying stock is currently trading for $19.67 per share in the secondary market. Based on this information, select the INCORRECT statement.

A) The option is out-of-the-money.
B) The option has a time value of $0.20.
C) The put option allows Mary to sell the underlying stock for a specified price within a specified period.
D) The option has an intrinsic value of $0.33.

A

A) Explanation
The answer is the option is out-of-the-money. The option is in-the-money because the option has a positive intrinsic value. The option has an intrinsic value of $0.33 ($20.00 – $19.67) and a time value of $0.20 ($0.53 – $0.33).

LO 5.2.1

227
Q

John sells a naked call option for a $3 premium. The call option has an exercise price of $20. Which of these statements is CORRECT?

A) John’s loss is limited to $20.
B) John’s maximum gain is $3.
C) The call will likely not be exercised while the stock is trading at $23.
D) The option will not be exercised until the market price is $17.

A

B) Explanation
The answer is John’s maximum gain is $3. John’s maximum loss is unlimited. Because the call option was written as a naked call option, the seller (John) will have to buy the stock at the prevailing market price if the call option is exercised by the buyer. The buyer would exercise the call option when the stock price exceeds $20. Any price level over $20 begins to reduce the call option buyer’s potential loss, and at $23 the seller and buyer break even.

LO 5.2.1

228
Q

All of these positions are used to create a zero-cost collar except

A) purchasing a put option on the stock.
B) purchasing a call option on the stock.
C) writing a call option on the stock.
D) a long position in the stock.

A

Explanation

B) The answer is purchasing a call option on the stock.

To create a zero-cost collar, the investor will not purchase a call option on the stock. Rather, the investor will use the premium received from writing the call option to subsequently purchase the put option on the underlying stock, thereby creating a cashless collar and protection from downside risk.

LO 5.2.2

229
Q

Due to an inheritance, Danielle now owns a large position in XYZ stock. She is concerned that the stock may decline in the upcoming months while she is deciding what to do with the investment. What type of investment strategy could her financial planner propose to protect the stock from substantial downside risk?

A) Write a call option
B) Zero-cost collar
C) Purchase a put option
D) Purchase an index future

A

Explanation

C) The answer is purchase a put option.

Danielle should purchase a put option (referred to as protective put or portfolio insurance) to protect herself from possible loss. Thereby, the portfolio is protected and the investor’s loss on the option is limited to the amount of the premium paid for the put.

LO 5.2.2

230
Q

Which of these strategies is considered a protective strategy?

A) Purchasing a call option on a stock already owned
B) Purchasing a hedge fund
C) Buying a put option on a stock already owned
D) Buying stock index futures

A

Explanation

C) The answer is buying a put option on a stock already owned.

A protective strategy limits downside risk. The purchase of a put option on a stock that is already owned reduces downside risk. If the price of a stock declines, the value of the put option increases and protects an investor holding a long position in the underlying stock.

LO 5.2.2

231
Q

An investor who creates a long straddle most likely expects the underlying security to

A) increase in volatility.
B) remain near its current price.
C) decrease in volatility.
D) increase in price.

A

A) Explanation
The answer is increase in volatility.

Long straddle profits if the volatility of the underlying security increases more than is reflected in current option prices, which would increase the value of both the call option and the put option. Either a significant increase or a significant decrease in the underlying price would benefit the holder of a long straddle. If the investor was expecting only an increase in price, long call would be a better choice of investment (as the premium paid on the long put would be a waste).

LO 5.2.2

232
Q

Select the CORRECT statements regarding the taxation of futures contracts.

I. Gains and losses on futures contracts are considered capital gains or losses regardless of the tax nature of the underlying asset.

II. At the end of the year, net gains or losses on futures contracts are treated as 60% short-term and 40% long-term.

A) Neither I nor II
B) Both I and II
C) II only
D) I only

A

Explanation

D) The answer is I only. Statement II is incorrect. At the end of the year, net gains on futures contracts are treated as 60% long-term and 40% short-term.

LO 5.3.1

233
Q

Which of these best describes a futures contract?

A) A futures contract is a derivative characterized by low risk and the potential for high return.

B) A futures contract is a self-liquidating, flow-through entity that invests exclusively in mortgage-backed securities.

C) A futures contract seeks to invest in an industry sector or index that is undervalued, but is expected to rise in value in the future.

D) A futures contract is an agreement between two parties to make or take delivery of a specified amount of a commodity or financial asset at a future time, place, and unit price.

A

Explanation
D) The answer is a futures contract is an agreement between two parties to make or take delivery of a specified amount of a commodity or financial asset at a future time, place, and unit price. To complete the futures contract, delivery of the commodity or asset may be made, but more often, the buyer (or holder) simply purchases an offsetting contract and cancels the original position.

LO 5.3.1

234
Q

Jim is a paper maker who purchases lumber from tree farmers around his state. Which of these hedge positions should Jim consider if he is concerned with rising lumber prices?

A) A long hedge; Jim should sell lumber futures contracts to protect against rising lumber prices.

B) A short hedge; Jim should sell lumber futures contracts to protect against rising lumber prices.

C) A long hedge; Jim should buy lumber futures contracts to protect against rising lumber prices.

D) A short hedge; Jim should buy lumber futures contracts to protect against rising lumber prices.

A

Explanation

C) The answer is a long hedge; Jim should buy lumber futures contracts to protect against rising lumber prices. A long hedge uses a long futures position to hedge a short position (purchasing lumber).

LO 5.3.1

235
Q

Open interest is

A) the number of unexecuted buy and sell orders for a particular commodity pending at the end of trading each day on a commodity exchange.

B) the number of futures contracts outstanding for a commodity on any given trading day.

C) a technical indicator, used only with financial futures, that indicates the dollar amount of bonds expected to be delivered at contract expiration.

D) the rate of interest charged on the difference between the market value of a commodity contract and the amount of margin on deposit in a customer’s account.

A

Explanation

B) The answer is the number of futures contracts outstanding for a commodity on any given trading day. Open interest is reported daily in the financial press. Traders like to be in the most active months of a contract for liquidity purposes.

LO 5.3.1

236
Q

Why would financial planners recommend their clients purchase natural resources for their investment portfolios?

I. Diversification
II. Low degree of risk
III. Positive correlation with other financial assets
IV. Pass through of certain tax benefits including depletion

A) II and III
B) I and IV
C) I, II, III, and IV
D) II, III, and IV

A

Explanation

B) The answer is I and IV. Statements II and III are incorrect. Investments in natural resources offer investors a high degree of risk and a negative correlation with other financial assets.

LO 5.4.1

237
Q

An investor who is interested in developing a portfolio of collectibles should probably do which one of these?

A) Avoid buying through knowledgeable dealers
B) Diversify over a variety of types of collectibles
C) Specialize in a type of collectibles
D) Buy relatively inexpensive collectibles

A

Explanation
C) The answer is specialize in a type of collectibles. Specializing in a type of collectible would give an investor a better knowledge of that collectible and its market direction and worth.

LO 5.4.1

238
Q

Interest in gold as an investment increases during periods of which of these?

A) Recession
B) Deflation
C) Economic growth
D) Economic or political uncertainty

A

Explanation

D) The answer is economic or political uncertainty. Economic or political uncertainty causes the demand for gold to grow.

LO 5.4.1

239
Q

All of these statements correctly describe international investing except

A) one method to engage in international investing is through American depositary receipts.

B) an emerging market is a market in a highly developed foreign economy with stable political and social institutions.

C) international investing offers diversification and potentially higher returns.

D) an international investor faces the additional risks of foreign currency risk and country risk.

A

Explanation

B) The answer is an emerging market is a market in a highly developed foreign economy with stable political and social institutions. Emerging markets are markets in lesser developed countries.

LO 5.5.1

240
Q

Jay has recently learned that foreign investments are a good way to diversify a portfolio. Select which of these statements regarding the risks and benefits of foreign investments, specifically American depositary receipts (ADRs), is CORRECT.

A) ADRs are free of exchange rate risk but are subject to market risk.

B) Because ADRs are only available through private placements, they are relatively illiquid and nonmarketable investments.

C) Foreign taxes paid on income earned from an ADR are eligible for the foreign tax credit.

D) ADRs are receipts for investments placed with foreign mutual funds.

A

C) Explanation
The answer is foreign taxes paid on income earned from an ADR are eligible for the foreign tax credit.

Although ADRs are dollar denominated, they are still subject to exchange rate risk because the dividends are declared in the local currency and converted to U.S. dollars. ADRs are receipts for shares of a foreign company, not shares of a mutual fund. Because ADRs are traded in the secondary market, they are relatively liquid and marketable investments. Foreign taxes paid are eligible for the foreign income tax credit.

LO 5.5.1

241
Q

Identify the CORRECT statement concerning international investing.

A) The addition of foreign securities to a portfolio may result in increased portfolio risk due to the different movements of foreign markets and U.S. markets.

B) The rates of return on foreign securities have always been less than those available from U.S. markets.

C) Information is not as readily available on foreign investments.

D) Foreign markets are usually mature and offer no growth advantages.

A

Explanation

C) The answer is information is not as readily available on foreign investments. Foreign markets offer economies of scale and growth opportunities. Investors may earn higher returns in foreign markets, particularly if they are less efficient than U.S. markets. Including foreign securities in an investment portfolio may lower risk through greater diversification.

LO 5.5.1

242
Q

Mary is traveling to Europe and has $1,000 (U.S.) that she wants to convert to euros. The current exchange rate is 0.90 euros = $1 (U.S.). How many euros will she receive in this exchange?

A) 1,800
B) 1,111
C) 900
D) 2,000

A

Explanation
The answer is 900. Determine the cost in U.S. dollars to purchase one euro. Using the HP 10bII+:

0.90, SHIFT, 1/x = $1.1111

Then,

$1,000 ÷ $1.1111 = 900 euros

or

$1,000 × 0.90 = 900 euros

LO 5.5.2

243
Q

Which of these are characteristics of foreign currency exchange?

I. When a strong foreign currency is converted into U.S. dollars, more dollars are received than if the foreign currency had stayed stable or declined.

II. An increase in the supply of a currency results in its devaluation.

III. A U.S. investor in foreign assets would want the U.S. dollar to strengthen against foreign currencies after the assets are purchased.

IV. When the U.S. dollar weakens against a foreign currency, the total return increases to a U.S. investor holding stocks denominated in that currency.

A) II, III, and IV
B) I and III
C) I, II, and IV
D) II and IV

A

Explanation

C) The answer is I, II, and IV.
When an investor has money invested in a foreign stock, the investor should want the dollar to decline relative to the foreign currency so that a currency gain occurs in addition to the security gain. Currencies are subject to the same supply/demand rules that apply to goods and services and to the money supply. Increasing supply results in decreasing price of the currency.

LO 5.5.2

244
Q

A Japanese bank has decided to use some of its U.S. dollar reserves, resulting from the U.S. merchandise trade deficit with Japan, to invest in U.S. Treasury bonds. The U.S. Treasury securities pay approximately 6% interest, compared to 2% interest paid on Japanese bonds. For the Japanese bank to retain at least this differential in interest income, which of these situations in the foreign exchange market would have to occur?

A) The yen would have to depreciate or remain steady relative to the dollar during the holding period of the bond.

B) The yen would have to appreciate relative to the dollar during the holding period of the bond.

C) The yen would have to appreciate relative to the dollar and interest rates in the United States would have to fall.

D) The yen would have to depreciate relative to the dollar and interest rates in the United States would have to rise.

A

Explanation

A) The answer is the yen would have to depreciate or remain steady relative to the dollar during the holding period of the bond.

When an investor in Country A invests in a security in Country B, the investor benefits if Country A’s currency depreciates relative to Country B (or Country B’s currency appreciates relative to Country A). Because rising interest rates cause bond prices to decline, the best possible scenario would be for the yen to depreciate and for U.S. interest rates to fall. This would give both a capital gain and a currency gain.

LO 5.5.2

245
Q

Frank’s current portfolio has an expected return of 10% and a standard deviation of 3%. Asset A has an expected return of 17%, a standard deviation of 6%, a beta of 1.3, and a correlation of –1.0 with Frank’s portfolio. Asset B has an expected return of 18%, a standard deviation of 2%, a beta of 1.0, and a correlation of 0.03 with Frank’s portfolio. Asset A’s correlation with the S&P 500 is –0.72, and Asset B’s correlation with the S&P 500 is 0.98. The S&P 500 is expected to increase 5% in the upcoming year.

Based on the information provided, choose the CORRECT statement.

A) Asset B is expected to increase 10% this year.

B) Frank’s portfolio is highly correlated with the S&P 500.

C) Beta is an appropriate measure of risk for Asset A.

D) Approximately 50% of Asset A’s variability of returns is explained by changes in the S&P 500.

A

D) Explanation
Asset B is highly correlated with the S&P 500, and its beta is 1.0, indicating an expected increase similar to that of the S&P 500. Asset B has a correlation with the S&P 500 of 0.98 and a correlation with Frank’s portfolio of only 0.03. Thus, Frank’s portfolio is not highly correlated with the S&P 500. Asset A’s coefficient of determination with respect to the S&P 500 is –0.72 × –0.72 = 0.52, or 52%, indicating that beta is a poor measure of risk for Asset A.

LO 6.2.5

246
Q

Which of the following is the risk that disappears in the portfolio construction process?

A) Purchasing power risk

B) Interest rate risk

C) Unsystematic risk

D) Systematic risk

A
C)
Unsystematic risk (diversifiable risk) is the risk that is eliminated when the investor builds a well-diversified portfolio.

LO 6.1.1

247
Q

All of the following statements correctly explain investment risk except

A) systematic risk may be reduced or eliminated by effective portfolio diversification.

B) a stock’s level of risk is a combination of market risk and diversifiable risk.

C) investors expect to earn a higher rate of return for assuming a higher level of risk.

D) the beta coefficient measures an individual stock’s relative volatility to the market.

A

A) Explanation
Unsystematic (diversifiable) risk may be effectively managed through portfolio diversification.

LO 6.1.1

248
Q

All of the following statements concerning risk and return are CORRECT except

A) all other things being equal, security prices increase as interest rates increase.

B) return on a typical investment consists of two components: yield and capital gain (loss).

C) interest rate risk is the variability of a security’s returns resulting from changes in interest rates.

D) capital gain (loss) is the change in price of a security over some period of time.

A

Explanation
A) All other things being equal, security prices move inversely to interest rate changes.

LO 6.1.1

249
Q

Taylor, a personal friend of yours, has been a practicing veterinarian for eight years. She is 35 years old and has a 3-year-old daughter. Taylor has a moderate risk tolerance, wants to save for retirement, and is considering increasing her investment in the following mutual fund. Taylor has asked you for your recommendation.

Risk-free return 7.0%

Return of market 12.5%

Growth and Income Fund
NAV (beginning of year)	$53.00
NAV (end of year)	$52.75
Dividend	$3.25
Capital gains distributed	$2.75
Beta	0.70
Realized return	10.85%

*Paid at time of next investment

Which of the following is CORRECT regarding the risk and return of the fund?

A) The fund has greater risk and less return than the market.

B) The fund has less risk and greater return than the market.

C) The fund has less risk and less return than the market.

D) The fund has equal risk and greater return than the market.

A

C) Explanation
A beta of 1 represents the risk of the market. A beta of less than 1 represents risk less than the market’s risk, and a beta of greater than 1 represents risk greater than that of the market.

LO 6.2.1

250
Q

You are considering buying a stock that has a mean return of 14% and a standard deviation of 20. You can expect the return to fall within what range 95% of the time?

A) –0.06 to 0.34

B) –0.26 to 0.54

C) Cannot be determined from the information given

D) –0.46 to 0.74

A

B) Explanation
A stock with a standard deviation of 20 will deviate from the mean by one standard deviation 68% of the time, two standard deviations 95% of the time, and three standard deviations 99% of the time. So for this stock, plus or minus 40 from the mean of 14% would be –26% and +54%.

LO 6.2.2

251
Q

Stock XYZ has an average return of 18% with a standard deviation of 21. Within what range could an investor expect a return to fall 68% of the time?

A) 0% to 21%

B) –3% to 18%

C) –3% to 39%

D) 3% to 39%

A

C)Explanation
By definition, an investment’s return will be within one standard deviation of the mean return 68% of the time. The mean return of 18% plus or minus one standard deviation is 18% – 21% (–3%) and 18% + 21% (39%).

LO 6.2.2

252
Q

Which of the following statements concerning a knowledge of the risk/return relationship is CORRECT?

I. Future risk/return relationships are not guaranteed to match past risk/return relationships.

II. Chances are that past relative relationships will not continue into the future.

III. A reduction in risk also means a reduction in the possible return on the investment.

IV. The smaller the dispersion of returns, the greater the risk associated with a particular investment.

A) I only

B) II, III, and IV

C) II and III

D) I and III

A

D)Explanation
Chances are that past relative relationships will continue into the future. The smaller the dispersion of returns, the lower the risk associated with a particular investment.

LO 6.1.1

253
Q

Mutual fund I has a standard deviation of 4% and an expected return of 10%. Mutual fund J has a standard deviation of 8% and an expected return of 13%. If I and J have a correlation coefficient of –1.0, which of the following statements is CORRECT?

A) J is less risky than I on a risk-adjusted basis.

B) I and J are perfectly negatively correlated.

C) A portfolio combining funds I and J may have an expected return less than 10%.

D) There is no combination of I and J such that the portfolio’s standard deviation is zero.

A

B)Explanation
J’s coefficient of variation is 8% ÷ 13% = 0.615. I’s coefficient of variation = 4% ÷ 10% = 0.40. I is less risky, on a risk-adjusted basis, than J. Because I and J are perfectly negatively correlated (correlation coefficient of –1.0), there exists a combination of I and J such that the standard deviation is zero. The expected return of a portfolio is the weighted average, which cannot be less than the lowest expected return of the portfolio components.

LO 6.2.4

254
Q

The Mountain Fund has a standard deviation of 22, a mean return of 15%, and a correlation coefficient with the S&P 400 Mid-Cap Index of 0.85. Mountain Fund is subject to how much systematic risk?

A) 90%

B) 22%

C) 72%

D) 85%

A

C) Explanation
R-squared gives us the amount of systematic risk, and we have been given R (correlation coefficient). So, we square 0.85 to come up with an R-squared of 0.7225, or 72%.

LO 6.2.5

255
Q

Identify the CORRECT statements pertaining to the types of risk.

I. Reinvestment rate risk is the uncertainty that surrounds the rate of return that can be earned on reinvested coupon income.

II. Interest rate risk is the risk that the market price of an investment will decline as the result of changes in market interest rates.

III. Exchange rate risk is the variability in returns on securities caused by currency fluctuations.

IV. Business risk is the uncertainty of operating income.

A) II, III, and IV

B) I, II, and IV

C) I, II, III, and IV

D) I and III

A

C). Explanation
All of these statements are correct. Investment risk may broadly be categorized as either unsystematic or systematic risk; both types of risk together constitute total, or absolute, risk.

LO 6.1.2

256
Q

Jane owns a large-cap index fund, and you want to help her diversify by introducing a small-cap international fund. You are considering a weighting of 35% in the small-cap fund and 65% in the large-cap fund. You have determined that the covariance between the two funds is 6. If the standard deviation of the large-cap fund is 18, and the standard deviation of the small-cap fund is 23, then what is the standard deviation of the portfolio?

A) 15.70%

B) 17.75%

C) 14.30%

D) 19.75%

A

C)

257
Q

Most fixed-income securities are subject to which of the following risks?

I. Purchasing power risk
II. Liquidity risk
III. Default risk
IV. Reinvestment rate risk

A) I, III, and IV
B) I and II
C) I, II, III, and IV
D) II, III, and IV

A

C)
Fixed-income securities are subject to a number of risks including purchasing power, liquidity, default, and reinvestment rate risk.

LO 6.1.2

258
Q

Stock XYZ has an average return of 12%; its returns fall within a range of –2% to +26% approximately 68% of the time. Which one of the following numbers is closest to the standard deviation of returns of Stock XYZ?

A) 28%

B) 19%

C) 14%

D) 8%

A

C)
A standard deviation of 14% means an investor can expect a return on an investment to vary ±14 from the average return approximately 68% of the time.

LO 6.2.2

259
Q

Which of the following statements regarding investment theory is NOT correct?

A) Combining two stocks with a negative covariance can significantly reduce the portfolio’s standard deviation.

B) In a well-diversified portfolio, diversifiable risk is zero.

C) A correlation coefficient of 0.14 between the returns of Stock A and Stock B indicates that very little of Stock A’s returns can be attributed to the returns of Stock B.

D) The beta coefficient may be used to help select a portfolio that is consistent with an investor’s willingness to assume unsystematic risk.

A

D) Explanation
Beta is a measure of systematic risk, not unsystematic risk. The beta coefficient may be used to help select a portfolio that is consistent with an investor’s willingness to assume systematic risk.

LO 6.2.1

260
Q

An investor is interested in holding a diversified portfolio to reduce unsystematic risk. This can best be accomplished by buying stock in

A) companies with low betas.

B) foreign companies.

C) companies with strong earnings and revenue growth.

D) companies with low correlation coefficients between them.

A

D) Explanation
Holding stocks that have a low correlation coefficient between them will result in a diversified portfolio that reduces and virtually eliminates the degree of unsystematic (business) risk in the portfolio. Buying stocks in international companies and stocks with low betas can help to reduce systematic risk, but only if they have low correlations with other stocks. Buying stocks in companies with strong revenue and earnings growth often results in acquiring significant company-specific risk that is attributable to the underlying business.

LO 6.2.4

261
Q

The All-American Fund has a covariance of +47 with the S&P 500, and a correlation coefficient of .77 with the S&P 500. Your client is wondering how similar the All-American Fund’s price movement has been when compared with the S&P 500. You advise your client that

A) 77% of the fund’s price movement is explained by the S&P 500.

B) 23% of the fund’s price movement is explained by the S&P 500.

C) 47% of the fund’s price movement is explained by the S&P 500.

D) 59% of the fund’s price movement is explained by the S&P 500.

A

D) Explanation
R-squared gives us the amount of systematic risk, and we have been given R (correlation coefficient). So we square 0.77 to come up with an R-squared of 0.5929, or 59%.

LO 6.2.5

262
Q

Which of these statements concerning portfolio diversification is CORRECT?

A) Only systematic risk is reduced as diversification is increased.

B) The benefits of diversification are not realized until at least 25 individual securities are included in the portfolio.

C) By increasing the number of securities in a portfolio, the total risk would be expected to fall at a decreasing rate.

D) Diversification reduces the portfolio’s expected return because diversification reduces a portfolio’s total risk.

A

C) Explanation
The answer is by increasing the number of securities in a portfolio, the total risk would be expected to fall at a decreasing rate. As more and more securities are added to a portfolio, diversification benefits begin to diminish. The main attraction of diversification is the reduction of risk without an accompanying loss of return.

LO 6.1.1

263
Q

Diversification reduces

A) purchasing power risk.
B) systematic risk.
C) unsystematic risk.
D) market risk.

A

C) Explanation
The answer is unsystematic risk. Unsystematic risk can be diversified away by investing in approximately 10–15 large company stocks in different industries and 25–30 small company stocks in different industries. Systematic risk cannot be reduced by diversification.

LO 6.1.1

264
Q

How can an investor attempt to realize a portfolio’s greatest amount of return per unit of risk?

I. Having a shorter investment time horizon.

II. Structuring a portfolio that contains assets with high correlation to each other.

A) Both I and II
B) II only
C) I only
D) Neither I nor II

A

Explanation

D) The answer is neither I nor II. Realizing the greatest amount of return per unit of risk may be pursued by 1) having a longer (not shorter) investment time horizon, 2) structuring a portfolio that contains assets with low (not high) correlation to each other, or both.

LO 6.1.1

265
Q

ABC Corporation is a manufacturer of electronic devices used in the manufacturing of airplanes. Five years ago, the corporation floated a $100 million bond issue that would be used to finance improvements at its main manufacturing and distribution center. However, orders for its products have dropped dramatically due to much lower than anticipated demand. The company believes it may miss paying the coupon payment on the bond issue in the upcoming fiscal year. Identify which of these risks the owners of ABC Corporation bonds may be subject to by holding the bonds.

A) Regulation risk
B) Reinvestment rate risk
C) Default risk
D) Market risk

A

C)Explanation
The answer is default risk. Default risk is the risk that a business will be unable to service its debt obligations.

LO 6.1.2

266
Q

Which of these are nondiversifiable risks?

I. Business risk
II. Interest rate risk
III. Market risk
IV. Purchasing power risk

A) I and II
B) III only
C) I, II, III, and IV
D) II, III, and IV

A

Explanation

D) The answer is II, III, and IV. Business risk is a type of diversifiable, or unsystematic, risk.

LO 6.1.1

267
Q

Identify which of these is NOT an example of systematic risk.

A) Business risk
B) Interest rate risk
C) Purchasing power risk
D) Reinvestment rate risk

A

Explanation

A) The answer is business risk. Also known as nondiversifiable risk, systematic risk reflects the uncertainty of returns associated with an investment in any type of asset. Business risk is an example of an unsystematic risk.

LO 6.1.2

268
Q

Bobby has these securities in his portfolio: ABC common stock, XYZ common stock, PQR mutual fund (domestic small cap), DEZ mutual fund (foreign small cap), 30-year Treasury bond, and 5-year Treasury note. Point out the risk that should NOT concern Bobby.

A) Reinvestment rate risk
B) Systematic risk
C) Financial risk
D) Default risk

A

Explanation
D) The answer is default risk. Treasuries are considered default risk-free. Financial risk is the uncertainty introduced from the method by which a firm finances its assets (i.e., debt versus equity financing). Reinvestment rate risk is the risk that as cash flows are received they will be reinvested at lower rates of return than the investment that generated the cash flows. Systematic risk is the risk that all securities are subject to and typically cannot be eliminated through diversification.

LO 6.1.2

269
Q

Steve and Haley, ages 48 and 45 respectively, invest in large-cap stocks, international stock mutual funds, and rental real estate. They consider themselves moderately aggressive investors. Their investment portfolio is subject to which of these investment risks?

I. Investment manager risk
II. Financial risk
III. Exchange rate risk
IV. Default risk

A) II and IV
B) I, II, and III
C) I only
D) I, II, III, and IV

A

Explanation

B) The answer is I, II, and III. Their investment portfolio is subject to all of these risks except default risk. Investment manager risk is associated with the skills and philosophy of their mutual fund portfolio managers. Financial risk is the risk that a company’s financial structure may negatively affect the value of an equity investment. By holding investments in international stock mutual funds, they are subject to exchange rate risk.

LO 6.1.2

270
Q

Which of these statements regarding investment risk is CORRECT?

I. A firm’s decision to buy back some of its own stock in the open market by borrowing funds through a new bond issue is an example of reinvestment rate risk.

II. Rising inflation represents purchasing power risk.

III. A decline in a firm’s share price as a result of a 20% decline in the S&P 500 Index represents market risk.

IV. A reduction in the value of an international stock mutual fund because of a depreciation of the Euro is an example of exchange rate risk.

A) I and II
B) II, III, and IV
C) IV only
D) I, II, and III

A

B) Explanation
The answer is II, III, and IV. Only statement I is incorrect. A firm’s decision to buy back some of its own stock in the open market by borrowing funds through a new bond issue is an example of financial risk.

LO 6.1.2

271
Q

Exchange rate risk refers to fluctuations in

A) the price of one currency relative to other currencies.
B) the value of an investor’s portfolio.
C) the values of bonds and other debt instruments.
D) the prices of stocks on the New York Stock Exchange.

A

A) Explanation
The answer is the price of one currency relative to other currencies. Relative currency prices and changes to them are the basis of exchange rate risk.

LO 6.1.2

272
Q

Which of these types of risk is associated with the degree to which a company utilizes debt to finance its operations?

A) Credit risk
B) Default risk
C) Business risk
D) Financial risk

A

D)
The answer is financial risk. Financial risk is associated with the degree to which a company utilizes debt to finance its operations.

LO 6.1.2

273
Q

ABC Holding Company has constructed a portfolio containing these four securities:

Security Percent of Portfolio Beta
Stock A 50% 0.80
Stock B 10% 1.54
Stock C 15% 0.60
Stock D 25% 1.16
Based on this information, choose the CORRECT statement.

A) The weighted beta for the portfolio is 0.9340.
B) Stock C is the riskiest.
C) Stock B is the least risky.
D) The weighted beta for the portfolio is 1.0250.

A

A) Explanation
The answer is the weighed beta for the portfolio is 0.9340.

Calculated as follows: (0.50 × 0.80) + (0.10 × 1.54) + (0.15 × 0.60) + (0.25 × 1.16) = 0.40 + 0.154 + 0.09 + 0.29 = 0.9340. With a beta of 1.54, Stock B is the riskiest; whereas Stock C, with a beta of 0.60, is the least risky.

LO 6.2.1

274
Q

Investors who want to bear the least amount of risk should acquire stocks with beta coefficients

A) less than 1.0.
B) greater than 1.0.
C) greater than 1.5.
D) less than 0.5.

A

D) Explanation
The answer is less than 0.5. When seeking investments having the least amount of risk, the lowest beta should be selected.

LO 6.2.1

275
Q

A beta coefficient of 1.3 indicates that a stock

A) has more unsystematic risk than the market.
B) has less unsystematic risk than the market.
C) is more volatile than the market.
D) is less volatile than the market.

A

Explanation

C) The answer is that the stock is more volatile than the market. A beta that is higher than 1.0 indicates that the stock’s volatility and risk are higher than that of the market.

LO 6.2.1

276
Q

What is the weighted average beta of a portfolio with 20% in Stock A with a beta of 0.9, 50% in Stock B with a beta of 1.2, and 30% in Stock C with a beta of 1.1?

A) 1.14
B) 1.18
C) 1.20
D) 1.11

A

Explanation

D) The answer is 1.11. Keystrokes are 0.9, INPUT, 20, ∑+, 1.2, INPUT, 50, ∑+, 1.1, INPUT, 30, ∑+, SHIFT, 6 (alternate function is weighted average) = 1.11.

LO 6.2.1

277
Q

When analyzing various investment alternatives, investors would generally choose which of these?

A) An investment exhibiting a low positive skewness and a leptokurtic distribution

B) An investment exhibiting a low positive skewness and a platykurtic distribution

C) An investment exhibiting a high positive skewness and a platykurtic distribution

D) An investment exhibiting a high positive skewness and a leptokurtic distribution

A

Explanation
D) The answer is an investment exhibiting a high positive skewness and a leptokurtic distribution. Investments exhibiting high positive skewness have a larger than average number of positive price movements. Also, investments exhibiting a leptokurtic distribution have more observations clustered closely around the mean, resulting in a lower variance. Investors prefer a large number of positive returns with low risk.

LO 6.2.2

278
Q

Three years ago, ABC Growth and Income Fund has produced these total returns:

Year	Return
1	18.5%
2	−14.3%
3	9.75%
Assuming a bell-shaped distribution of returns, calculate the percent of returns that should fall above 21.63%.

A) 50%
B) 34%
C) 16%
D) 84%

A

Explanation
The answer is 16%. The standard deviation of the returns is 16.98% and the arithmetic mean is 4.65%. One standard deviation to the right of 4.65% is 21.63%. Fifty percent of returns lie below 4.65% and an additional 34% of returns fall between 4.65% and 21.63% resulting in 16% of returns falling above 21.63%.

Calculations:

Arithmetic mean

(18.5 − 14.3 + 9.75) ÷ 3 = 4.65%

Standard deviation (keystrokes are shown for the HP 10bII+)

  1. 5 [Σ+]
  2. 3 [+/−][Σ+]
  3. 75 [Σ+]

[SHIFT] [Sx,Sy] (8 key)

16.9843, or 16.98%

LO 6.2.2

279
Q

STU Corporation stock has an average rate of return of 24% and a standard deviation of 10%. The risk-free rate of return is 4%. Assuming the historical returns for STU stock are normally distributed, calculate the probability that this stock will have a return in excess of the risk-free rate of return.

A) 34.0%
B) 95.0%
C) 2.5%
D) 97.5%

A

Explanation
The answer is 97.5%. The probability of a return above 24% is 50%. The probability of a return between 4% and 24% is 47.5% (95% ÷ 2).

Therefore, the probability of a return above 4% is 97.5% (50% + 47.5%).

LO 6.2.2

280
Q

A distribution with a mean that is less than its median most likely

A) is negatively skewed.
B) has negative excess kurtosis.
C) is positively skewed.
D) symmetrical.

A

Explanation
A) The answer is that the distribution is negatively skewed. A distribution with a mean that is less than its median is a negatively skewed distribution. A negatively skewed distribution is characterized by many small gains and a few extreme losses. Note that kurtosis is a measure of the peakedness of a return distribution. In a symmetrical distribution, the mean, median, and mode are all equal.

LO 6.2.2

281
Q

If a security has an average return of 14.2% and a standard deviation of 8.4%, then

A) the security’s returns can be expected to be between 8.4% and 14.2% approximately 95% of the time.

B) the security’s returns can be expected to be between 5.8% and 22.6% approximately 68% of the time.

C) the security’s returns can be expected to always be positive.

D) the security’s annual volatility can be expected to be within a range approximately 8.4% above and 8.4% below the current fair market value.

A

B) Explanation
The answer is the security’s returns can be expected to be between 5.8% and 22.6% approximately 68% of the time. This security can be expected to have a return that does not range beyond one standard deviation on either side of its average return approximately 68% of the time.

LO 6.2.2

282
Q

Consider this information regarding two possible investments: Stocks J and K.

Stock J:

Expected return: 11.5%

Standard deviation: 8%

Stock K:

Expected return: 8.2%

Standard deviation: 6%

Identify which of these investments you would prefer and why.

A) Stock K because it has the highest coefficient of variation
B) Stock K because it has the least risk
C) Stock J because it has the lowest coefficient of variation
D) Stock J because it has the highest expected return

A

C). Explanation
The answer is Stock J because it has the lowest coefficient of variation. The stock with the lower coefficient of variation (CV) provides the least amount of risk for a given level of return. CV = standard deviation of asset ÷ expected return of asset.

Stock J: CV = 0.08 ÷ 0.115 = 0.6957

Stock K: CV = 0.06 ÷ 0.082 = 0.7317

LO 6.2.3

283
Q

You have recommended a growth mutual fund to a new client. The client considered your recommendation and asked why he should not invest in another fund that he had been following, which appeared to have a better performance over the past three years. You explained the concept of risk-adjusted performance and obtained this information about the two funds:

Your Fund	Client Fund
Three-year total return	13.5%	14.75%
Average P/E ratio	20%	24%
Standard deviation	19%	23%
Beta	1.03	1.24
Which fund would you recommend based on each fund's relationship between risk and return?

I. Your fund, because its coefficient of variation is 1.41, compared to the client’s coefficient of variation of 1.56

II. Client fund, because its higher beta dictates that its return should also be higher, which in fact occurred

III. Client fund, because standard deviations and betas change over time and the statistics are close enough so that the fund with the better return should be chosen

A) I only
B) II and III
C) I and III
D)II only

A

A) Explanation
The answer is I only. The standard deviation is divided by the total return to obtain the coefficient of variation. A beta is higher does not mean that any higher return is acceptable. The client’s fund has higher risk as measured by both standard deviation and beta, but taking this higher risk does not provide sufficient return based on the coefficient of variation calculation.

LO 6.2.3

284
Q

Stock A has an expected mean return of 15% and a standard deviation of 22%; Stock B has an expected mean return of 11% and a standard deviation of 13%; and Stock C has an expected mean return of 18% and a standard deviation of 24%. You want to recommend one of these stocks to a client who is most interested in owning stocks that are more likely to deliver the expected mean return. Which stock should you recommend to meet this client’s requirement?

A) None of these
B) Stock C
C) Stock A
D) Stock B

A

D) Explanation
The answer is Stock B. The coefficient of variation is a measure of the degree of variation of returns compared with the expected mean return. The security with the lowest coefficient of variation is the one most likely to deliver periodic returns closest to its expected return. The coefficients of variation of the three securities are 1.47 for Stock A, 1.18 for Stock B, and 1.33 for Stock C. Stock B should be recommended.

LO 6.2.3

285
Q

Bobby owns ABC stock that has mean return of 10.65%, a beta of 1.12, and a standard deviation of 9.05%. He decides to purchase MEJ stock that has a mean return of 11.5%, a beta of 0.98, and a standard deviation of 12.3%. Assume these stocks are weighted in the portfolio 70% for ABC and 30% for MEJ. Also, these stocks exhibit a covariance of 19.86. Calculate the standard deviation for this two-asset portfolio.

A) 10.02%
B) 7.88%
C) 1.16%
D) 3.23%

A

B) Explanation
The answer is 7.88%. To determine the standard deviation of a two-asset portfolio, use this formula:

[W2Aσ2A + W2Bσ2B + 2WAWB(COVAB)]1/2

[(0.72 × 9.052) + (0.32 × 12.32) + (2 × 0.7 × 0.3 × 19.86)]1/2

[(0.49 × 81.90) + (0.09 × 151.29) + (8.3412)]1/2

[40.1310 + 13.6161 + 8.3412]1/2

62.08831/2

= 7.8796, or 7.88%

Note the standard deviation for the portfolio is lower than the standard deviations for each security. This result directly supports the low correlation between the returns of ABC and MEJ.

LO 6.2.4

286
Q

All of these statements concerning the use of the correlation coefficient in reducing portfolio risk are CORRECT except

A) combining two securities with zero correlation (statistical independence) reduces portfolio risk, but cannot eliminate it.

B) because securities typically have some positive correlation with each other, risk can be reduced, but seldom eliminated.

C) combining securities with perfect positive correlation provides no portfolio risk reduction.

D) combining two securities with perfect negative correlation provides no portfolio risk reduction.

A

D) Explanation
The answer is combining two securities with perfect negative correlation provides no portfolio risk reduction. Combining two securities with perfect negative correlation could eliminate risk altogether. This is the principle behind hedging strategies.

LO 6.2.4

287
Q

Which one of these factors has the greatest impact on the standard deviation of a two-asset portfolio?

A) The standard deviation of each security in the portfolio
B) The weight of each security in the portfolio
C) Covariance
D) The portfolio’s beta

A

Explanation

C) The answer is covariance. Covariance is the most important variable in minimizing the standard deviation of a portfolio. The weight and standard deviation are not as critical as the covariance of the two securities. Beta is not used in the formula to compute a portfolio’s standard deviation.

LO 6.2.4

288
Q

An analysis of the monthly returns for the past year of a mutual fund portfolio consisting of two funds revealed these statistics:

                                Fund A      Fund B Total return	               18%           11% Standard deviation	23%		16% Percentage of portfolio	35%		65%
Correlation coefficient (R)		0.25	
What is the standard deviation of the portfolio?

A) 13.16%
B) 19.50%
C) 14.66%
D) 18.45%

A

Explanation
The answer is 14.66%. The covariance may be calculated first:

COV = (23) × (16) × (0.25)

COV = 92

289
Q

Portfolio A has a standard deviation of 55%, and the market has a standard deviation of 40%. The correlation coefficient between Portfolio A and the market is 0.50. Calculate the percentage of total risk that is unsystematic.

A)
100%
B)
25%
C)
75%
D)
50%
A

Explanation
The answer is 75%. The coefficient of determination explains the percentage of change in the dependent variable that can be explained by changes in the independent variable. Therefore, 25% (0.50 × 0.50) of returns are explained by changes in the market. To determine the percentage of returns that are explained by unsystematic risk, subtract the systematic risk from 1. Therefore, the return explained by unsystematic risk is (1 − 0.25) = 0.75, or 75%.

LO 6.2.5

290
Q

The Gemini Fund has a correlation coefficient of 0.80 with the S&P 500 Index. How much of the price movement of the Gemini Fund can be explained by the S&P 500 Index?

A) 100%
B) 75%
C) 80%
D)
64%
A

Explanation

D) The answer is 64%. The correlation coefficient (R) has been given, so it needs to be squared (R2) in order to come up with the coefficient of determination (0.802 = 0.64).

LO 6.2.5

291
Q

An analysis of the monthly returns for the past year of a mutual fund portfolio consisting of two funds revealed these statistics:

Fund A		Fund B
Total return	18%		11%
Standard deviation	23%		16%
Percentage of portfolio	35%		65%
Correlation coefficient (R)		0.25	
What is the coefficient of determination (R2) of Fund A and Fund B?

A) 84.64%
B) 6.25%
C) 50.00%
D) 21.49%

A

B) Explanation
The answer is 6.25%. The coefficient of determination is the square of the correlation coefficient (0.25)2 = 0.25 × 0.25 = 0.0625, or 6.25%.

LO 6.2.5

292
Q

Your client purchased a call option of RBN Corp. for $470. The exercise price was $40, and the market price of RBN Corp. was $43.50. Six months later, the market price of RBN Corp. was $50 and the client sold the call for $750.

What was the holding period return on this investment?

A) 62.5%

B) 28.8%

C) 37.5%

D) 59.6%

A

D) Explanation
HPR =750−470/ 470=0.596=59.6%
LO 7.1.1

293
Q

An investor who owns a sector fund that has substantial unsystematic risk and would like to know how a portfolio manager performed on a risk-adjusted basis would use which of the following indicators?

A) Jensen’s alpha

B) Beta

C) Sharpe ratio

D) Treynor ratio

A

C) Explanation
Sharpe uses standard deviation and assumes the portfolio is not well diversified and measures total risk.

LO 7.2.1

294
Q

Sammy owns a 7% corporate bond that is currently trading at $1,040. The bond matures in 22 years; however, it is callable in nine years at a 3% premium over par. What is the yield-to-call on Sammy’s bond?

A) 6.57%

B) 6.97%

C) 6.65%

D) 6.82%

A

C) Explanation
The call premium is 3%, which would be a price of $1,030. The keystrokes are as follows:

HP 10bII+
(1040)	PV
1030	FV
35	PMT
18	N
I/YR =	3.3255 × 2 = 6.65%

LO 7.4.1

295
Q

Your client purchased the Zenith Fund three years ago at $13.16. Here are the year-end prices of the fund up until today:

20X7. $14.21
20X8. $15.86
20X9. $14.78

What is the geometric return of Zenith Fund for this three-year period?

A) 5.88%

B) 4.61%

C) 3.95%

D) 5.26%

A

C) Explanation
The $13.16 has grown to $14.78 over a three-year period, so ($13.16) is the PV: $14.78 FV, 3 N, and solve for I/YR, which equals 3.95%.

LO 7.1.1

296
Q

Zenith Mutual Fund has had the following annual returns: +12%, +18%, +22%, and –13%. What is the Zenith Fund’s geometric mean return?

A) 9.75%

B) 11.66%

C) 10.17%

D) 8.83%

A

D) Explanation
The simplest way to do this problem is to see how much $1 would have grown to over the four years, and then do a simple time value of money calculation.
$1.00 × 1.12 × 1.18 × 1.22 × 0.87 = $1.4027. (1) PV,
1.4027 FV,
4 N,
I/YR = 8.8281%.

LO 7.1.1

297
Q

A(n) ______________________ average allows small companies to have as much influence as large companies in the average; a(n)_________________ average gives greater influence to large companies than to small companies in the average; and a(n) ______________________ average gives greater influence to high-priced stocks than to low-priced stocks in the average.

A) equally weighted; capitalization-weighted; price-weighted

B) capitalization-weighted; price-weighted; equally weighted

C) price-weighted; equally weighted; capitalization-weighted

D) price-weighted; capitalization-weighted; equally weighted

A

A) Explanation
For a price-weighted index, higher priced stocks have more influence on the overall movement of this index than lower priced stocks. For a market capitalization weighted index, such as the S&P 500, a stock with a market capitalization value of $25 million will have 10 times the impact of a stock with a market capitalization value of a $2.5 million company.

LO 7.3.1

298
Q

What is the taxable equivalent yield on a municipal bond with an 8.75% return for an investor in the 24% marginal tax bracket?

A) 11.51%

B) 6.65%

C) 8.75%

D) 2.10%

A

A) Explanation
The formula for solving this problem is 8.75% ÷ (1 – 0.24) = 8.75% ÷ 0.76 = 11.51%.

LO 7.4.1

299
Q

Which of the following statements regarding performance measures is CORRECT?

A) The reliability of their betas is important for the Jensen and Sharpe performance measures.

B) The Sharpe ratio uses beta as its measure of risk.

C) A negative alpha indicates the investment lost money.

D) Jensen’s alpha may be used by itself to judge an investment.

A

D) Explanation
Beta is the risk measure for alpha, but Sharpe uses standard deviation as its risk measure. Therefore, the reliability of beta is relevant for alpha. Jensen’s alpha can be used by itself to judge an investment; the Sharpe ratio must be used in comparison with another Sharpe ratio in judging an investment. A negative alpha indicates the investment did not perform as well as expected given the risk taken. For example, an alpha of –1 means the investment underperformed by 1% compared to what it was expected to return. Accordingly, a negative alpha does not necessarily mean the investment lost money.

LO 7.2.1

300
Q

Janice, who is in the 35% marginal income tax bracket, would like to purchase a bond for her investment portfolio. Assuming all of the bonds are of similar investment quality, which would produce the highest after-tax yield?

A) 2.75% U.S. Treasury bond

B) 5.25% corporate bond

C) 3.55% municipal bond

D) 2.25% U.S. Treasury note

A

C) Explanation
Janice should purchase the municipal bond based on the following after-tax yield calculations:

U.S. Treasury bond [2.75% × (1 − 0.35)]

1.79%

Corporate bond [5.25% × (1 − 0.35)]

3.41%

Municipal bond (tax-free)

3.55%

U.S Treasury note [2.25% × (1 − 0.35)]

1.46%

LO 7.4.1

301
Q

To create a market index similar to the Dow Jones Industrial Average, which of the following approaches should be used?

A) Dollar-weight the stocks in the index

B) Value-weight the stocks in the index

C) Price-weight the stocks in the index

D)Take a geometric mean return of the stocks in the index

A

C) Explanation
Higher priced stocks within this index have more influence on the overall movement of this index than lower priced stocks. That is, a $25 stock has five times the impact of a $5 stock.

LO 7.3.1

302
Q

Quincy has narrowed his choice down to the following four mid-cap funds, which is a category he wants to add to his portfolio. Which fund should Quincy choose?

              A Fund.   J Fund.    Q Fund      Z Fund

Return 12% 9% 13% 11%
Alpha 0.52 –1.04 0.66 1.10
Treynor 0.68 0.21 0.58 0.62
Sharpe 0.44 0.32 0.21 0.37
R-squared. 0.88 0.78 0.82 0.84

A) J Fund

B) A Fund

C) Z Fund

D) Q Fund

A

C) Explanation
The R-squared for all four funds is greater than 0.70, meaning beta is reliable. Alpha uses beta, and is a measure of absolute return, and the best measure to use if beta is reliable. The fund with the highest alpha is the Z fund.

LO 7.2.1

303
Q

Your client purchased a call of KLN Corp. for $800. The exercise price was $35, and the market price of KLN Corp. stock was $38. Six months later, the market price of KLN Corp. stock was $40 and the client sold the call for $1,250.

What was the holding period return on this investment?

A) 8.57%

B) 63.82%

C) 14.28%

D) 56.25%

A
D) 
Explanation
HPR=sale  price − purchase  price / purchase  price
=
1
,
250
−
800
800
=
56.25
%
LO 7.1.1
304
Q

Ophelia is considering the purchase of the Quest Mutual Fund, which has a beta of 1.2, a standard deviation of 15, and a return of 11%. The current risk-free rate is 4%, and the market risk premium is 7%. Should Ophelia purchase the fund?

A) Yes, because the fund has an 11% return.

B) No, because of the fund’s Sharpe ratio.

C) No, because the fund has a negative alpha.

D) Yes, because of the fund’s Treynor ratio.

A

C) Explanation
This question can be answered with or without a calculation. We have nothing to compare Quest Mutual Fund to, and both Sharpe and Treynor are comparative measures, so neither can be used to evaluate the fund. The answer “Yes, because the fund has an 11% return” just restates the return of the fund without taking risk into account.

305
Q

Tripp is an investor in the 32% marginal tax bracket. If he invests in a 4.75% municipal bond, his taxable equivalent yield (TEY) would be

A) 6.99%.

B) 3.23%.

C) 6.27%.

D) 4.75%.

A

A) Explanation
taxable equivalent yield = tax-exempt yield ÷ (1 − marginal tax rate) = 4.75% ÷ (1 − 0.32) = 6.99%

LO 7.4.1

306
Q

Betty’s equity portion of her investment portfolio is composed of five different stocks. She would like to determine the overall risk-adjusted performance of the stocks, which have risen 18% in the past year. They are equally dollar-weighted and have an overall weighted beta of 1.2. The stock market has been strong lately, rising 15% over the last 12 months.

Which of the following measures should Betty use in determining the risk-adjusted performance and why?

A) Sharpe ratio, because it takes into account the performance of the overall market

B) Treynor ratio, because it measures overall risk

C) Sharpe ratio, because the portfolio is not fully diversified

D) Jensen’s alpha, because the portfolio contains stocks only

A

Explanation
C) Because the portfolio is not fully diversified (only five stocks), the Sharpe ratio is the only one that can be used. This ratio uses standard deviation in its formula. Beta, which measures only systematic risk, assumes a diversified portfolio, and it is used in the Treynor ratio and Jensen’s alpha.

LO 7.2.1

307
Q

All of the following statements concerning bond yield measurements are CORRECT except

A) if the bond is selling at a premium, the coupon rate is greater than the YTM.

B) if the bond is selling at a discount, the market price is less than the par value.

C) if the current yield is 7% and the coupon rate is 6%, the bond is selling at a discount.

D) if the current yield is less than the yield to maturity (YTM), the bond is selling at par.

A

D) Explanation
This is a false statement. A bond is selling at par when the current yield equals the YTM.

LO 7.4.1

308
Q

Jill is in the 24% marginal tax bracket. She owns a $1,000 par value public purpose municipal bond that pays $35 interest semiannually.

What pretax yield on corporate bonds would be comparable to the yield on Jill’s current investment?

A) 5.32%

B) 6.28%

C) 8.96%

D) 9.21%

A

D)

309
Q

Robin purchased a 20-year bond with a duration of 11 years for $1,323.18. Which of the following statements is CORRECT?

A) The current yield is higher than both the coupon rate and the yield to maturity.

B) The coupon rate is lower than the YTM, and the current yield should be higher than the coupon rate.

C) The yield to maturity (YTM) is less than both the current yield and the coupon rate.

D) The coupon rate is higher than the yield to maturity, and the YTM is higher than the current yield.

A

C)
Explanation
CR = coupon rate

CY = current yield

YTM = yield to maturity

Premium bonds: CR > CY > YTM

Par bonds: CR = CY = YTM

Discount bonds: CR < CY < YTM

Because the bond was purchased at a premium, the yield to maturity is less than both the current yield and the coupon rate.

LO 7.4.1

310
Q

A portfolio of stocks generates the following returns over a four-year period:

20X6	=	19%
20X7	=	12%
20X8	=	-8%
20X9	=	14%
What is the geometric mean return for the four-year period?

A) 8.28%

B) 10.03%

C) 9.25%

D) 8.73%

A

D) Explanation
The simplest way to do this problem is to see how much $1 would have grown to over the four years, and then do a simple time value of money calculation. $1.00 × 1.19 × 1.12 × .92 × 1.14 = $1.3978. (1) PV, 1.3978 FV, 4 N, solve for I/YR = 8.73%.

LO 7.1.1

311
Q

Five years ago, XYZ Company issued a 20-year bond with a 4.75% coupon paid semiannually. The bond may be called at 104% of par, 10 years after issue. Assuming the bond is currently selling for $990, calculate the bond’s yield to call.

A) 4.95%
B) 2.84%
C) 5.18%
D) 5.68%

A

D) Explanation
The answer is 5.68%. The bond’s yield to call is calculated as follows:

Note: XYZ Company has the option to call the issue in five years.

PV = (990)

N = 10 (5 × 2)

PMT = 23.75 (47.50 ÷ 2)

FV = 1,040 (1,000 × 1.04)

Solve for I/YR = 2.84 × 2 = 5.68%

LO 7.4.1

312
Q

Jack experienced returns in his growth mutual fund over the last five years of 2%, 5.5%, -9%, 18%, and 26%. Calculate both the arithmetic and geometric means for this series of returns.

A) Arithmetic mean = 2.13%; geometric mean = 45.60%
B) Arithmetic mean = 8.50%; geometric mean = 7.80%
C) Arithmetic mean = 7.80%; geometric mean = 8.50%
D) Arithmetic mean = 42.50%; geometric mean = 14.60%

A

B) Explanation
The answer is Arithmetic mean = 8.50%; geometric mean = 7.80%.

Arithmetic mean: [(2% + 5.5% − 9% + 18% + 26%) ÷ 5] = 8.50%.

Geometric mean: PV = –1; FV = (1 + 0.02)(1 + 0.055)(1 – 0.09)(1 + 0.18)(1 + 0.26) = 1.4559; N = 5; solve for I/YR = 7.80%.

LO 7.1.1

313
Q

Assuming Mary earned a 3% return from dividend reinvestment, a 2% return from capital gain reinvestment, and a 9% return from share price appreciation on her mutual fund, calculate her total return.

A) 3%
B) 14%
C) 12%
D) 5%

A

B) Explanation
The answer is 14%. Total return on a stock or mutual fund may be thought of as the sum of the capital appreciation/depreciation on the underlying principal of the investment and any income or earnings generated from that investment. Therefore, Mary’s total return equals 14% (3% + 2% + 9%).

LO 7.1.1

314
Q

Johnny owns a municipal bond with a coupon rate of 4.25%. Assuming the annual inflation rate is 1.65%, calculate Johnny’s real rate of return on his bond.

A) 1.58%
B) 2.60%
C) 2.56%
D) 4.25%

A

C) Explanation
The answer is 2.56%. Johnny realized a real rate of return of 2.56%.

Real rate of return = {[(1 + 0.0425) ÷ (1 + 0.0165)] – 1} × 100 = 2.5578, or 2.56%.

LO 7.1.1

315
Q

Brantley recently purchased a new video game system on his credit card. Assuming the nominal annual percentage rate (APR) is 14.95% (compounded daily), calculate the effective annual rate (EAR).

A) 15.02%
B) 13.26%
C) 16.12%
D) 14.95%

A

C) Explanation
The answer is 16.12%. The effective annual rate on his credit card is 16.12%, calculated as follows:

EAR = [1 + (0.1495 ÷ 365)]365 – 1 = 0.1612, or 16.12%.

LO 7.1.1

316
Q

Jonathan purchased 500 shares of CPM stock for $12 per share. At the end of the first year, he made another purchase of 500 shares at a stock price of $12 per share. At the end of the third year, he sold all of the stock for $17 per share. In addition, the stock paid a dividend of 0.35 per share at the end of each year. Calculate the dollar-weighted return to Jonathan over the three-year period.

A) 13.50%
B) 11.63%
C) 15.60%
D) 17.46%

A

D)
The answer is 17.46%. Jonathan earned a dollar-weighted rate of return of 17.46% on CPM stock over the three-year period, calculated as follows:

CF0 = −12 × 500 = −6,000

CF1 = (−12 × 500) + (0.35 × 500) = −5,825

CF2 = (0.35 × 1,000) = 350

CF3 = (0.35 × 1,000) + (17 × 1,000) = 17,350

Solve for the internal rate of return (IRR/YR) = 17.4626, or 17.46%

LO 7.1.1

317
Q

Crowder made an investment that paid him an 8% nominal rate of return for the year in which he held the investment. During that year, the inflation rate was 3%. Based on this information, calculate Crowder’s inflation-adjusted return (real return).

A) 3.08%
B) 5.10%
C) 4.85%
D) 2.67%

A

C) Explanation
The answer is 4.85%. The inflation-adjusted return (IAR) is computed as:

IAR = [((1 + nominal rate of return) ÷ (1 + inflation rate)) − 1] × 100

= ((1.08 ÷ 1.03) − 1) × 100 = 4.8544, or 4.85%

LO 7.1.1

318
Q

Jefferson originally purchased 100 shares of XYZ stock for $45 per share. The stock is currently trading at $60 per share. The stock paid dividends of $2 per share in year 1 and $2.30 per share in year 2 (all paid at year end). If Jefferson has held the stock for two years, what is his holding period return?

A) 42.9%
B) 23.5%
C) 22.0%
D) 19.0%

A

A) Explanation
The answer is 42.9%. This is calculated as: [($60 - $45) + $2 + $2.30] ÷ $45 = 42.9%.

LO 7.1.1

319
Q

Harry purchased 100 shares of MNL common stock five years ago at a cost of $4,300. The stock paid the following dividends:

Year	Amount
1	$180
2	$180
3	$200
4	$225
5	$230
At the time the fifth-year dividend was paid, Harry sold the stock for $8,900.

What was Harry’s average annual compound rate of return (IRR) on MNL stock?

A) 9.35%
B) 12.68%
C) 15.66%
D) 19.21%

A

D) Explanation
The answer is 19.21%. Calculation follows:

HP 10bIi+
4,300	+/–, CFj
180	CFj
180	CFj
200	CFj
225	CFj
230	+
8,900	=, CFj
SHIFT, IRR/YR
(answer = 19.21%)
LO 7.1.1
320
Q

Nancy bought $4,000 worth of common stock two years ago. She received dividends of $30 each quarter for the first year and $35 each quarter for the second year. The stock currently is worth $4,100. The graph below summarizes inflows and outflows.

What is the internal rate of return that the stock has earned?

A) 3.61%
B) 4.45%
C) 1.11%
D) 0.99%

A

Explanation
B) The answer is 4.45%.

Keystrokes:

HP 10BII+
(set for 4 P/YR)
4000,	+/–, CFj
30,	CFj, 4, SHIFT, Nj
35,	CFj, 3, SHIFT, Nj
4100,	+, 35, =, CFj
SHIFT, IRR/YR
LO 7.1.1

PreviousNextGo to Summary

321
Q

The performance of two growth and income mutual funds is displayed below:

                                          ABC Fund        XYZ Fund Average  annual rate of return.                  8.65%       6.78% Standard deviation of returns	5.86%	9.98% Beta	                                        0.75	         1.00

Assuming a risk-free rate of return of 5%, which of these statements is CORRECT?

A) All of these statements are correct.
B) Based on the Treynor ratio, ABC Fund has a better risk-adjusted performance than XYZ Fund.
C) XYZ Fund has a higher level of systematic risk than ABC Fund.
D) The Sharpe ratio for XYZ Fund is 0.1784 and 0.6229 for ABC Fund.

A

Explanation
A) The answer is all of the statements are correct. When compared with another investment, the higher the Treynor ratio, the better the risk-adjusted performance of the asset. Therefore, ABC Fund with a Treynor ratio of 0.0487 has a better risk-adjusted performance than XYZ Fund. XYZ Fund has a beta greater than ABC Fund, indicating a higher level of systematic risk.

Calculations:

Treynor ratio for ABC Fund is 0.0487 [(0.0865 – 0.05) ÷ 0.75]

Treynor ratio for XYZ Fund is 0.0178 [(0.0678 – 0.05) ÷ 1.00]

Sharpe ratio for ABC Fund is 0.6229 [(0.0865 – 0.05) ÷ 0.0586]

Sharpe ratio for XYZ Fund is 0.1784 [(0.0678 – 0.05) ÷ 0.0998]

LO 7.2.1

322
Q

Consider the following information for the CPM International Growth Fund:

Average annual rate of return	7.45%
Average market rate of return	8.50%
Beta	1.25
Standard deviation	4.55%
Risk-free rate of return	3.50%
Select the statement that is NOT correct.

A) The fund manager underperformed the market over the given time frame.
B) The Treynor ratio for the fund is 0.0400.
C) The Sharpe ratio for the fund is 0.8681.
D) Jensen’s alpha for the fund is –2.30%.

A

Explanation
B) The answer is the Treynor ratio for the fund is 0.0400. Based on the information provided, the Treynor ratio for the fund is 0.0316, calculated as follows:

Sharpe ratio = (0.0745 – 0.0350) ÷ 0.0455 = 0.8681

Treynor ratio = (0.0745 – 0.0350) ÷ 1.25 = 0.0316

Jensen’s alpha = 7.45% – [3.50% + (8.50% – 3.50%) 1.25] = –2.30%

When Jensen’s alpha is positive, the fund manager outperformed the overall market. In this case, alpha indicates that the fund manager has underperformed.

LO 7.2.1

323
Q

Your client, Trace, owns a portfolio that earned 12% during the current year. His portfolio has a beta of 1.3 and a standard deviation of 14%. During the current year, the market (as represented by the S&P 500 index) earned 9%. If the current risk-free rate is 5%, which of the following accurately illustrates the Treynor ratios for Trace’s portfolio and the market?

A) 0.0538; 0.0400
B) 0.0050; 0.0400
C) 0.0050; 0.0100
D) 0.0538; 0.0100

A

Explanation
A) The answer is 0.0538; 0.0400. Calculations are as follows.

Trace’s Treynor = (0.12 – 0.05) ÷ 1.3 = 0.0538

Market’s Treynor = (0.09 – 0.05) ÷ 1 = 0.0400. The market has a beta of 1.0.

LO 7.2.1

324
Q

Mutual fund QUE has a correlation coefficient with the market of 0.82, a beta of 1.05, and a standard deviation of 4%. The risk-free rate of return is 3.5%, and the return on the market is 12%. Mutual fund POI has a Sharpe ratio of 2.05, a Treynor ratio of 0.11, and an alpha of 0.70%. Decide which of the following a rational investor would select if the market’s standard deviation is 2% and QUE realized a 13% return.

A) POI over QUE because QUE’s Treynor ratio is 0.09.
B) QUE over POI because QUE’s coefficient of variation is 0.31.
C) POI over QUE because QUE’s alpha is 0.58%.
D) QUE over POI because QUE’s Sharpe ratio is 2.38.

A

D) Explanation
The answer is QUE over POI because QUE’s Sharpe ratio is 2.38.

QUE’s alpha = 13% – [3.5% + (12% – 3.5%) 1.05] = 0.58%

QUE’s Treynor ratio = (0.13 – 0.035) ÷ 1.05 = 0.09

QUE’s Sharpe ratio = (0.13 – 0.035) ÷ 0.04 = 2.38

QUE’s coefficient of variation = 4% ÷ 13% = 0.31

Because QUE’s R2 equals 67% (0.82 × 0.82), alpha and Treynor are not appropriate performance measures for comparison purposes. Because we do not know POI’s coefficient of variation, we must use the Sharpe ratio to select the better risk-adjusted return.

LO 7.2.1

PreviousNextGo to Summary

325
Q

If a mutual fund’s beta and standard deviation are expected to decrease in the future, its average annual return and the market average annual return are expected to remain the same, and the risk-free rate is expected to remain constant, which of the following shows the real effect this would have on the following performance measures?

Option.   Alpha	   Sharpe Ratio
A	      increase	   decrease
B	      decrease	   decrease
C	      decrease	increase
D	      increase	increase

A) Option D
B) Option A
C) Option C
D) Option B

A

A) Explanation
The answer is Option D. A decrease in the risk level decreases the denominator of the Sharpe ratio, while the numerator stays constant, thereby increasing the Sharpe ratio. The decreased risk level, as measured by beta, decreases the expected return for the fund, while the actual portfolio return remains constant, thereby increasing the alpha.

LO 7.2.1

326
Q

An investor who would like to know how a portfolio manager performed relative to how the manager was expected to perform on a risk-adjusted basis would use which one of the following indicators?

A) Jensen’s alpha
B) Treynor ratio
C) beta
D) Sharpe ratio

A

Explanation
The answer is Jensen’s alpha. The indicator that measures performance in relation to what was expected on a risk-adjusted basis is Jensen’s alpha. A positive number (alpha) indicates that the manager performed better than expected on a risk-adjusted basis.

LO 7.2.1

327
Q

A mutual fund with an investment objective of growth and income has an alpha of +4, a beta of 1.5, and a Sharpe ratio of 1.15. The fund

A) should be purchased, because it has a relatively low level of risk in relation to return.
B) should not be purchased even though the rate of return compensates for the level of risk.
C) should be purchased, because the rate of return is high in relation to risk.
D) should not be purchased, because it has a low level of return in relation to risk.

A

Explanation
The answer is should be purchased, because the rate of return is high in relation to risk. A positive alpha indicates the fund performed better than it should have on a risk-adjusted basis. Also, an alpha of +4, which is very high, means it performed 4% better than expected.

LO 7.2.1

328
Q

Bonnie Brown is trying to decide which one of two fully diversified mutual funds has performed better on a risk-adjusted basis. Fund A has had an annual rate of return of 12% for the last four years, with a beta of .9 and an alpha of +2.6%. Fund B has had an annual rate of return of 14% for the last five years, with a beta of 1.2 and an alpha of +1.8%.

Which fund has had the better risk-adjusted performance, and why?

A) Fund B, because it has a longer track record
B) Fund A, because it has a lower beta
C) Fund B, because it has a higher rate of return
D) Fund A, because it has a higher alpha

A

Explanation

D) The answer is Fund A, because it has a higher alpha. Alpha is a measure of risk-adjusted performance. Because it was given in the problem, no calculation is required. Alpha and the alpha can be used to determine which fund has the best risk-adjusted performance.

LO 7.2.1

329
Q

Select the CORRECT statement regarding security market indexes and averages.

A) The Russell 2000 Index is used to benchmark small capitalization companies.

B) The S&P 500 Index is used by most professionals as a benchmark for U.S. large-cap equity investments.

C) The Wilshire 5000 Index is often used as a measure of the overall market within the United States.

D) All of these statements are correct.

A

Explanation

D) The answer is all of these statements are correct. Averages and indexes are constructed to inform investors about changes in the market. They also serve as benchmarks for the performance of investors’ portfolios and the performance of money managers.

LO 7.3.1

330
Q

Which of the following statements regarding security market indexes is CORRECT?

A) Market indexes reflect the average price behavior of a group of stocks at a given point in time.
B) The S&P 500 Index automatically adjusts for stock splits and dividends by focusing on market value instead of price.
C) The Russell 2000 Index is a well-known index used to benchmark large capitalization companies.
D) The Wilshire 100 Index is used as a measure of the financial stock sector.

A

Explanation

B) The answer is the S&P 500 Index automatically adjusts for stock splits and dividends by focusing on market value instead of price.

The Russell 2000 Index is a well-known index used to benchmark small capitalization companies. A market average, not a market index, reflects the average price behavior of a group of stocks at a given point in time. An index measures the current price behavior of a group of stocks in relation to a base value. The Wilshire 5000 index is used as a measure of the U.S. broad market.

LO 7.3.1

331
Q

When using a security market index to represent a market’s performance, the performance of that market over time is best represented by:

A) the change in the standard deviation of the index.
B) the percent change in the index value.
C) the change in the index value.
D) the index value.

A
Explanation
B) The answer is the percent change in the index value. Percentage changes in the value of a security market index over time represent the performance of the market, segment, or asset class from which the securities are chosen.

LO 7.3.1

332
Q

A taxable bond is selling at a yield to maturity (YTM) of 7.5%, and a similar quality tax-free bond is selling at a YTM of 5.2%. Which one of the following combined federal and state marginal tax brackets is the lowest bracket at which an investor living in the bond-issuing state should choose the tax-free bond over the taxable bond?

A) 35%
B) 28%
C) 15%
D) 31%

A

Explanation
The answer is 31%. The basic formula must be rearranged:

Tax equivalent yield
=
Tax-exempt yield
1
−
Marginal tax rate (federal + state)

So,

Marginal tax rate
=
1
−
Tax-exempt yield
Tax equivalent yield

Using this formula, we get:

1
−
0.052
0.075
=
0.3067

The closest answer, rounded, is 31%.

At the 15% combined marginal bracket, the tax-equivalent yield is 6.12%; at the 28% combined marginal bracket, the tax-equivalent yield is 7.22%; and at the 35% combined marginal bracket, the tax-equivalent yield is 8%.

But at the 31% combined marginal bracket, the tax-equivalent yield is 7.54%, which is the first bracket where the yield on the taxable bond (at 7.5%) is not high enough to justify purchasing it instead of the tax-exempt bond.

LO 7.4.1

333
Q

Billie purchased a 10-year U.S. Treasury bond with a 6.5% coupon paid semiannually. Assuming the bond is currently trading at $1,075, calculate its yield to maturity (YTM).

A) 3.25%
B) 6.50%
C) 5.51%
D) 2.76%

A

Explanation
C) The answer is 5.51%. YTM: PV = –1,075, FV = 1,000, PMT = 32.50 (6.5% × 1,000 ÷ 2), N = 20 (10 × 2), solve for I/YR = 2.757 × 2 = 5.51%. The YTM on the bond is 5.51%, which is lower than the coupon rate of 6.5%, further validating that the bond is trading at a premium.

LO 7.4.1

334
Q

A $1,000 U.S. Treasury note maturing in eight years is selling for $938.12. The semiannual coupon payment is $35. What is the yield to maturity (YTM) for the note?

A) 8.33%
B) 4.03%
C) 8.06%
D) 6.26%

A

Explanation
C) The answer is 8.06%. The note’s YTM is computed as follows:

PV = −938.12

PMT = 35

FV = 1,000

N = 8 × 2 = 16

Solve for I/YR = 4.03 × 2 = 8.06%

LO 7.4.1

335
Q

Andy purchased a four-year bond with a coupon rate of 7.5% paid semiannually. The bond is trading for $1,025 in the secondary market. Calculate the bond’s yield to maturity (YTM).

A) 3.39%
B) 4.34%
C) 6.78%
D) 8.05%

A

Explanation
C) The answer is 6.78%. The bond’s YTM is calculated as follows:

PV = –1,025; FV = 1,000; PMT = 37.50 (1,000 × 7.5% ÷ 2); N = 8 (4 × 2); solve for I/YR = 3.388 × 2 = 6.78%.

LO 7.4.1

336
Q

Robinson owns a municipal bond with a coupon rate of 2.75%. He is currently in the 32% federal marginal income tax bracket and resides in a state that does not impose a state income tax. Calculate his municipal bond’s taxable equivalent yield (TEY).

A) 6.75%
B) 2.75%
C) 3.66%
D) 4.04%

A

D) Explanation
The answer is 4.04%. The bond’s TEY is calculated as follows:

2.75% ÷ (1 – 0.32).

LO 7.4.1

337
Q

ABC Corporation has issued a 30-year callable bond with a 5.75% coupon at par. The current market price of the bond is $989.50. Calculate the current yield of this bond.

A) 5.81%
B) 5.69%
C) 5.75%
D) 2.91%

A

Explanation
A) The answer is 5.81%. The current yield is calculated as follows:

$57.50 ÷ $989.50 = 0.05811, or 5.81%.

LO 7.4.1

338
Q

Assuming a bond is currently selling for $960 with a $40 semiannual interest payment, calculate its current yield.

A) 8.00%
B) 8.33%
C) 4.00%
D) 4.17%

A

Explanation

B) The answer is 8.33%. The current yield is calculated by dividing the bond’s annual interest payment by the bond’s current market price. Therefore, the current yield of this bond is 8.33% ($80 ÷ $960).

LO 7.4.1

339
Q

Five years ago, XYZ Company issued a 20-year bond with a 4.75% coupon paid semiannually. The bond may be called at 104% of par, 10 years after issue. Assuming the bond is currently selling for $990, calculate the bond’s yield to call.

A) 4.95%
B) 2.84%
C) 5.18%
D) 5.68%

A

D) Explanation
The answer is 5.68%. The bond’s yield to call is calculated as follows:

Note: XYZ Company has the option to call the issue in five years.

PV = (990)

N = 10 (5 × 2)

PMT = 23.75 (47.50 ÷ 2)

FV = 1,040 (1,000 × 1.04)

Solve for I/YR = 2.84 × 2 = 5.68%

LO 7.4.1

340
Q

Jane considers herself to be a conservative investor. To generate additional income, she wants to add an investment-grade bond to her portfolio. She lives in a state that does not have an income tax and she is in the 35% federal income tax bracket. Select the best choice for her portfolio.

A) Bond B, A rated corporate debenture with a 4.75% coupon rate
B) Bond C, D rated corporate debenture with a 6% coupon rate
C) Bond A, AA rated municipal bond with a 3.5% coupon rate
D) Bond D, AAA rated Treasury bond with a 1.5% coupon rate

A

Explanation

C) The answer is Bond A, AA rated municipal bond with a 3.5% coupon rate. Even though Bond C has the highest after-tax rate of return, this bond would not be appropriate for Jane based on her desire for an investment-grade bond. Therefore, Bond A would be the best choice.

Calculations:

Bond A: 3.5%

Bond B: 4.75% × (1 – 0.35) = 3.0875%

Bond C: 6% × (1 – 0.35) = 3.90%

Bond D: 1.5% × (1 – 0.35) = 0.9750%

LO 7.4.1

341
Q

Jana believes that the allocation to emerging market equities in her portfolio has become overvalued, so she trims the holdings in that asset class and reinvests the proceeds in other asset classes that she believes are undervalued. Jana’s approach to asset allocation can best be described as

A) core-satellite asset allocation.

B) dynamic asset allocation.

C) strategic asset allocation.

D) tactical asset allocation.

A
D) Explanation
Tactical asset allocation continuously adjusts the asset allocation and class mix in an attempt to take advantage of changing market conditions and overall investor sentiment.

LO 8.5.2

342
Q

The anticipation of inflation suggests that the investor should

A) anticipate higher interest rates.

B) buy bonds.

C) avoid real estate investments.

D) sell stocks of gold companies.

A

A) Explanation
Real assets, which includes gold and real estate, should do well in inflationary times. Bonds do poorly because interest rates will increase to fight inflation, and increases in interest rates cause bond prices to fall.

LO 8.5.1

343
Q

In general, rising interest rates result in which of the following combinations?

A) Rising bond prices and falling stock prices

B) Falling stock and bond prices

C) Rising stock and bond prices

D) Falling bond prices and rising stock prices

A

B) Explanation
In general, rising interest rates result in falling stock and bond prices.

LO 8.5.1

344
Q

In analyzing the position of a portfolio in terms of risk/return on the capital market line (CML), superior performance exists if the fund’s position is ________ the CML, inferior performance exists if the fund’s position is ________ the CML, and equilibrium position exists if it is ________ the CML.

A) above; on; below

B) above; below; on

C) below; on; above

D) below; above; on

A

B) Explanation
According to modern portfolio theory, the CML defines performance of a portfolio. All portfolios should plot on the CML in proportion to the risk of the portfolio. The Y axis is return and the X axis is the risk level. If a fund has superior performance, its return will be above the return level for the given risk level; if inferior, its return will be below the return level for the given risk level; and if it is in equilibrium, then it will plot exactly on the CML.

LO 8.2.1

345
Q

A well-crafted investment policy statement will take all of the following into account except

A) anticipated liquidity needs.

B) the client’s time horizon.

C) investments the client is especially interested in considering for her portfolio.

D) hobbies the client intends to pursue in retirement.

A

D) Explanation
A properly crafted IPS will keep both the client and the advisor on track, and minimize any disagreements or confusion.

LO 8.4.1

346
Q

If information is generated randomly and information announcements are independent,

A) price changes will be influenced by the information about past prices and volume.

B) prices will change very quickly in response to the new random information.

C) price changes will be independent of each other and tend to move in a predictable fashion.

D) prices will not reflect all fully available information.

A

B) Explanation
If information is generated randomly and information announcements are independent, prices will tend to fully reflect all available information. In addition, price changes will be independent of one another and tend to occur random patterns.

LO 8.1.1

347
Q

Which of the following correctly describes the efficient frontier in portfolio theory?

A) It illustrates the optimal tradeoff between long- and short-term capital gains.

B) It quantifies systematic and unsystematic risk.

C) It identifies the optimal portfolio for the investor.

D) It indicates the highest returns for given levels of risk.

A

D) Explanation
At any point along the efficient frontier the highest return for a given level of risk is present. The optimal portfolio is the point on the efficient frontier that intersects with the investor’s indifference curve.

LO 8.1.1

348
Q

A client has a portfolio of blue-chip stocks that were purchased many years ago by her spouse. The spouse is now deceased and the client is considering her needs for income and feels the dividend yield on the stocks is not sufficient. You have decided she should be in 60% fixed-income. Which of the following sets of factors related to the recommended changes in the portfolio is the most important for the portfolio advisor to review?

A) Tax issues, risk tolerance level, and client goals

B) Risk tolerance level, liquidity, and social investing

C) Tax issues, liquidity, and legal constraints

D) Client goals, cash flows, and legal constraints

A

A) Explanation
Investments purchased years ago will probably have a low tax basis, which could affect decisions to sell them, or they may have become worthless, in which case they can be used as a tax deduction. Likewise, the risk level of the investment is probably unknown and may not be compatible with the client’s risk tolerance level and goals.

LO 8.4.1

349
Q

Roger is willing to invest in securities with above-average risk if he is rewarded for doing so. He has been following the stock of a company that he likes, but is concerned because the stock dropped 9% the last time the S&P 500 dropped 7%. Roger believes that an 11% return for the market next year would be good. The current market risk premium is 7.5% and the Treasury bill rate is 5.75%. The stock Roger has been following has the following characteristics:

Standard deviation.   18%
Dividend yield.  2.4%
P/E ratio.  17
P/E ratio relative to S&amp;P 500.  1.4
Beta.  1.25

Using the CAPM formula, calculate the required rate of return for the stock and determine if the stock appears to meet Roger’s criteria of investing in above-average-risk stocks only if he is rewarded for doing so.

A) The required rate of return is 15.13%, and the stock meets Roger’s criteria.

B) The required rate of return is 13.25%, and the stock meets Roger’s criteria.

C) The required rate of return is 11.00%, and the stock does not meet Roger’s criteria.

D) The required rate of return is 12.31%, and the stock does not meet Roger’s criteria.

A

A) Explanation
Required rate of return is 5.75% + (7.5%)1.25 = 15.13%. The fact that Roger believes 11% would be a good return is not relevant for computation of the required rate of return—only the computed market risk premium is relevant. The market return is computed as 7.5% + 5.75% = 13.25%. Roger will invest in an above-average risk stock (beta = 1.3) if he can be rewarded for taking the extra risk. The reward is the greater-than-market return of 15.13%.

LO 8.2.1

350
Q

A client complains to you that his portfolio performance merely matched the market over the last few years, and questions whether the fees you charge him are justified when you haven’t outperformed the market. You try to explain to your client that he should base his performance expectations on the strong form of the efficient market hypothesis that states

A) neither fundamental nor technical analysis nor inside information will result in superior performance.

B) fundamental analysis can lead to superior performance, but technical analysis cannot.

C) security prices are considered to be randomly determined.

D) inside information can lead to superior performance.

A

A) Explanation
Inside information may help in the semistrong form; fundamental analysis may help in the weak form; there is no hypothesis under which security prices are considered to be randomly determined.

LO 8.3.1

351
Q

Roberta is a private investor who researches individual stock purchases thoroughly. She uses charts and historical price movement to determine entry and exit points for her stock trades. Based on her behavior, Roberta

A) believes in the semistrong form of the efficient market hypothesis.

B) does not believe in the efficient market hypothesis (EMH) in any form.

C) believes in the weak form of the efficient market hypothesis.

D) believes in the strong form of the efficient market hypothesis.

A

B) Explanation
One who believes that fundamental analysis can result in superior performance believes in the weak form of the EMH. The weak form states that all historical price information is fully reflected in the stock price, therefore technical analysis will not produce superior results. The semistrong hypothesis states that neither fundamental nor technical analyses deliver superior performance. The strong form states that all information, including insider knowledge, is already reflected in the stock price.

LO 8.3.1

352
Q

Which of the following are implications of the weak form of the efficient market hypothesis (EMH)?

I. Stock prices fully reflect all historical price behavior.
II. Consistently superior performance is common.
III. Fundamental analysis may produce superior investment performance.
IV. Fundamental and technical analysis can produce superior investment performance.

A) I and III

B) I only

C) II and IV

D) I and IV

A

A)Explanation
Technical analysis is not considered valuable under any of the forms of the EMH; fundamental analysis is considered valuable under the weak form only.

LO 8.3.1

353
Q

Which of the following characteristics are associated with the market anomaly known as the neglected-firm effect?

I. Low price-to-earnings (P/E) ratio stocks outperform high P/E stocks.

II. Stocks of foreign companies outperform known domestic stocks.

III. Neglected-firm stocks underperform large capitalization stocks.

IV. Stocks not frequently followed by analysts outperform widely followed stocks.

A) I only

B) III and IV

C) IV only

D) II and III

A

C) Explanation
Neglected firms are those that are neglected (not followed) by many financial analysts. When such companies can be found, the market for those companies may not be efficient, and investors who can take the time to analyze these companies may be able to take advantage of undervalued situations.

LO 8.3.1

354
Q

An active portfolio strategy is based on which of the following premises?

A) The investor’s ability to obtain public information

B) The portfolio manager’s access to corporate management

C) The stock market is efficient

D) The stock market is inefficient

A

D) Explanation
If the market is efficient, then a buy-and-hold strategy would be best. Only if the market is inefficient is it worth the costs involved in active management to attempt to generate superior returns.

LO 8.3.2

355
Q

As investors’ required rates of return decrease due to a decrease in interest rates caused by expectations of lower inflation rates, the stock market and investors can be expected to react in which of the following ways?

A) The value of stocks will decrease, and investors will avoid them because a higher required rate of return reduces the value of future stock benefits.

B) The value of stocks will increase, but investors will avoid them because they expect a market correction.

C) The value of stocks will decrease, and investors will buy them because the decrease represents the bottom of the market cycle.

D) The value of stocks will increase, and investors will buy them because the lower required return increases the present value of future stock benefits.

A

D) Explanation
As an investor’s required rate of return decreases, the present value of future benefits of the stock increases, and therefore the value of the stock increases. Investors will probably buy stocks because they perceive that interest rates are stabilizing.

LO 8.5.1

356
Q

Which of the following are true statements about the capital asset pricing model (CAPM)?

I. The security market line (SML) determines which investment is optimal.

II. Beta is used as a measure of risk on the capital market line (CML).

III. Systematic risk is the relevant risk in a diversified portfolio under the CAPM.

IV. CAPM determines the investor’s required rate of return for a given risky investment.

A) II, III, and IV

B) II and IV

C) III and IV

D) I and III

A

C) Explanation
Options III and IV are correct because beta, used in the CAPM formula to compute required return, is a measure of systematic risk. Option I is incorrect because the SML plots only risk/return relationships; it does not determine which is optimal. Option II is incorrect because beta is used on the SML, and standard deviation on the CML.

LO 8.2.1

357
Q

Assume that the economic forecast for the coming year is expected to be one of increasing inflation and interest rates. The GDP is expected to be strong. Which of the following types of investments would be advisable for the coming year and why?

I. Liquid investments, such as money market funds and short-term securities, to allow the investor flexibility to reinvest as rates increase

II. Long-term debt, such as 20-year government bonds, to lock in current interest rates

III. Stock in public utilities and durable goods firms, since they benefit from a rising interest rate environment

IV. Tangible assets, such as real estate and gold, to keep pace with the rate of inflation

A) I and IV

B) I, II, and III

C) II and IV

D) I, III, and IV

A

A) Explanation
Long-term bonds decrease in value in a rising interest rate environment. Stock in public utilities and durable goods firms does not benefit from a rising interest rate environment.

LO 8.5.1

358
Q

An active approach to portfolio management is more likely to reward investors in which of the following asset classes?

I. Emerging market equities
II. U.S. large-cap stocks
III. U.S. small-cap stocks
IV. European large-cap stocks

A) I and III

B) I only

C) I and II

D) I, III, and IV

A

A) Explanation
Investors who follow an active approach to portfolio management will benefit most from a portfolio containing emerging market equities and U.S. small-cap stocks, which tend to be more volatile investments.

LO 8.5.2

359
Q

Which statement regarding the concepts of modern portfolio theory (MPT) is NOT correct?

A) Markowitz used risk (as measured by beta) and expected return as the basis for determining appropriate assets or portfolios.

B) An infinite number of portfolios exist on the efficient frontier.

C) Indifference curves represent the risk-reward trade-off that investors are willing to make.

D) For any given level of risk, investors prefer higher returns to lower returns.

A

Explanation
A) The answer is Markowitz used risk (as measured by beta) and expected return as the basis for determining appropriate assets or portfolios. Harry Markowitz’s theory uses standard deviation as a measure of portfolio risk.

LO 8.1.1

360
Q

Indifference curves, which represent the risk-reward trade-off that the investor is willing to make, will

I. cross the efficient frontier in two locations.
II. lie tangent to the efficient frontier.
III. will not intersect the efficient frontier.

A) II and III
B) I only
C) I and II
D) I, II, and III

A

D) Explanation
The answer is I, II, and III. The portfolio that lies at the point of tangency of an indifference curve and the efficient frontier is the optimal portfolio for the investor.

LO 8.1.1

361
Q

Based on the efficient market hypothesis, select the CORRECT statements concerning an efficient market.

I. Security prices will adjust quickly to new information.
II. Security prices are rarely far from their justified price.
III. Security analysis will permit investors to consistently outperform the market.
IV. Current stock prices reflect all available information for a company.

A) I, II, and IV
B) I, II, and III
C) I and IV
D) III only

A

A)Explanation
The answer is I, II, and IV. Efficient markets will not allow investors to consistently outperform the market.

LO 8.1.1

362
Q

Which of the following statements regarding indifference curves is CORRECT?

A) For risk-averse investors, show decreasing returns for increases in risk
B) Reflect happier investors when the curve is lower
C) Represent all points where an investor is equally satisfied with the risk/return trade-off
D) Represent the return patterns of two or more equally attractive stocks

A

Explanation
C) The answer is represent all points where an investor is equally satisfied with the risk/return trade-off. Indifference curves express an individual’s preference regarding two items - risk and return. The more risk averse an investor is, the steeper the slope is for that investor’s indifference curve. Conversely, the less risk averse an investor is, the flatter the slope for that investor’s indifference curve.

LO 8.1.1

363
Q

Which of the following statements regarding the efficient frontier is CORRECT?

A) Combines all assets that can be held in a portfolio
B) Shows all portfolios that offer the highest expected returns for each level of risk
C) Represents the assets or asset combinations with the least return for the given level of risk
D) Represents an inferior set of assets or asset combinations available to investors

A

B) Explanation
The answer is shows all portfolios that offer the highest expected returns for each level of risk. Efficient frontiers take all of the possible combinations of assets that can be held in a portfolio into account in order to represent the set of portfolios that offer the highest expected return for a given amount of risk.

LO 8.1.1

364
Q

Identify which of the following statements pertaining to the capital market line (CML) is CORRECT.

I. Lending portfolio—As the investor proceeds back along the CML toward the risk-free rate of return, he is becoming conservative in making investments and is investing more in risk-free government securities

II. Borrowing portfolio—As the investor proceeds along the CML, he is becoming more aggressive in making investments, including making use of margin and leverage

A) Both I and II
B) Neither I nor II
C) I only
D) II only

A

A) Explanation
The answer is both I and II. The development of the capital market line allows investors to see that the risk and the potential return of various asset classes do increase along a relatively straight line.

LO 8.2.1

365
Q

Which of the following would cause the risk premium an investor expects to earn on a stock to increase when using the capital asset pricing model (CAPM)?

A) An increase in standard deviation
B) An increase in the correlation coefficient
C) An increase in beta
D) A decrease in covariance

A

Explanation
C) The answer is an increase in beta. All other factors remaining equal, an increase in beta will cause the risk premium to increase. In addition, the resultant expected rate of return will increase.

LO 8.2.1

366
Q

Identify which of the following regarding the capital asset pricing model (CAPM) is NOT correct.

A) The stock risk premium is the inducement necessary to entice the individual to invest in a particular stock.

B) CAPM uses standard deviation as a measure of market risk.

C) The market risk premium is the incentive required for the individual to invest in the securities market.

D) CAPM does not consider unsystematic risk.

A

B) Explanation
The answer is CAPM uses standard deviation as a measure of market risk. CAPM accounts for the impact of systematic risk (as measured by beta) only and does not take into consideration unsystematic risk, which is assumed to have been diversified away.

LO 8.2.1

367
Q

All of the following statements correctly describe capital market theory except

A) the security market line (SML) is the graphical depiction of the capital asset pricing model (CAPM).

B) beta is a measure of volatility, or relative unsystematic risk, for stock or portfolio returns.

C) the market risk premium is the difference between the expected return for the equities market and the risk-free rate of return.

D) the security market line (SML) depicts the trade-off between risk and expected return for all assets, whether individual securities, inefficient portfolios, or efficient portfolios.

A

B) Explanation
The answer is beta is a measure of volatility, or relative unsystematic risk, for stock or portfolio returns. Beta is a measure of relative systematic risk for stock or portfolio returns.

LO 8.2.1

368
Q

An investor considering investing in a particular security is more concerned with

A) historical return.
B) actual return.
C) expected return.
D) trailing P/E ratio of the stock.

A

Explanation
C) The answer is expected return. Expected return is what determines an asset’s value. The expected return must be greater than the investor’s required return to induce the investor to make the investment. Historical return is only important to the extent that it may impact future return.

LO 8.2.1

369
Q

The security market line (SML)

A) presents the relationship between a security’s return and the return of the market portfolio.

B) shows a security’s expected return as a function of its systematic risk.

C) indicates the market portfolio as the only optimal portfolio.

D) plots diversifiable risk on the horizontal (X) axis.

A

Explanation

B) The answer is shows a security’s expected return as a function of its systematic risk. The security market line (SML) shows the relationship between the rate of return and systematic risk (beta). Thus, the SML depicts a security’s expected return as a function of its systematic risk. The intersection between the efficient frontier and a line from the risk-free rate depicts all the optimal portfolios composed of a combination of the market portfolio and the risk-free asset. The market portfolio is the only optimal portfolio comprised solely of risky securities.

LO 8.2.1

370
Q

Which of the following statements regarding the capital market line (CML) is CORRECT?

A) Describes the required return of individual stocks

B) Uses beta as a risk measure

C) Is not useful for diversified portfolios

D) Provides a direct relationship between the risk and return for a well-diversified portfolio

A

Explanation

D) The answer is provides a direct relationship between the risk and return for a well-diversified portfolio. The capital market line (CML) graphically depicts the relationship of risk and return for efficient well-diversified portfolios. The CML uses standard deviation as a risk measure.

LO 8.2.1

371
Q

Identify the efficient market hypothesis that suggests an investor can achieve above-market returns by only utilizing insider information.

A) Weak
B) Semistrong
C) All of these forms
D) Strong

A

B) Explanation
The answer is semistrong. The semistrong form suggests that fundamental analysis is of no value and only through the use of insider information can an investor achieve above-market returns.

LO 8.3.1

372
Q

Which of the following statements about the efficient market hypothesis (EMH) and associated anomalies are CORRECT?

I. An investor purchasing a high price-to-earnings (P/E) ratio is exploiting the P/E effect anomaly.

II. An investor studying annual reports and analysts’ reports in his stock selection process believes that markets are weak-form efficient.

III. An investor who buys the securities of firms that are not followed by many analysts is trying to benefit from the neglected-firm effect.

IV. An investor who befriends the chauffeur of a firm’s CEO to solicit information about the firm’s plans before making investment decisions believes the markets are strong-form efficient.

A) I and II
B) II and III
C) III and IV
D) I and III

A

B) Explanation
The answer is II and III. The P/E effect suggests that portfolios consisting of stocks with low price-to-earnings ratios have higher average returns than do portfolios consisting of stocks with high P/E ratios. Strong-form market efficiency suggests that all public and private information is included in market prices. A person who solicits private information believes that it is possible to profit by making trading decisions based on private information and does not believe that the markets are efficient in the strong form. Weak-form efficiency suggests that all historical price and volume information is included in stock prices but that gains may be made by analyzing other publicly available information. An investor studying annual reports and analysts’ reports to make stock selections indicates that the person is conducting fundamental analysis, because the investor believes that the markets are weak-form efficient.

LO 8.3.1

373
Q

The semistrong form of the efficient market hypothesis states that current market prices reflect all available information on the history of prices.
all publicly available information concerning a company.

A) Both I and II
B) I only
C) Neither I nor II
D) II only

A

A) Explanation
The answer is both I and II. The semistrong form states that all publicly available information, including past stock price history, is reflected in current stock prices.

374
Q

An individual who is a proponent of the efficient market hypothesis (EMH) will likely invest in which of the following?

A) Sector mutual funds
B) Growth mutual funds
C) Index funds
D) Balanced mutual funds

A

C) Explanation
The answer is index funds. An individual who believes in the EMH will likely invest in index funds. Inherent in this strategy is a belief that an investor cannot outperform the market with active portfolio management techniques. The remaining choices all incorporate an active portfolio management philosophy.

LO 8.3.2

375
Q

Which statement concerning an active portfolio management strategy is CORRECT?

A) An actively managed portfolio has low total transaction costs.

B) The goal of active portfolio management is to earn a return that exceeds the risk-adjusted return of a passive benchmark portfolio.

C) The key to success for an actively managed portfolio is to minimize trading activity.

D) An actively managed portfolio has lower risk than a passive benchmark portfolio in most cases.

A

B) Explanation
The answer is the goal of active portfolio management is to earn a return that exceeds the risk-adjusted return of a passive benchmark portfolio. This is net of transaction costs. Such a strategy also involves higher transaction costs and, usually, risk.

LO 8.3.2

376
Q

Which of the following statements correctly distinguishes an investor who practices indexing?

I. The investor purchases index mutual funds.

II. The investor is practicing an active form of portfolio management.

III. The investor is attempting to beat the market (i.e., S&P 500).

IV. Indexing and purchasing index funds tends to exhibit low administrative costs and a low turnover of existing assets.

A) II and III
B) I, II, and III
C) II, III, and IV
D) I and IV

A

Explanation
D) The answer is I and IV. In actuality, the investor is following a passive portfolio management style. The purpose of indexing is not to beat the market, but merely match its long-term performance.

LO 8.3.2

377
Q

Nathan is concerned that he and his wife may not have enough money for retirement. When meeting with their financial planner, the planner used a Monte Carlo simulation in producing their retirement plan. Identify which of the following statements regarding a Monte Carlo simulation is NOT correct.

A) With this analysis, the user is attempting to predict the variability of an option contract.

B) Monte Carlo analysis uses a random number generator to provide an output with specific probabilities of outcomes.

C) In a Monte Carlo simulation, each of the variables is also given a probability distribution to allow for real-world uncertainty.

D) Small changes in the projected rate of return will make dramatic differences in the outcome.

A

Explanation
A) The answer is with this analysis, the user is attempting to predict the variability of an option contract. A Monte Carlo simulation provides insight into the range of outcomes. With this analysis, the user gets a best-case and a worst-case scenario.

LO 8.4.1

378
Q

Which of the following is a written document that sets forth a client’s objectives, sets limitations on the portfolio manager, provides guidance to the portfolio manager, and provides a means for evaluating performance?

A) Risk-profile questionnaire
B) Financial planning disclosure
C) New account form
D) Investment policy statement

A

D) Explanation
The answer is investment policy statement. The investment policy statement sets forth a client’s objectives, sets limitations on the portfolio manager, provides guidance to the portfolio manager, and provides a means for evaluating performance.

LO 8.4.1

379
Q

An effective investment policy statement

I. allows the investment manager to determine which client assets are income producing and which assets are personal use assets.

II. sets forth the client’s investment objectives.

III. places investment limitations on the investment manager.

IV. provides a means for evaluating investment performance.

A) I and IV
B) II, III, and IV
C) II and III
D) I, II, III, and IV

A

B) Explanation
The answer is II, III, and IV. The statement of financial position provides the planner with a list of assets stated at fair market value and any corresponding debt. Subsequently, the planner can analyze income-producing assets to determine whether they are generating income or growth at optimum levels or if there might be better alternatives for these assets.

LO 8.4.1

380
Q

All of the following affect an investor’s risk tolerance except

A) tax bracket.
B) family situation.
C) years of experience with investing in the markets.
D) investment time horizon.

A

A) Explanation
The answer is tax bracket. Tax concerns play an important role in investment planning; however, these constitute an investment constraint, not an investment objective (i.e., risk tolerance).

LO 8.4.1

381
Q

Which of the following is NOT likely to be an advantage of a valid investment policy statement?

A) Provides for short-term strategy shifts in response to short-term dramatic value declines

B) Promotes long-term discipline in investment decisions

C) Identifies and documents investment objectives and constraints

D) Allows for a continual dynamic process in meeting investor objectives

A

A) Explanation
The answer is provides for short-term strategy shifts in response to short-term dramatic value declines. The investment policy statement does not provide for shifts in strategy due to short-term value declines.

LO 8.4.1

382
Q

Which of the following statements about the importance of risk and return in the investment objective is least accurate?

A) The return objective may be stated in dollar amounts even if the risk objective is stated in percentages.

B) Expressing investment goals in terms of risk is more appropriate than expressing goals in terms of return.

C) The return and risk objectives have to be consistent with reasonable capital market expectations, as well as the client constraints.

D) The investor’s risk tolerance is likely to determine what level of return will be feasible.

A

Explanation
B) The answer is expressing investment goals in terms of risk is more appropriate than expressing goals in terms of return. Expressing investment goals in terms of risk is not more appropriate than expressing goals in terms of return. The investment objectives should be stated in terms of both risk and return. Risk tolerance will likely help determine what level of expected return is feasible.

LO 8.5.1

383
Q

In order to do an effective job of investment counseling, the investment adviser should examine and review the client’s

I. financial goals.
II. risk tolerance and risk exposure.
III. tax situation.
IV. liquidity and marketability needs.

A) I, II, III, and IV
B) III and IV
C) I, II, and IV
D) I and II

A

A) Explanation
The answer is I, II, III, and IV. In order to do an effective job of investment counseling, the adviser should examine and review the client’s financial goals, risk tolerance and risk exposure, tax situation, liquidity and marketability needs, and financial statements.

LO 8.5.1

384
Q

Choose the statement that is NOT correct regarding strategic asset allocation.

A) Once the allocation is determined, it typically remains constant until some life-changing event occurs.

B) Periodic rebalancing is necessary to bring the portfolio back in line with the strategic mix.

C) The purpose of strategic asset allocation is to determine an appropriate allocation based on the client’s long-term financial goals and risk tolerance.

D) Changing market conditions play a large role in determining the proper asset allocation.

A

D) Explanation
The answer is changing market conditions play a large role in determining the proper asset allocation. With strategic asset allocation, periodic rebalancing is necessary to bring the portfolio back in line with the strategic mix, but changing market conditions play a very small role in determining the actual mix of the portfolio.

LO 8.5.2

385
Q

When formulating a financial plan, a client will need to be informed of the various investments that may be used in the implementation phase of the plan. Therefore, an investment policy statement, which includes an evaluation of the possible investment strategies, should accompany the financial plan. Identify which of the following is NOT a characteristic of the investment policy statement.

A) A written document that sets forth the client’s objectives

B) Gives guidance to the investment manager

C) Allows the investment manager to charge any fee possible to reach the client’s financial goals

D) Provides a means for evaluating investment performance

A

Explanation
C) The answer is allows the investment manager to charge any fee possible to reach the client’s financial goals. An investment policy statement may or may not address the fees and commissions due to the investment manager for handling the investment process for the client. However, the investment manager is restricted by contract on how the fee charged to a client for services rendered is calculated.

LO 8.5.2

386
Q

While managing his portfolio, James’s investment adviser attempts to take advantage of perceived market inefficiencies. His investment adviser is not concerned with James’s long-term goals; rather, the interest lies in continuously changing the investment mix to take advantage of overall investor sentiment. Based on this information, choose the type of portfolio management style that the investment adviser is using to manage James’s money.

A) Strategic asset allocation

B) Portfolio ratio analysis

C) Tactical asset allocation

D) Buy and hold

A

C) Explanation
The answer is tactical asset allocation. Tactical asset allocation continuously adjusts the asset allocation in an attempt to take advantage of changing market conditions.

LO 8.5.2

387
Q

Which of the following statements concerning portfolio selection is CORRECT?

I. An alternative way to use the Markowitz model as a selection technique is to think in terms of asset classes, such as domestic stocks, foreign stocks of industrialized countries, stocks of emerging markets, and bonds.

II. The asset allocation decision refers to the allocation of portfolio assets to broad asset classes, such as cash equivalents, bonds, and equities.

A)I only
B) II only
C) Both I and II
D) Neither I nor II

A

C) Explanation
The answer is both I and II. Statements I and II are both correct. Asset allocation is the process of apportioning an investor’s wealth among different countries and asset classes for investment purposes.

LO 8.5.2

388
Q

Which of the following statements regarding asset allocation is CORRECT?

I. Asset allocation is the main determinant of a portfolio’s total return.

II. The purpose of strategic asset allocation is to determine an appropriate allocation based on the long-term financial goals of the client.

A) I only
B) Both I and II
C) II only
D) Neither I nor II

A

B) Explanation
The answer is both I and II. Both of these statements are correct.

LO 8.5.2

389
Q

From Case Study:
Frank has become very interested in the stock market and enjoys spending his spare time researching companies in the medical field. He believes studying and analyzing the industry, combined with his advanced exposure to trends and new innovations in medicine, will give him an advantage in achieving superior performance in medical stock investment opportunities. Choose the form of the efficient market hypothesis (EMH), if any, that Frank is considered subscribing to.

A) Weak form
B) Strong form
C) None of these
D) Semistrong form

A

Explanation
The answer is weak form. Frank is subscribing to the weak form of the EMH because he believes fundamental analysis and insider information will yield superior performance.

LO 9.2.2

390
Q

If held to maturity, which of the debt securities in Julie’s brokerage account provides the highest yield to maturity (YTM)?

A) The 30-year, $50,000, zero-coupon Cooper Co. bond
B) The YTM is the same for each debt security if held to maturity
C) The 25-year, $15,000, 6% coupon U.S. Treasury bond
D) The 5-year, $10,000, 7.5% coupon U.S. Treasury note

A

Explanation
A) The answer is the 30-year, $50,000, zero-coupon Cooper Co. bond. If held to maturity, this bond provides the highest YTM. Calculations are shown using the HP 10bII+.

25-year, $15,000, 6% coupon U.S. Treasury bond:

13,138.64 [+/–] [PV]

15,000 [FV]

50 [N]

450 [PMT]

Solve for [I/YR]

3.5321 (semiannual) × 2 = 7.06% YTM

30-year, $50,000, zero-coupon Cooper Co. bond:

4,093.40 [+/–] [PV]

50,000 [FV]

60 [N]

0 [PMT]

Solve for [I/YR]

4.2593 (semiannual) × 2 = 8.52% YTM

5-year, $10,000, 7.5% coupon U.S. Treasury note:

10,351.18 [+/–] [PV]

10,000 [FV]

10 [N]

375 [PMT]

Solve for [I/YR]

3.3313 (semiannual) × 2 = 6.66% YTM

LO 9.2.1

391
Q

Frank has discovered he is receiving a 10% stock dividend from Stock B in his investment portfolio. Which of the following statements regarding the stock dividend is CORRECT?

A) The stock dividend is taxable as ordinary income.

B) As a result of the stock dividend, Frank’s basis per share in Stock B is $78.62.

C) As a result of the stock dividend, Frank will own 1,695 shares of Stock B, with a FMV of $127,600.

D) As a result of the stock dividend, the fair market value (FMV) per share of Stock B will be approximately $72.73.

A

Explanation

D) The answer is as a result of the stock dividend, the fair market value (FMV) per share of Stock B will be approximately $72.73.

A stock dividend is generally not taxable. As a result of the stock dividend, Frank will own 1,595 shares of Stock B, with a FMV of $116,000. The FMV per share and Frank’s basis per share are both reduced proportionately to reflect the stock dividend.
FMV: $116,000 ÷ 1,595 = $72.73;
Basis: $114,000 ÷ 1,595 = $71.47.

LO 9.1.2

392
Q

From Case Study:
Because Julie considers herself to be a moderate risk investor, which of the following stocks should she choose for her investment portfolio?

A) XYZ stock with a beta of 2.16
B) ABC stock exhibiting a platykurtic distribution
C) GHI stock with a high standard deviation
D) JEM stock with a beta of 0.98

A

Explanation

D) The answer is JEM stock with a beta of 0.98. Because she considers herself to be a moderate risk investor, Julie would prefer a stock with a low beta. Investments exhibiting a platykurtic distribution have more observations dispersed further from the mean, resulting in a higher standard deviation.

LO 9.2.1

393
Q

From Case Study:

Regarding the financial planning engagement you have with the Edmonds, which of the following statements is CORRECT?

You must communicate to the Edmonds any limitations on the scope of the engagement.
Your recommendations for the Edmonds must be written and prepared in a clear, understandable manner.

A) II only
B) Both I and II
C) Neither I nor II
D) I only

A

B) Explanation
The answer is both I and II. You must communicate any limitations on the scope of the engagement and your recommendations must be written and prepared in a clear, understandable manner.

LO 9.3.2

394
Q

Which of the following statements regarding unit investment trusts (UITs) is NOT correct?

I. At the maturity date of the portfolio, the securities are generally liquidated, and the proceeds are distributed to the investors or trust beneficiaries.

II. UITs are actively managed, as portfolio managers typically attempt to match the return of a stated index.

III. Units in a UIT are priced in the secondary market at a premium or discount to NAV.

IV. The majority of UIT offerings are listed on the major stock exchanges.

A) I and III
B) I only
C) II and IV
D) II, III, and IV

A

D) Explanation
The answer is II, III, and IV. UITs are considered unmanaged or passively managed because the initial securities (usually bonds) included in the portfolio are typically held until maturity. UIT units are sold at NAV plus a commission. Units are sold in the secondary market but not on the major exchanges.

LO 3.3.1

395
Q

Which of the following combinations will result in a bond with the greatest price volatility?

A) High coupon and short maturity
B) Low coupon and long maturity
C) Low coupon and short maturity
D) High coupon and long maturity

A

Explanation
B) The answer is low coupon and long maturity. Price volatility is measured by duration. Duration is inversely related to the bond’s coupon rate and directly related to the bond’s term to maturity.

LO 4.1.2

396
Q

All of the following correctly explain advantages of investing in real estate investment trusts (REITs) except

A) liquidity.
B) diversification.
C) professional management.
D) tax deferral.

A

D) Explanation
The answer is tax deferral. Tax deferral is not an advantage of investing in REITs. REITs are professionally managed assets investing in a diversified portfolio of real estate holdings. Many REITs trade on the exchanges and over the counter, thereby providing investors with liquidity.

LO 5.1.1

397
Q

Assume the nominal return on 30-year U.S. T-bonds is 6.5%, and the inflation rate is 1.75%. Which of the following is the real rate of return on the T-bonds?

A) 3.71%
B) 4.67%
C) 8.36%
D) 4.75%

A

Explanation
B) The answer is 4.67%.
The real rate of return is 4.67%, calculated as follows:
{[(1 + 0.065) ÷ (1 + 0.0175)] –1} × 100 = 0.0467, or 4.67%.

LO 7.1.1

398
Q

According to Markowitz, an investor’s optimal portfolio is determined when the investor’s

A) highest indifference curve is tangent to the efficient frontier.
B) indifference curve meets the efficient frontier.
C) lowest indifference curve is tangent to the efficient frontier.
D) indifference curve crosses the efficient frontier.

A

A) Explanation
The answer is highest indifference curve is tangent to the efficient frontier. The optimal portfolio for an investor is determined as the point when the investor’s highest indifference curve is tangent to the efficient frontier.

LO 8.1.1

399
Q

Based on the following information, which of the following is the expected rate of return for Softco Corporation?

Stock’s beta 0.80
Forecasted market rate of return 15%
Risk-free rate of return 6.5%

A) 17.20%
B) 8.50%
C) 6.80%
D) 13.30%

LO 8.2.1

A

Explanation

D) The answer is 13.30%.

Using the capital asset pricing model, the expected rate of return is 13.30%
[6.5% + (15% – 6.5%)0.80].

400
Q

To which of the following financial intermediaries would a business look for startup capital?

I. Commercial banks
II. Credit unions
III. Investment bankers
IV. Mutual funds

A) III and IV
B) II and III
C) I, II, and III
D) I and III

A

D) Explanation
The answer is I and III. Startup capital is typically available at commercial banks and from investment bankers. It is not available through credit unions, which typically provide only consumer loans. Mutual funds invest in stocks or bonds on secondary markets.

LO 1.1.1

401
Q

Advantages of investing in exchange-traded funds (ETFs) include all of the following except

A) tax efficiency.
B) daily pricing.
C) low expense ratios.
D) ability to sell short.

A

B) Explanation
The answer is daily pricing. ETFs are continually priced during a trading day, just like stocks.

LO 3.4.1

402
Q

Nancy bought 50 shares of ABC stock for $50 per share. She made additional purchases at the end of each of the following years:

Year 1: 10 shares at $52 per share

Year 2: 10 shares at $53 per share

Year 3: 10 shares at $45 per share

ABC stock has not paid any dividends during her holding period. At the end of Year 4, the stock is trading for $55 per share. Which of the following is Nancy’s return over the past four years on ABC stock (assume she sells the stock for the current trading price)?

A) 2.95%
B) 3.25%
C) 4.65%
D) 5.00%

A

A)
Explanation
The answer is 2.95%. The dollar-weighted return is calculated as follows:

CF0 (2,500) [50 × $50]

CF1 (520) [10 × $52]

CF2 (530) [10 × $53]

CF3 (450) [10 × $45]

CF4 4,400 [80 × $55]

Solve for IRR/YR = 2.95%

LO 7.1.1

403
Q

Assume an investor has purchased a bond with the following characteristics:

Seven years to maturity
$1,000 face value
6% annual coupon (paid semiannually)
8.43% current yield
$711.74 current market price
Which of the following is the bond's yield to maturity?

A) 9.83%
B) 12.25%
C) 6.13%
D) 4.92%

A

Explanation

B) The answer is 12.25%. The bond’s yield to maturity is calculated as follows:

PV = –711.74

FV = 1,000

PMT = 6% × $1,000 ÷ 2 = 30

N = 7 × 2 = 14 (number of periods until maturity)

I/YR = 6.1254 × 2 = 12.2508, or 12.25%

LO 7.4.1

404
Q

The Pratts are considering purchasing JKL stock based on a stock tip from their neighbor, Alicia. Alicia has provided them with the following information:

Current dividend: $2.50
Expected dividend growth rate for years 1 and 2: 8%
Expected dividend growth rate for years 3+: 10%
Based on this information, what is the intrinsic value of JKL stock if the Pratts have a required rate of return for this investment of 14%?

A) $123.59
B) $66.37
C) $67.50
D) $41.67

A

B) Explanation
The answer is $66.37. Using the multistage growth dividend discount model, calculate JKL stock’s intrinsic value:

Step 1: Compute the value of each future dividend until the growth rate stabilizes (Years 1–2).

D1 = $2.50 × 1.08 = $2.70

D2 = $2.70 × 1.08 = $2.92

Step 2: Use the constant growth dividend discount model to compute the remaining intrinsic value of the stock at the beginning of the year when the dividend growth rate stabilizes (Year 3).

D3 = $2.92 × 1.10 = $3.21

V = $3.21 ÷ (0.14 – 0.10) = $80.25

Step 3: Use the uneven cash flow method to solve for the net present (intrinsic) value of the stock.

CF0= $0

CF1= $2.70

CF2 = $2.92 + $80.25

I/YR= 14%

Solve for NPV = $66.37

LO 9.2.2

405
Q

Which of the following incomes is not taxed under Social Security self-employment tax?

I. Rental real estate income
II. Small part-time repair shop income
III. Shareholder’s share of S corporation’s income in excess of salary
IV. Income of an individual working as an independent contractor

A) I, II, III, and IV

B) I, II, and III

C) II and IV

D) I and III

A

Achieved
D) Explanation
Income from rental property is not subject to Social Security tax, nor are dividends to S corporation shareholders/employees as long as the employees’ compensation is considered reasonable.

LO 5.3.1

406
Q

Christopher, who has an interest in multiple businesses, has the following income for 2018:

Net Schedule C income: $80,000
Dividends and interest: $11,000
General partnership K-1 income: $10,000
S corporation K-1 income: $29,000
What is Christopher's total self-employment tax for 2018 (round up to the nearest dollar)?

A) $6,359

B) $12,717

C) $11,304

D) $16,955

A

B) Explanation
Christopher’s self-employment income is $90,000. The dividends, interest, and K-1 distributions from the S corporation are not self-employment income. The self-employment tax is calculated as follows: $90,000 × 0.9235 = $83,115 net self-employment earnings. Total self-employment tax is $12,717 ($83,115 × 0.1530).

LO 5.3.1

407
Q

ABC Corporation has the following items of income and expense:

Taxable income $300,000
Federal income tax $80,000
Dividends paid (all in current year) $40,000
Accumulated earnings and profits at end of preceding tax year $160,000
Assume that XYZ Corporation is not a personal service corporation and cannot establish a valid business purpose for the excess accumulations. What is the amount, if any, of the accumulated earnings tax payable?

A) $6,000

B) $0

C) $26,000

D) $18,000

A
D) Explanation
Taxable income	$300,000
Federal income tax	(80,000)
Dividends paid (all in current year)	(40,000)
Accumulated earnings credit ($250,000 – $160,000)1	(90,000)
Accumulated taxable income	$90,000
20%
Accumulated earnings tax	$18,000

1The accumulated earnings credit is the $250,000 accumulation limit minus accumulated earnings and profits at the end of the preceding tax year.

LO 5.2.1

408
Q

Which of the following statements best describes a weakness of the sole proprietorship form of business?

A) The business is in constant danger of bankruptcy.

B) The business depends solely on the owner.

C) The owner has too many partners.

D) The owner has no control.

A

B) Explanation
A sole proprietorship is owned by one individual who is personally liable for the business and all its operations. Therefore, the business depends solely on the owner.

LO 5.2.3

409
Q

Jessica is self-employed selling insurance. She uses a bedroom in her apartment as an office about 60% of the time. The square footage of the bedroom represents 15% of the overall space in the apartment. She also uses the bedroom to store personal items and, occasionally, as a bedroom for guests. Jessica’s annual rental payments this calendar year were $7,500 and the annual utility costs were $1,200. How much of a deduction can Jessica take as an office expense for this tax year?

A) $1,125

B) $0

C) $567

D) $675

A

B) Explanation
No deduction is allowed because the office is not used regularly and exclusively as a principal place of business for Jessica’s insurance business, as required by the Internal Revenue Code to qualify as a deductible office expense.

LO 5.4.1

410
Q

On which Schedule of the Form 1040 would a sole proprietor of an appliance repair business report self-employment income?

A) Schedule C

B) Schedule E

C) Schedule A

D) Schedule F

A

A) Explanation
Schedule C of Form 1040 (Profit or Loss From Business) would be used to report the income from a sole proprietorship. If it were a farm, the taxpayer would use Schedule F.

LO 5.2.2

411
Q

All of the following statements regarding accounting methods are correct except

A) under the hybrid method of accounting, a taxpayer may account for some items using the accrual method and some items using the cash method.

B) individuals and sole proprietorships may use the cash method of accounting.

C) the cash method of accounting is generally required when the sale of merchandise is an income-producing factor.

D) once an accounting method has been adopted, it cannot be changed without approval from the IRS.

A

C) Explanation
The accrual method of accounting is required when the sale of merchandise is an income-producing factor.

LO 5.1.1

412
Q

Yetunde has been selling sports memorabilia online for six years. It is not her primary employment, but she has been doing well with sales. While Yetunde will be itemizing her deductions this year, she believes she can report her income and expenses on Schedule C. Her gross sales from the sports memorabilia this year are $55,000. Her total expenses are $37,600, including cost of goods sold of $28,000. Yetunde has kept detailed records since she began selling memorabilia and can track her profit and loss for each year. In Year 1, she had a loss of $5,000; in Year 2, there was a loss of $2,000; in Year 3, she had a profit of $9,000; in Year 4, Yetunde had another loss of only $500; and Year 5 was a good year with a profit of $12,000. How should Yetunde report her sports memorabilia business for Year 6?

A) Yetunde can use Schedule C to report the expenses of her business but must report the $55,000 as other income on the front of the Form 1040.

B) Because this is not Yetunde’s primary employment, the business is deemed a hobby and she should use hobby rules to report the income and expenses.

C) Only cost of goods sold can be expensed on Schedule C; the rest of the business expenses are miscellaneous itemized deductions subject to the 2%-of-AGI floor.

D) Because Yetunde has a profit from the business in three of the last five years, the business is presumed not to be a hobby and she will report income and expenses using Schedule C.

A

D) Explanation
Because Yetunde had a profit in Years 3, 5, and 6, the hobby rules do not apply and Yetunde can use Schedule C to report income and expenses from her business.

LO 5.4.2

413
Q

To elect S corporation status for the current calendar year, which of the following must be done for a newly incorporated business?

I. Secure the consent of the board of directors of the corporation.

II. File the election (IRS Form 2553) before the 15th day of the 3rd month of the tax year the election is to take place.

III. Issue two classes of stock.

IV. File the election (IRS Form 2553) at any time before the end of the corporation’s tax year.
Elect S corporation status when filing the corporation’s initial tax return.

A) V only

B) II only

C) I, II, and IV

D) I, III, and IV

A
B) Explanation
IRS Form 2553—Election by a Small Business Corporation has to be filed by the 15th day of the 3rd month of the tax year. The shareholders must secure the consent of the IRS to be taxed as an S corporation. In general, only one class of stock is allowed in an S corporation.

LO 5.2.2

414
Q

Old Citywear, Inc., manufactures clothing in styles from the 1940s and 1950s. The company has been in existence four years and had the following taxable income or loss:

Tax Year	Taxable Income
2018	$5,000
2019	($25,000)
2020	$15,000
2021	$15,000

Which of the following statements regarding Old Citywear’s net operating loss in 2019 is CORRECT?

A) Old Citywear may carry forward the $25,000 of the NOL to 2020 and 2021 and carry forward any remaining loss indefinitely.

B) Old Citywear may carry back $5,000 of its 2019 NOL to 2018.

C) Because it cannot utilize all of the net operating loss in 2019 for 2018, Old Citywear must carry forward the entire $25,000 to a future tax year when it can be used in its entirety.

D) Old Citywear may carry back $3,000 of its 2019 NOL to 2018.

A

A) Explanation
NOLs cannot be carried back but can be carried forward indefinitely years following the loss year. The loss is limited in amount to 80% of the total of taxable income reported for the carryforward years. In this case, the original 2019 $25,000 loss was carried forward and applied to the 2020 taxable income, reducing it to $3,000. The NOL in 2019 is then applied to the 2021 taxable income of $15,000, reducing it to $3,000. The balance of the NOL ($1,000) is a carryforward.

LO 5.1.1

415
Q

Gordon, who is married and files jointly with his spouse, purchased Section 1244 stock for $110,000. Seven years later, Gordon sold his shares of the stock for $15,000. What is the character of the loss?

A) $100,000 Section 1244 loss against ordinary income in the year the stock is sold

B) $110,000 Section 1244 loss against ordinary income in the year the stock is sold at a loss

C) $95,000 Section 1244 loss against ordinary income in the year the stock is sold

D) $95,000 Section 1244 loss deducted against ordinary income at a rate of $3,000 annually as a capital loss beginning in the year the stock is sold

A

C) Explanation
Gordon and his spouse have a $95,000 Section 1244 loss ($110,000 purchase price − $15,000 sale price) against ordinary income in the year the stock is sold. If a married investor sells Section 1244 at a loss (or if the stock becomes worthless), up to $100,000 of the loss may be deducted as an ordinary loss and is not subject to the $3,000 loss limitation. Any amount over the $100,000 will be treated as a capital loss subject to the $3,000 annual limitation ($1,500 if married filing separately) under the capital loss rule.

LO 5.2.1

416
Q

Landry has been baking pies and cakes for several years that are very popular when he sells them locally. He works full-time in the oil field and baking has become a hobby he enjoys. He is meeting with his new financial planner, Richard, to review his plans for retirement in 10 years. He explains that he will continue his baking after retirement but not necessarily full time. Richard has asked for as much documentation as possible about his current and prior years’ sales and expenses for this hobby. Landry is confused about why it matters. What should Richard do for Landry as his planner?

I. Richard should review Landry’s previous income tax returns to ascertain if the hobby income was reported.

II. When reviewing the documentation for the hobby, Richard is also determining whether or not the hobby could be classified as a business.

III. Landry should file amended returns and deduct all of his hobby expenses from the hobby income before it is entered on his Form 1040 if Landry has not previously reported the hobby.

A) I and II

B) I only

C) II and III

D) I, II, and III

A

A) Explanation
Statement III is incorrect. If Landry has not previously reported the hobby income and expenses, Richard should advise him to file amended returns, reporting the income on the front of the Form 1040. Hobby expenses are not deductible except for allocated property taxes and mortgage interest.

LO 5.4.2

417
Q

Which of the following statements comparing a C corporation and a general partnership is CORRECT?

I. Both a C corporation and general partnership can have more than one owner.

II. Both a C corporation and general partnership are regarded as distinct entities for tax purposes.

III. Profits are divided equally in both business forms.
Neither a C corporation nor general partnership pays federal tax on income.

A) I only

B) II only

C) I and IV

D) I and III

A

A) Explanation
One similarity of a partnership and a C corporation is that they may both have more than one owner. However, only a C corporation is regarded as a distinct, separate entity for tax purposes. Partnerships are flow-through entities. C corporations may retain profits. Partnerships are not required to distribute profits equally. C corporations pay tax on income.

LO 5.2.3

418
Q

Tom is a self-employed accountant. He uses one room of his house exclusively and regularly as an office for the business. The square footage of the room comprises 10% of the total square footage of the home. Tom incurred the following expenses related to the home during the tax year:

Utilities	$1,500
Property tax	$600
Mortgage interest	$4,000
Insurance on home	$750
Maid service	$1,200
Misc. repairs	$950

If Tom’s office meets the tax law requirements of a qualified home office and the gross income from the business limitations on the deduction are ignored, what is the maximum business deduction Tom can take?

A) $805 plus a depreciation deduction based on 10% of the cost of the home

B) $780 plus a depreciation deduction based on 10% of the cost of the home

C) $900 plus a depreciation deduction based on 10% of the cost of the home

D) $900

A

C) Explanation
Tom can deduct 10% of all the expenses. The calculation is $9,000 × 10% = $900. A 10% depreciation deduction is also allowed based on the cost of the home. It should be noted that in order to take this deduction, Tom’s gross income from the business must be sufficient to cover all of his other operating expenses in addition to the home office deduction. The deduction for home office expenses cannot create a loss from the business for the taxpayer.

LO 5.4.1

419
Q

A business fails and the owners lose their investment in the unincorporated business, along with their personal residences, automobiles, and other personal property. What type of business entity does this suggest?

A) General partners in a general partnership

B) Preferred shareholders in a corporation

C) Limited partners in a limited partnership

D) Common shareholders in a corporation

A

A) Explanation
Multiple ownership of an unincorporated business is characteristic of a general partnership. More than one owner personally liable indicates a general partnership business form. Common shareholders, preferred shareholders, and limited partners do not have personal property at risk; only their investment in the company is at risk.

LO 5.2.3

420
Q

Which of the following statements regarding Section 1244 stock is CORRECT?

I. Stock issued in exchange for services rendered to the issuing corporation qualifies as Section 1244 stock.

II. An investor can obtain capital gain treatment if she makes a profit and ordinary income treatment (subject to limits) if she sells at a loss.

III. Single taxpayers are limited to an annual $50,000 ordinary income deduction.

IV. At the time of forming a regular or C corporation, the total capital contributions for stock to the corporation may not exceed $1 million in investor money.

A) I only

B) II, III, and IV

C) III only

D) I and II

A

B) Explanation
Statements II, III, and IV are correct. Statement I is incorrect. Stock issued in exchange for services rendered to the issuing corporation does not qualify as Section 1244 stock.

LO 5.2.1

421
Q

Jim is a single taxpayer. During the current year, he sold Section 1244 stock for $40,000. Jim had held the stock for three years. His basis in the stock was $130,000. What is the tax result from the sale of the stock?

A) Jim has an ordinary loss of $50,000 and a long-term capital loss of $40,000.

B) Jim has a long-term capital loss of $100,000.

C) Jim has an ordinary loss of $50,000 this year and a carryforward of ordinary loss of $40,000.

D) Jim has an ordinary loss of $100,000.

A

A) Explanation
The loss on the sale, exchange, or worthlessness of Section 1244 stock may be treated as an ordinary loss up to $100,000 annually on a joint return or $50,000 annually on any other return. Thus, as a single taxpayer, Jim has a $50,000 ordinary loss. Any excess loss is a capital loss—short- or long-term depending on the holding period.

LO 5.1.1

422
Q

For purposes of the accumulated earnings tax (AET), what amount may a regular corporation retain in accumulated earnings before having to prove a reasonable business need for the retained funds?

A) $250,000

B) $25,000

C) None of these

D) 15% of the current year’s earnings

A

A) Explanation
Every regular corporation can accumulate up to $250,000 in retained earnings without having to prove a reasonable business need.

LO 5.2.1

423
Q

Which of the following statements regarding calculating self-employment (SE) tax is CORRECT?

I. The method for calculating the deductible employer share of SE tax is to multiply the net SE income by 0.9235 × 0.0765 up to the taxable wage base of $137,700 in 2020.

II. The shortcut method for calculating total SE tax for SE income at or below the taxable wage base in 2020 is to simply multiply the amount of SE income by 0.1413 (0.9235 × 0.1530).

A) II only

B) I only

C) Neither I nor II

D) Both I and II

A

D) Explanation
Both statements are correct.

LO 5.3.1

424
Q

Which of the following factors may be considered a strength of the partnership form of business entity?

I. Ease of management
II. Ease of establishment
III. Lack of special taxes

A) II only

B) I only

C) II and III

D) I, II, and III

A

D) Explanation
All of these factors are strengths of the partnership form of business entity.

LO 5.2.3

425
Q

Carol owns and operates a retail electronics store with annual sales of approximately $2 million. The store also specializes in repairing computers and other small electronics. Approximately 30% of her sales are with extended credit terms.

What method of tax accounting may be used for Carol’s business?

I. The accrual method, because inventory is such a large component of the business

II. The cash method, because it provides flexibility in the timing of income and expenses

III. The hybrid method, because the business involves both inventory and service

IV. The installment sale method, to spread the gain over a longer time frame

A) I, II, III, and IV
B) I only
C) I, II, and III
D) III only

A

C) Explanation
The cash method may be used because average annual gross receipts are under $26 million for the prior tax years. (This limit was increased by the Tax Cuts and Jobs Act, or TCJA.) The accrual method may always be used. The hybrid method may be used because the business involves both inventory and service. The installment method may not be used, as dealers are not allowed to use the installment method.

LO 5.1.1

426
Q

Jena owns and operates a string of retail electronics stores with approximately $30 million of sales annually. Approximately 20% of her sales are with extended credit terms.

What method of tax accounting is most appropriate for Jena’s business?

A) The accrual method, because inventory is such a large component of the business

B) The hybrid method, because the business involves both inventory and service

C) The installment sale method, to spread the gain over a longer time frame

D) The cash method, because it provides flexibility in the timing of income and expenses

A

A) Explanation
The accrual method of accounting generally is mandatory when inventory constitutes a significant income-producing factor. Thus, the cash method is incorrect. The hybrid method is incorrect because there is no indication that service constitutes a significant portion of the business. Also, the installment sale method is not available for sales of inventory, or sales with revolving credit terms. With annual sales of $30 million, the accrual method exception does not apply. If she had annual sales of under $26 (2020) million, she could still use the cash method, even though she has inventory.

LO 5.1.1

427
Q

A small business with under $20 million in gross receipts and no inventory should most likely use which accounting method?

A) Accrual
B) Cash
C) LIFO
D) Hybrid

A

B) Explanation
A typical small business would use the cash method of accounting. Due to cost and complexity, the accrual and hybrid methods would NOT be recommended. A business is not required to use the accrual method until it has at least $26 million in gross receipts (2020). A business that has no inventory would not use LIFO.

LO 5.1.1

428
Q

Which of the following methods of accounting is mandatory for businesses that maintain inventory and have gross receipts in excess of $26 million?

A) IRS method of accounting

B) Accrual method of accounting

C) Hybrid method of accounting

D) Cash method of accounting

A

B) Explanation
Under the cash method of accounting, a taxpayer generally reports income when any cash is collected (or the constructive receipt income tax doctrine applies) and reports expenses when any cash payment is made. The accrual method is the conceptual opposite of the cash method and requires recognition of taxable income in the same tax year it is reported on the taxpayer’s financial statements; it is mandatory for businesses maintaining inventory and have gross receipts in excess of $26 million. The hybrid method of accounting is a combination of the accrual method and the cash method of accounting. There is no IRS method of accounting.

LO 5.1.1

429
Q

Which of the following is NOT an advantage of the cash basis method of accounting?

A) Taxpayers have more control over each year’s income and expenses.

B) Income is reported when it is received.

C) Constructive receipt serves to defer income.

D) Taxpayers can keep simple records.

A

C) Explanation
Constructive receipt, which serves to accelerate income, is not considered an advantage of the cash method of accounting.

LO 5.1.1

430
Q

Which one of the following types of investors derives the greatest tax benefit from investing in preferred stocks?

A) Corporate investors
B) Mutual funds investors
C) Nonprofit institutional investors
D) Government investors

A

A) Explanation
Because 50% of the preferred dividends received by a corporation are exempt from federal income taxes, a corporation gains a tax advantage. The government and nonprofit organizations pay no income taxes. Mutual funds are also exempt from taxation. (The Tax Cuts and Jobs Act, or TCJA, reduced the dividend-received deduction to 50%.)

LO 5.2.1

431
Q

Jim, a single taxpayer, sells Section 1244 stock for a $250,000 loss during the tax year. The stock had been held for two years. Which one of the following is CORRECT regarding the treatment of the loss?

A) $50,000 is treated as an ordinary loss; $200,000 is treated as a Section 1244 carryforward.

B) $100,000 is treated as an ordinary loss; $150,000 is treated as long-term capital loss.

C) $250,000 of the loss is fully deductible as a capital loss.

D) $50,000 is treated as an ordinary loss; $200,000 is treated as long-term capital loss.

A

Explanation
Loss on the sale of Section 1244 stock is deductible as an ordinary loss up to $100,000 per year on a jointly filed return, or $50,000 on any other return (which applies to Jim as a single taxpayer). Any Section 1244 loss in excess of the annual limits is treated as a capital loss—long term or short term, depending on the holding period. The $100,000/$150,000 option would be correct if Jim were a married taxpayer filing jointly.

LO 5.2.1

432
Q

Kappa Corporation has the following items of income and expense:

Taxable income: $310,000
Federal income tax paid: $80,000
Dividends paid: $20,000
Accumulated earnings and profits at the end of the preceding year: $90,000
Kappa is an engineering firm with 100% of the stock owned by its three employee-shareholders. The corporation cannot establish a valid business purpose for excess accumulations. How much accumulated earnings tax is payable by Kappa?

A) $42,000
B) $34,000
C) $46,000
D) $30,000

A

D) Explanation
The accumulated earnings tax applies to corporate accumulated earnings in excess of $250,000, generally, for which there is no valid business reason for accumulating the funds. For a personal service corporation (PSC), the accumulations limit is $150,000. Engineering is one of the personal services specifically listed for determining a PSC. Expenses otherwise not allowed in computing the corporate income tax are allowed in computing the accumulated earnings tax, as is summarized here. (Note that the accumulated earnings credit of $60,000 is the difference between the $150,000 safe harbor and the $90,000 of accumulations at the beginning of the year.) The accumulated earnings tax rate is 20%, the highest rate that may apply to qualified dividends.

Taxable income	$310,000
Federal income tax	(80,000)
Dividends paid	(20,000)
$210,000
Less: accumulated earnings credit	(60,000)
Excess accumulations	$150,000
× 20%
$30,000
LO 5.2.1
433
Q

Which of the following forms does a C corporation file?

A) Form 1065
B) Form 1120
C) Form 1120-C
D) Form 1120-S

A

B) Explanation
The C corporation files a tax return with the IRS—this is the Form 1120.

LO 5.2.1

434
Q

John, an accountant, is considering forming an S corporation for his practice. He will be the sole employee of the corporation. Which of the following statements accurately describes the income tax consequences of the proposed arrangement?

A) The corporation would not be considered a personal service corporation (PSC), and the income would be subject to John’s personal income tax rates.

B) The corporation would be considered a PSC, subject to a flat 21% tax rate.

C) The corporation would not be considered a PSC; therefore, the income would be subject to the graduated corporate income tax rates.

D) The corporation would be considered a PSC, subject to graduated tax rates.

A

A) Explanation
The personal service corporation (PSC) classification for a C corporation was effectively repealed with the Tax Cuts and Jobs Act (TCJA). S corporations do not have corporate income tax rates. As an S corporation, all income would flow through to John to be taxed at his individual rates.

LO 5.2.2

435
Q

Your client, Mary, is contemplating the formation of an S corporation for her small manufacturing business. What can you accurately tell Mary about forming an S corporation?

I. Mary would have limited liability.

II. The number of shareholders is limited to 100.

III. Mary’s death would require reorganization of the corporation.

IV. The corporation’s items of income, deductions, and tax credits would flow through to Mary’s income tax return.

A) III only
B) I, II, and IV
C)II and III
D) I and IV

A

B) Explanation
The S corporation form is similar to the partnership in operating as a conduit for tax purposes. It also is similar to a C corporation, since it features limited liability for shareholders. In addition, the death of a shareholder in an S corporation does not create a need to reorganize the business. Finally, unlike a C corporation, an S corporation may have no more than 100 shareholders.

LO 5.2.2

436
Q

Which of the following statements is CORRECT regarding a minority non-employee shareholder in an S corporation?

I. Reports income when the corporation has net income for a tax year

II. Votes for the board of directors at the annual shareholders meeting

III. Receives a K-1 annually to prepare a personal income tax return

IV. Reports on a personal income tax return a pro rata share of corporate profit or loss

A) I and III
B) II and IV
C) I, II, III, and IV
D) I, II, and III

A

C) Explanation
All of the statements are correct. The fact that the shareholder has a minority interest in the corporation has no bearing on the correct answers. The flow-through of a proportionate share of net income is reported on the K-1. The shareholder is allowed to vote for the board.

LO 5.2.2

437
Q

Which of the following businesses may be considered a specified service business (SSB) for purposes of the qualified business income (QBI) deduction?

A) A medical practice
B) A furniture manufacturer/wholesaler
C) An automobile repair shop
D) An engineering firm

A

A) Explanation
An SSB is any trade or business involving the performance of services in health, law, accounting, actuarial science, performing arts, consulting, athletics, financial services, brokerage services, or any trade or business where the principal asset of such trade or business is the reputation or skill of one or more of its employees; or, any trade or business that involves the performance of services that consist of investing and investment management, trading, or dealing in securities, partnership interests, or commodities. Thus, the medical practice would be an SSB, but none of the other businesses would. Engineering and architecture are specifically excluded from the definition of SSB.

LO 5.2.2

438
Q

Which of the following statements about S corporations is CORRECT?

A) S corporation status is automatic if there are fewer than 100 shareholders.

B) S corporations are prohibited from earning passive income.

C) S corporations are prohibited from having more than one class of stock.

D) S corporations may have nonresident aliens as shareholders.

A
C) Explanation
An S corporation may have only one class of stock, although differences in voting rights are allowed within that one class of stock. An S corporation may have no more than 100 shareholders; however, the corporation must elect S status by filing a Form 2553—the treatment is not automatic. An S corporation may have passive income, although excess passive income may trigger the sting tax if the corporation had been a C corporation with earnings & profits (E&P). Nonresident aliens may not be shareholders in an S corporation.

LO 5.2.2

439
Q

Alisha, a CFP® certificant and fee-only financial planner, has assisted Roger, a self-employed physician, in income tax return preparation and investment planning during the year. On which of the following schedules may Alisha’s fee be deductible by Roger on his federal income tax return?

I. Schedule A—itemized deductions
II. Schedule C—profit or loss from business
IIIl Schedule D—capital gains and losses

A) I, II, and III
B) I only
C) II only
D) I and II

A

C) Explanation
The investment planning fees are not deductible (the Tax Cuts and Jobs Act, or TCJA, repealed the Tier II miscellaneous itemized deductions). This expense may not be added to the basis of securities. The tax preparation fee is partly nondeductible (the portion attributable to the individual return), and the portion of the fee for completing the Schedule C is a business expense that would be deducted on the Schedule C.

LO 5.2.3

440
Q

Glen and Debbie both have significant net worth and are currently in the highest marginal income tax bracket. They have developed a process that allows them to neutralize toxic chemical waste. They want to form a business that will protect their net worth in case the business fails or it becomes involved in lawsuits. They expect the business to produce significant profits immediately, as they have agreements in the works with several large chemical companies. Also, they would like to share ownership with other family members as they get closer to retirement.

Which business form would be most appropriate for Glen and Debbie at this time?

A) Limited partnership
B) S corporation
C) General partnership
D) C corporation

A

D) Explanation
The S corporation (and the partnership entities) are conduits, and would cause the income to be added to their other significant individual income. The use of the C corporation would allow for protection from lawsuits or business failure. Also, the general partnership and the limited partnership both would cause a flow-through of the income, and the general partnership would not provide protection from personal liability. There is no indication that either party wants to be a general partner in a limited partnership. The use of a C corporation would be appropriate since it is a separate taxable entity, and the profits would not flow through. They could receive a reasonable salary, and the qualified dividends that are paid out would be subject to long-term capital gain rates.

LO 5.2.3

441
Q

Irwin, a young financial planner, operates his practice as an LLC. His LLC net income is $100,000, and the taxable income on his single return is $140,000. The LLC paid no wages, and has no depreciable property.

What is the amount of Irwin’s qualified business income (QBI) deduction?

A) $0, because he has no depreciable property
B) $28,000
C) $0, because he paid no wages
D) $20,000

A

D) Explanation
For a specified service business (SSB), if the taxpayer’s taxable income is below the phaseout range of $163,300 to $213,300 (single 2020), the QBI deduction is fully available. An SSB involves performance of services in the fields of health, law, accounting, actuarial science, performing arts, veterinary services, consulting, athletics, financial services, or brokerage services. For pass-through entities, the owner/taxpayer may be allowed a deduction equal to 20% of the lesser of the QBI, or 20% of the taxpayer’s taxable income (reduced by net capital gain). Thus, the available deduction is 20% of the QBI of $100,000, or $20,000.

LO 5.2.3

442
Q

Which of the following forms of business would file a Schedule C to report the income or loss from the business?

A) C corporation
B) Limited partnership
C) S corporation
D) Sole proprietorship

A

D) Explanation
The sole proprietorship is the business that files a Schedule C with the individual’s Form 1040. The limited partnership files a Form 1065, and the C corporation files a Form 1120. The S corporation files an 1120-S.

Note that a single-member LLC would also file a Schedule C, as it would be treated as a disregarded entity for income tax purposes.

LO 5.2.3

443
Q

In 2020, which of the following is a method in calculating total self-employment tax, where net income from self-employment is above the taxable wage base?

I. Calculate self-employment income; multiply by 0.9235 (1 − 0.0765); subtract the taxable wage base, and from that, multiply the excess over the taxable wage base by 2.9% (Medicare portion of the tax).
Multiply the taxable wage base by 15.3%, and add the results of the previous two steps together to arrive at the total self-employment tax.

II. Calculate self-employment income; multiply by 0.9235 (1 − 0.0765); and multiply the result by 15.3%.

III. Calculate self-employment income; multiply by 14.13%.

IV. Calculate self-employment income; multiply by 15.3%; multiply the excess over the taxable wage base by 2.9% (Medicare portion of the tax); and add the results of the previous two steps together to arrive at the total self-employment tax.

A) I only
B) II and III
C) I and II
D) I and IV

A

A) Explanation
The steps to calculate self-employment tax for 2020 where net income from self-employment is above the taxable wage base are as follows:

Step 1: Calculate self-employment income.

Step 2: Subtract 7.65% or multiply by 0.9235 (1 − 0.0765).

Step 3: From Step 2, subtract the taxable wage base and multiply the excess over the taxable wage base by 2.9% (Medicare portion of the tax).

Step 4: Multiply the taxable wage base by 15.3%.

Step 5: Add the results of Steps 3 and 4 together to arrive at the total self-employment tax.

LO 5.3.1

444
Q

What percentage, if any, of a taxpayer’s self-employment tax may be deductible as an adjustment to income?

A) 50%
B) 75%
C) 25%
D) 0%

A
A) Explanation
Only half (50%) of a taxpayer's self-employment tax liability is deductible as an adjustment to income.

LO 5.3.1

445
Q

What is Bobby’s total self-employment tax on his Schedule C net income of $50,000? Round your answer to the nearest dollar.

A) $3,533
B) $7,650
C) $7,065
D) $3,825

A

C) Explanation
Bobby’s total self-employment tax is $7,065 ($50,000 × 0.9235 = $46,175; $46,175 × 0.1530 = $7,065).

LO 5.3.1

446
Q

Ron has the following income for the current tax year:

Schedule C net income: $12,000
Flow-through of general partnership operating income: $15,000
Wages: $80,000
Net income from rental of real estate: $10,000
What is the amount of Ron’s self-employment tax?

A) $3,815
B) $5,228
C) $1,696
D) $9,467

A

A) Explanation
The items included in the computation of self-employment income are the Schedule C income and the flow-through of general partnership operating income. The total of these items is $27,000. This is then reduced by 7.65% of this amount, and then multiplied by 15.3% to give us $3,815. Remember, the 7.65% subtraction always applies. Because the total of the self-employment income and the wages received are less than the wage base, no special computation is necessary.

LO 5.3.1

447
Q

Which of the following is subject to the self-employment tax?

I. Distributive share of limited partnership operating income

II. Flow-through of S corporation income

III. Distributive share of general partnership operating income

IV. Interest or dividends from investments

A) II and IV
B) I, II, III, and IV
C) I and III
D) III only

A

D) Explanation
The general partnership operating income is self-employment income. By definition, the other items of income are not subject to the self-employment tax.

LO 5.3.1

448
Q

Amanda’s share of general partnership net income is $60,000 in 2020. In addition, her distributive share of S corporation income is $10,000. What is Amanda’s self-employment tax, if any, for 2020? Round your answer to the nearest dollar.

A) $9,891
B) $0
C) $10,710
D) $8,478

A

D) Explanation
The distributive share of income from an S corporation is not subject to the self-employment tax.

Actual earnings	$60,000
Less: 7.65%	– (4,590)
Net earnings from self-employment	$55,410
Multiply by	× 15.3%
Self-employment tax	$8,478
LO 5.3.1
449
Q

All of the following are examples of self-employment income for purposes of the self-employment tax except

A) net Schedule F income.
B) income from a sole proprietorship.
C) salary paid to an S corporation shareholder.
D) net Schedule C income.

A

C) Explanation
Wages and salary paid to an S corporation shareholder are not self-employment income.

LO 5.3.1

450
Q

Which of the following is a permissible method for determining the business part of a residence for purposes of the home office expense deduction?

A) Amount of utility use compared to that of the whole house

B) Number of rooms in the house used for business compared to total rooms in the house

C) Accrual method of allocation

D) Relative use method of allocation

A

B) Explanation
Both the relative square footage of the home office area and the relative number of rooms may be used in determining the business part of a home. The amount of utility use can be a deduction, but is not used in determining the business part of a home. There is no recognized method known as the relative use method of allocation.

LO 5.4.1

451
Q

Mark operates a sole proprietorship from his apartment. His gross income for the current tax year is $61,000. Business expenses not associated with his home office total $63,000. Expenses associated with the home office total $4,200.

How much of the home office expense, if any, may Mark deduct for the current year?

A) $2,000
B) $275
C) $0
D) $4,200

A

C) Explanation
The home office expense deduction is limited to the earned income from the business. In other words, the home office expense deduction can generally neither create nor add to a loss. In this situation, the $61,000 of gross income is reduced by the $63,000 of business expenses not associated with the home office, to leave no earned income. Thus, of the $4,200 of home office expenses, none would be deductible in the current year. Note that the entire $4,200 of home office expenses would be subject to a carryforward.

LO 5.4.1

452
Q

Ron, age 43, and Sandy, age 41, are married with two children, Michael, age 12, and Victoria, age 8, who has been blind since her birth. Ron is an architect and general partner with XYZ partnership. Sandy is self-employed as an attorney and works out of a home office. Her home office is exclusively and regularly used for business, and the home office is her principal place of business. Their information for the tax year 2020 is as follows:

AGI: $217,300
Itemized deductions (including qualified residential mortgage interest, taxes paid, and charitable contributions): $33,000
Early in the current year, Sandy’s father died. Sandy is the sole beneficiary of her father’s entire estate. The estate is presently in the probate process. Sandy’s mother, Lisa, age 68, has moved in with them but provides her own support. She was married to Sandy’s father when he died earlier this year.

This is Ron’s second marriage. He makes monthly support payments to his former spouse and his daughter.

Because both Ron and Sandy are considered to be self-employed, they make quarterly estimated tax payments each year to cover both their income tax and self-employment tax obligations.

Based on the information provided in the case scenario for Ron and Sandy, which of the following statements regarding Sandy’s home office deduction is CORRECT?

A) The net income from the business use of Sandy’s home must equal or exceed the business expenses (including depreciation).

B) Sandy is not eligible to deduct her home office expenses.

C) Sandy may deduct her qualifying home office expenses when calculating AGI.

D) The amount Sandy may deduct this year for qualifying home office expenses is unlimited.

A

C) Explanation
Because Sandy is self-employed, she can deduct her qualifying home office expenses when calculating AGI (as a line item on Schedule C). The amount of her home office expense deduction this year is limited to the net income from her business. A profit from the business is not one of the tests for qualification of a home office for tax deduction purposes. Rather, it is one of the limitations on the deductible amount after it qualifies.

LO 5.4.1

453
Q

Mike is involved in the business of breeding show horses on a part-time basis as a sole proprietor. He has shown a significant net loss in this business every year for the last six years, and he has used these losses to reduce the tax liability on the salary he has earned from his law practice.

With which one of the following potential tax traps should Mike be most concerned?

A) Assignment of income
B) Hobby loss
C) Passive activity
D) Substance over form

A

B) Explanation
The fact that this business has not been profitable and is merely a side business increases the likelihood it could be reclassified as a hobby loss. Mike has operated the business only part time, and he has not demonstrated that he is attempting to make it a profitable business. The fact that huge losses have occurred every year overrides the presumption that Mike is in the business to make a profit.

LO 5.4.2

454
Q

Harriet builds remote-control cars as a hobby. Which hobby expenses must she deduct from hobby income first?

A) Cost of goods sold
B) Cost recovery deductions
C) Operating expenses
D) None are deductible

A

A) Explanation
Starting in 2018, due to the passage of the Tax Cuts and Jobs Act (TCJA), most hobby expenses are no longer deductible with the notable exception of cost of goods sold.

LO 5.4.2

455
Q

Phillip’s classic automobile was completely destroyed in an earthquake that was declared a federal disaster. Unfortunately, his insurance paid only $12,000, while the fair market value was $22,000. His basis in the automobile was $17,000. Phillip’s AGI is $35,000.

What is the amount, if any, of Phillip’s deductible casualty loss?

A) $0

B) $6,000

C) $1,400

D) $1,500

A

C) Explanation
The deduction is based on the lesser of basis or the decrease in FMV—reduced by the insurance, the $100 floor per occurrence, and 10% of AGI.

Lesser of decrease in FMV ($22,000) or

adjusted basis ($17,000)

$17,000
Less insurance coverage	(12,000)
$5,000
Less $100 floor	(100)
$4,900
Less 10% of AGI	(3,500)
Deductible loss on Schedule A	$1,400

LO 6.2.3

456
Q

Jimmy owns an apartment building in Cleveland. He would like to exchange his apartment building for another asset in a like-kind exchange. Which of the following assets could Jimmy receive to qualify for like-kind exchange treatment?

A) A piece of manufacturing equipment for his business

B) Residential rental property in Mexico

C) A residence in which he will live

D) Commercial rental property in Detroit

A

D) Explanation
An exchange of an apartment building for commercial rental property may be a like-kind exchange. As long as qualifying real property (property used in a trade or business or held for investment purposes) is exchanged for qualifying real property, the exchange is like-kind.

LO 6.2.2

457
Q

In January of 1996, Lawrence, then age 55, sold his principal residence in Seattle and took advantage of the once-in-a-lifetime exclusion available under Section 121. At that time, the maximum exclusion was $125,000. He excluded his entire gain on the sale, which was $100,000. Later that year, he purchased a new residence in Denver that he used as his principal residence. Earlier this year, he sold the Denver residence for a realized gain of $200,000.

What is the maximum amount of gain, if any, that Lawrence may exclude under Section 121?

A) $200,000

B) $0

C) $175,000

D) $25,000

A

A) Explanation
Use of the once-in-a-lifetime exclusion prior to May 7, 1997, does not preclude use of the full Section 121 exclusion for sales after that date. Thus, Lawrence may exclude the entire gain of $200,000. His gain recognized (the taxable gain) would be $0.

LO 6.2.5

458
Q

Two years ago, Morton purchased equipment (seven-year property) for use in his business at a cost of $12,000. Cost recovery deductions totaled $7,392. The equipment was sold for $13,000.

What is the amount of cost recovery deductions, if any, that must be recaptured?

A) $6,000

B) $7,392

C) $1,392

D) $4,608

A

B) Explanation
The cost basis of the property, $12,000, would be reduced by the cost recovery deductions taken ($7,392). This leaves an adjusted basis of $4,608. When the property subsequently sold for $13,000, the difference between the sales price ($13,000) and the adjusted basis of $4,608 was the gain realized ($8,392). The recapture is the lesser of the gain realized ($8,392) or the cost recovery deductions taken ($7,392). The remaining $1,000 gain is Section 1231 gain.

LO 6.1.3

459
Q

The substitute basis of the qualifying asset received in a like-kind exchange is the asset’s

A) basis increased by the gain realized but not recognized.

B) fair market value increased by the gain realized but not recognized.

C) basis reduced by the gain realized but not recognized.

D) fair market value reduced by the gain realized but not recognized.

A

D) Explanation
The substitute basis of a qualifying asset received in a like-kind exchange is the asset’s fair market value reduced by the gain realized but not recognized. The deferred gain reduces the basis of the acquired asset, such that when that asset is sold, there is a larger gain recognized.

LO 6.2.2

460
Q

During 2020, Stanton purchased several items of depreciable, tangible personalty—with a total cost of $2.71 million—for use in his business. Stanton has taxable (earned) income from his sole proprietorship of $360,000 (without regard to the Section 179 expense). He also has wages from a part-time job of $55,000.

What is the maximum Section 179 expense amount that Stanton may deduct in the current year?

A) $360,000

B) $1,040,000

C) $920,000

D) $415,000

A

D) Achieved
Explanation
The Section 179 expense is first limited by the amount of qualifying property placed in service during the tax year. To the extent that the taxpayer purchases and places into service more than $2.59 million of qualifying property during the tax year, the benefit of the Section 179 election is reduced on a dollar-for-dollar basis. In this situation, Stanton placed an excess $120,000 of property into service. This reduces the maximum election from $1.04 million to $920,000—this is the maximum Section 179 expense of $1.04 million reduced by the dollar amount of property placed into service in excess of $2.59 million. The Section 179 expense election is then limited to the taxpayer’s taxable, or earned, income. Because the business is a sole proprietorship, for purposes of Section 179, salary or wages received as an employee are considered to be from the active conduct of a trade or business. Thus, the total earned income in this situation is $415,000. The maximum Section 179 expense election is $920,000, but for Stanton, the current year deduction is limited to his earned income of $415,000.

LO 6.1.5

461
Q

Carol, a single taxpayer, sold her home in March of the current tax year for $380,000. She had lived in the home as her principal residence for 14 years. She paid a broker’s commission of $20,000. The cost, and basis, of the former home was $60,000. What is the amount of gain recognized by Carol on the sale?

A) $320,000

B) $300,000

C) $70,000

D) $50,000

A

D) Explanation
Remember that the commission paid reduces the gain realized. The gain realized from the sale is $300,000 ($380,000 sale price, reduced by the $20,000 commission and the basis of $60,000). After subtracting the $250,000 Section 121 exclusion, the gain recognized is $50,000.

LO 6.2.5

462
Q

This year, Randy sold a classic painting to his neighbor, Norm, under the following terms:

The price was $22,000, equal to the fair market value.
Randy’s basis in the painting was $5,000.
Starting this year, Norm will pay in five annual installments of $4,000 plus accrued interest.
Norm will make a $2,000 down payment.
Ignoring interest income, what amount of gain will Randy recognize for the current year?

A) $4,500

B) $4,636

C) $3,000

D) $2,000

A

B)
Explanation
The profit on the sale of $17,000 divided by the $22,000 contract price equals a 77.27% gross profit percentage. This is multiplied by the $6,000 of payments received during the year to calculate the amount of gain recognized ($4,636):

$
17,000
 profit
$
22,000
 contract price
=
77.27
%
×
$
6,000
 payments
=
$
4,636
 recognized
LO 6.2.4
463
Q

On June 1, 2017, Hugh and Judy, both age 45, moved into a new house that they had custom built at a cost of $524,000. Because of a slow real estate market in their area, Hugh and Judy were unable to sell their previous home until January 1, 2020. Their previous home, which they had lived in for 15 years, sold for a price of $916,000. Their basis in their old home was $312,000.

How much gain, if any, must Hugh and Judy recognize for tax purposes as a result of this sale?

A) $92,000

B) $604,000

C) $104,000

D) $0

A

C) Explanation
The gain realized is $604,000. This is the difference between the sales price of the residence—$916,000—and the basis of $312,000. They lived in the new home for only 19 months before the old home sold. Thus, for the previous home, they meet the two-out-of-five-year rule allowing for up to a $500,000 exclusion of gain on the sale of a principal residence under Section 121. The realized gain of $604,000, reduced by the $500,000 exclusion, leaves a gain recognized (subject to tax) of $104,000.

LO 6.2.5

464
Q

During 2020, William purchased office furniture for his real estate offices at a total cost of $185,000. Due to a down real estate market, he only has taxable (earned) income of $15,000 from this sole proprietorship (without regard to the Section 179 expense). He took a part-time teaching job and has wages of $5,000.

What is the maximum Section 179 expense amount that William may deduct in the current year?

A) $5,000

B) $15,000

C) $20,000

D) $185,000

A

C) Explanation
The Section 179 deduction is subject to a taxable (earned) income limitation. Because the business is a sole proprietorship, wages received (even from a completely unrelated source) are considered to be from the active conduct of a trade or business. With only $20,000 of earned income, only $20,000 may be used.

LO 6.1.5

465
Q

VUE Corp. entered into an equipment expansion in 2020. All of the equipment purchased totaled $215,000. Corporate taxable income before the election was $25,000. What is the maximum Section 179 expense election available to VUE Corp. in 2020?

A) $0

B) $215,000

C) $190,000

D) $25,000

A

D) Explanation
The maximum Section 179 election amount available to VUE Corp. is $25,000. The election is limited to the lesser of the cost of the asset, the amount of business taxable income, or the annual Section 179 election amount for 2020 ($1,040,000).

LO 6.1.5

466
Q

During the current year, Chuck has Section 1231 gains totaling $14,000. He also has $3,000 of Section 1231 losses. Four years ago, Chuck reported a net Section 1231 loss of $7,000. These are the only two years in which Chuck has had Section 1231 gains or losses.

What is the amount and character of the current year’s Section 1231 gains and losses?

A) $7,000 ordinary income, $4,000 long-term capital gain

B) $11,000 long-term capital gain

C) $4,000 ordinary income, $7,000 long-term capital gain

D) $4,000 long-term capital gain

A

A) Explanation
Of the current year’s $11,000 Section 1231 gain, $7,000 is treated as ordinary income. This is due to the $7,000 of unrecaptured Section 1231 losses during the five-year lookback period. The remaining $4,000 of current Section 1231 gain is treated as long-term capital gain.

LO 6.1.4

467
Q

Suzy, a single taxpayer, sells her residential rental property in March 2020 for $400,000. Suzy acquired the property in July 2010 for $300,000 and has been depreciating it using the straight-line method for realty under MACRS. Assume the amount of depreciation taken is $114,000. Suzy has taxable income of $190,000 and is in the 32% marginal income tax bracket.

What is the amount and character of the gain recognized as a result of the sale?

A) $114,000 gain taxed at 25%, $100,000 gain taxed at 15%

B) $214,000 gain taxed at 15%

C) $214,000 gain taxed at 25%

D) $114,000 gain taxed at 28%, $100,000 gain taxed at 15%

A

A) Explanation
There is $114,000 of unrecaptured Section 1250 income, creating 25% long-term capital gain; and $100,000 of regular long-term capital gain (at taxable income of $190,000, it is 15% rate). Calculate the gain realized and recognized:

Amount realized $400,000
Less adjusted basis (186,000)
Gain realized and recognized $214,000

Calculate the unrecaptured Section 1250 (25% LTCG) income:

Lesser of:
(a) Straight-line depreciation taken $114,000
(b) Gain realized and recognized $214,000
Lesser of these two amounts $114,000

Calculate the regular long-term capital gain:

Gain recognized $214,000
Less unrecaptured Section 1250 income (25% long-term capital gain) (114,000)
Regular long-term capital gain $100,000

In this example, we have total gain recognized (Section 1231 gain) of $214,000, comprised of $114,000 of unrecaptured Section 1250 income and taxed at a maximum rate of 25%, and $100,000 of regular long-term capital gain, eligible for the maximum 15% or 20% long-term capital gain rates. The gain created by the straight-line depreciation on realty is unrecaptured Section 1250 income, subject to a maximum rate of 25%. The gain created by actual appreciation of the asset is regular long-term capital gain under Section 1231.

LO 6.1.4

468
Q

Frank inherited 100 shares of stock from his uncle, Jesse. Jesse purchased the stock eight years ago for $12 per share. The fair market value on the date of Jesse’s death was $9 per share, and the fair market value six months after the date of death was $10 per share.

Assume the administrator elected the alternate valuation date. What is Frank’s per-share basis in the acquired stock?

A) $9.50

B) $12.00

C) $9.00

D) $10.00

A

D) Explanation
The basis of property acquired by inheritance where the administrator elects the alternate valuation date is the fair market value on that alternate valuation date. Thus, in this situation, the fair market value of $10 as of the alternate valuation date would be Frank’s basis in the inherited property.

LO 6.1.1

469
Q

Bob is involved in a like-kind exchange. In the exchange, he assumes a mortgage of $15,000, is relieved of a mortgage of $26,000, and receives $7,000 in cash. How much boot did Bob receive in the transaction?

A) $7,000

B) $18,000

C) $33,000

D) $11,000

A

B) Explanation
The net debt relief is considered to be boot, as is the cash received. The net debt relief of $11,000 plus the cash received of $7,000 equals $18,000.

LO 6.2.2

470
Q

During the current year, Peter has Section 1231 gains totaling $8,000. He also has $1,000 of Section 1231 losses. Four years ago, Peter reported a net Section 1231 loss of $2,000. These are the only two years in which Peter has had Section 1231 gains or losses.

What is the amount and character of the current year’s Section 1231 gains and losses?

A) $7,000 ordinary income

B) $7,000 long-term capital gain

C) $2,000 ordinary income, $5,000 long-term capital gain

D) $5,000 ordinary income, $2,000 long-term capital gain

A
C) Explanation
Section 1231 gains	$8,000
Less Section 1231 losses	(1,000)
Current year net Section 1231 gain	$7,000
Lesser of:	
(a) Unrecaptured net Section 1231

losses claimed during the lookback

period

$2,000
(b) Current year’s net Section 1231

gain

$7,000
Current year’s net Section 1231 gain

characterized as ordinary income

$2,000

LO 6.1.4

471
Q

Several years ago, Jack purchased a duplex to use as a rental property. The cost of the duplex was $79,500. He paid an attorney $800 to draft the legal documents related to the purchase. He paid $7,500 for improvements in the first year and this year paid $1,900 for various repairs to the property. He claimed a first-year cost recovery deduction of $3,000.

What is Jack’s adjusted basis in the duplex?

A) $82,900

B) $87,800

C) $84,800

D) $82,100

A

C) Explanation
The cost of the duplex of $79,500 is increased by the capitalized costs (i.e., $800 for the attorney’s fees and $7,500 for improvements). This amount would be reduced by the cost recovery deduction of $3,000, leaving an adjusted basis of $84,800. The repairs of $1,900 represent a current deduction and therefore do not impact the basis of the property.

LO 6.1.2

472
Q

Anne is the owner of 1,000 shares of the GRQ Mutual Fund Group. Recently, because of favorable market conditions, she sold 1,000 shares of the fund for a market price well in excess of her adjusted cost basis. She is now interested in computing her gain on these shares. Anne uses the specific identification method to determine basis.

Which of the following is an income tax implication for Anne in the sale of her mutual fund shares?

A) The gain is computed using the excess of the fair market value over the cost of those shares first purchased.

B) The gain is computed using the excess of the fair market value over the cost of those shares last purchased.

C) The gain is computed by subtracting the documented basis of identified shares from their sale price.

D) The gain is computed according to the average cost of all shares as compared to the total sale proceeds.

A

C) Explanation
When using the specific identification method of computing basis in the sale of mutual fund shares, it is necessary to identify the shares being sold based on their purchase date. The documented basis of those shares is then used to compute the gain (or loss) realized.

LO 6.2.1

473
Q

In February, Perry purchased a new computer (five-year property) for use in his business. The cost of the computer was $4,300, while freight and setup charges totaled $600. What is the first-year cost recovery deduction using the MACRS table?

A) $430

B) $490

C) $980

D) $790

A

C) Explanation
The freight and setup charges of $600 must be capitalized—that is, added to the cost of the computer—to give a total basis of $4,900. The first-year deduction using the MACRS table is 20%. Thus, 20% times $4,900 equals $980. Please note that the MACRS table has all applicable conventions already built into the table; therefore, no adjustment is necessary for the half-year convention. Note that bonus depreciation is not used, as the fact pattern states to use the MACRS table.

LO 6.1.3

474
Q

Billy received 100 shares of stock from his uncle, Ray. Ray purchased the stock eight years ago for $12 per share. The fair market value on the date of Ray’s death was $9 per share, and the fair market value six months after the date of death was $10 per share.

Assume that Billy inherited the stock and that the administrator did not elect the alternate valuation date. What is Billy’s per-share basis in the acquired stock?

A) $10.00

B) $9.50

C) $12.00

D) $9.00

A

D) Explanation
The basis of property acquired by inheritance is simply the fair market value on the date of the decedent’s death. In this case, that value was $9 per share.

LO 6.1.2